Instructional Material For MATH20063 Differential Equations 1

You might also like

Download as pdf or txt
Download as pdf or txt
You are on page 1of 119

Republic of the Philippines

POLYTECHNIC UNIVERSITY OF THE PHILIPPINES


Office of the Vice President for Academic Affairs
Santa Rosa Campus

INSTRUCTIONAL MATERIAL FOR


DIFFERENTIAL EQUATIONS
(MATH 20063)

COMPILED BY:

KENNETH D. DUGAY, LPT

July, 2020

1
Foreword

Hello my dear students! Congratulations on passing your first two Calculus subjects in College
(Differential and Integral). Now we continue your Calculus journey by using the concepts of your previous
learnings to solve more useful and exciting problems with more applications.

Our situation is far from the common or typical learning approach. Truly, this pandemic brought
many inconveniences not only in our education system but also in our typical day-to-day life. But I don’t
want to just look at the negativities in life. As my life verse stated,
“”And we know that all things work together for good to those who love God, to those who are the
called according to His purpose.” – Romans 8:28
There is a purpose for everything. It may be for us to grow and realize some things in our lives.
But in the sense of our education system, our troubles pushed us teachers to “reinvent the wheel” of
learning. Maybe this is the time to look at the strategies that may work and those that are needed to be
improved. This may be our chance to step out of the “traditional” way.
On the other way around, these sentiments should not only be applied to us teachers but to the
students as well. I know that we are plunged into this situation with little to no warning. You were not
given the proper transition period required to prepare yourself not only with the equipment but also
mentally. But it is what it is. Learning must continue as we humans are lifelong learners.
This module is specially designed for three different sets of learners: Fully-online, Partially-online
and Not Online. Lecture videos and solved problems were uploaded in my YouTube channel: STEM
Teacher PH. Just look for the playlist about Differential Equations there. This also contains text
explanations of the concepts for those students who do not have the device or the connection to play
those videos. You may select the mode of learning that is suitable to your situation.
The module contains step-by-step explanation for the concepts in our subject. But of course,
since this subject is for those who passed both the previous Calculus subjects, we will not dwell deep on
some differentiation and integration processes. As per your other Math subjects, strong foundation of
Algebraic processes like factoring, laws of exponents and radicals, and rules of logarithm must be
possessed.
That’s it for the introduction. I hope that you will also find this subject as interesting as I do. As I
always put in my social media posts - #KeepOnLearning

Kenneth D. Dugay, LPT


Subject Facilitator

i
Table of Contents

Unit 1: Introduction to Differential Equations


Lesson 1 – Basic Concepts of Differential Equations……………………………………………………1
Lesson 2 – Solutions Leading to Differential Equations…………………………………..……………..5
Unit 2: Methods for Solving Equations of Order One
Lesson 1 – Separation of Variables………………………………………………………………………13
Lesson 2 – Homogeneous Equations……………………………………………………………………23
Lesson 3 – Linear Coefficient in Two Variables…………………………………………………………34
Unit 3: Equations involving Integrating Factors
Lesson 1 – Exact Equations………………………………………………………………………………42
Lesson 2 – Integrating Factor by Formula………………………………………………………….……52
Lesson 3 – Linear Equations……………………………………………………………………...………62
Lesson 4 – Bernoulli Equations…………………………………………………………...………………71
Lesson 5 – Substitution Method……………………………………………………………..……………78
Unit 4: Applications of Differential Equations of Order One………………………………………85
Unit 5: Differential Operators
Lesson 1 – Properties of Differential Operators…………………………………………………………99
Lesson 2 – Solutions of Homogeneous Linear Equations with Real Roots…………………..…....104
Lesson 3 – Solutions of Homogeneous Linear Equations with Imaginary Roots………….…...….110
References……………………………………………...……………………………………………………….115

ii
Assessment Instructions

This area is designated to explain the process of how your performance in this subject will be
assessed. At the end of each lesson, there are problems that you will be solving. You will be providing for
the value(s) being asked in the “Activities/Assessment” portion of this module.
The instruction is meant to be both synchronous and asynchronous. There will be an allotted time
for an online class using video conferencing software. It is suggested that the class will meet every other
week. Asynchronous method implies that you can work on the contents and the required activities at your
own timing within the given deadline by the instructor, usually before the next synchronous session. It
means that you have 2 weeks to work on the requirements so your time wisely and do not procrastinate.
The activities can be encoded in MS Word or it can be answered in any WHITE paper. These
activities can be submitted in two ways. First is through the “Submit Assignment” feature of Google
Classroom. The class code will be given during the orientation. For those students who do not have
devices or access to internet connection, you may schedule an appointment with me to submit your work
in the school. This will be done while following proper physical distancing protocols.
There will be two long quizzes for every grading period. It will be given during the morning of the
scheduled quiz and it will be submitted before the end of the day. For the examinations, a scheduled 3-
hour period will be allotted for the students to finish it. Another 30 minutes extension will then be given for
the students to compile their work. They may take a picture then submit it through Google Classroom. If
technical difficulties occur, you may inform me ahead of time so that I can give proper considerations.
Those students who do not have internet connection may ask me for an appointment so that we can
schedule you quiz or exam ahead of time.

iii
Polytechnic University of the Philippines
Santa Rosa Campus

Unit 1: Introduction to Differential Equations

Overview
Everything around us is subject to change. Differential Equations (DE) as a subject is mostly
concerned about these phenomena. We can see the effect of these changes concerning one or many
variables to a dependent variable.

Differential Equations has many applications in Physics, Chemistry and Engineering. We can
create more comprehensive Mathematical models and predictions to understand better the things that are
happening and may happen to us. One specific and relevant application of DE is the SIR Model or the
Susceptible-Infectious-Recovered Model in epidemiology. Differential Equations can be used to produce
a model to predict the spread of an infectious disease like COVID-19.

For us to understand more about Differential Equations, we will also talk about something that is
more familiar to us which is the functions. Solving for a DE will yield to a solution that is in a form of a
function. But for this unit, we will be using a method of obtaining a DE given its solution. Then we will
explore some families of curves usually discussed in Analytic Geometry (Pre-Calculus for SHS STEM
students) and see what their DE will look like.

Lesson 1
Basic Concepts of Differential Equations

Learning Objective
After successful completion of this module, you should be able to:

▪ recall the basic notations for derivatives;


▪ know the forms of differential equations;
▪ determine the kinds of differential equations by observation; and
▪ identify the order and degree of any given differential equations.

Course Materials
Lecture Video

(Click the picture for the link or type in: https://bit.ly/IntroToDE )

Compiled by: Kenneth D. Dugay, LPT |1


Polytechnic University of the Philippines
Santa Rosa Campus

Discussion

Notations

Before you proceed with the discussion, let us first recall the notations used for derivatives:

First Derivative Second Derivative Third Derivative


𝑑𝑦 𝑑2 𝑦 𝑑3 𝑦
𝑑𝑥 𝑑𝑥 2 𝑑𝑥 3

𝑦′ 𝑦 ′′ 𝑦 ′′′

𝑓 ′ (𝑥) 𝑓 ′′ (𝑥) 𝑓 ′′′ (𝑥)

𝑦̇ 𝑦̈ 𝑦⃛

These are the notations that are used to show the derivative of y or a function of x with respect to
x. Different books and resources are using and interchanging these notations so I decided to show it to
you so that you will not be confused when you check out different available materials.

Kinds of Differential Equations

1) Ordinary Differential Equation - unknown function (dependent variable usually y) depends on only
one independent variable

Examples of Ordinary Differential Equations:

𝑑𝑦
(1) = 5𝑥 + 3
𝑑𝑥
𝑑2 𝑦 𝑑𝑦
(2) 𝑥 𝑑𝑥 2 + 2 𝑑𝑥 = 1
4 2
𝑑3 𝑦 𝑑4 𝑦
(3) 4 (𝑑𝑥 3 ) + 𝑠𝑖𝑛𝑥 (𝑑𝑥 4 ) + 5𝑥𝑦 = 5𝑥 +3
4 2
𝑑3 𝑦 𝑑5 𝑦 𝑑𝑦 7
(4) (𝑑𝑥 3 ) + 3 (𝑑𝑥 5 ) + 𝑦 3 (𝑑𝑥 ) = 6

All of the DE above are ordinary since the value of our dependent variable y depends only on one
independent variable x. This is regardless of taking its first derivative, second derivative, third derivative,
and so on. We can also observe that our we are only getting the derivative of y, dy, with respect to ONLY
x, or dx.

2) Partial Differential Equation - unknown function depends on two or more independent variables

Example of Partial Differential Equations:

𝜕2 𝑧 𝜕2 𝑧
𝜕𝑦 2
− 𝜕𝑥 2 = 0

Compiled by: Kenneth D. Dugay, LPT |2


Polytechnic University of the Philippines
Santa Rosa Campus

This DE is considered partial not only because of the notation for partial derivative 𝜕 but also
because we are taking the derivative of our dependent variable z in our example with respect to both y
and x. In this case, we are given with one dependent variable and two independent variables.

Order and Degree

1. Order - of a DE is the order of highest derivative appearing in the equation.


2. Degree - of a DE that can be written as polynomial in the unknown function is the power to which the
highest-order derivative is raised.

Examples:

Determine the order and degree of the given differential equation:

𝑑𝑦
(1) = 5𝑥 + 3
𝑑𝑥

𝑑𝑦
We only have one term containing derivatives in this equation. Looking at , it is only getting the
𝑑𝑥
first derivative, so its order is 1. Since that term is the highest order then it is only raised to 1 (no other
𝑑𝑦
exponents in ), then its degree is also 1.
𝑑𝑥

𝑑2 𝑦 𝑑𝑦
(2) 𝑥 𝑑𝑥 2 + 2 𝑑𝑥 = 1

Here, we will be considering the two terms with derivatives. Then we select the one with the
𝑑2 𝑦
highest order which is
𝑑𝑥 2
. Since it is on its second derivative, then its order is 2. Its degree is only 1
since its exponent is only 1.

4 2
𝑑3 𝑦 𝑑4 𝑦
(3) 4 (𝑑𝑥 3 ) + 𝑠𝑖𝑛𝑥 (𝑑𝑥 4 ) + 5𝑥𝑦 = 5𝑥 + 3

We have two terms containing derivative here, first is on its third derivative and the other is on its
2
𝑑4 𝑦
fourth derivative. Then we select the highest order which is ( 4 ) . So its order is 4 and its degree is 2
𝑑𝑥
since it is raised to 2.

4 2
𝑑3 𝑦 𝑑5 𝑦 𝑑𝑦 7
(4) (𝑑𝑥 3 ) + 3 (𝑑𝑥 5 ) + 𝑦 3 (𝑑𝑥 ) = 6

This example now contains three terms with derivatives. But the middle term is on its 5 th
derivative while the others is only on their third and first derivative. So we select that term and it will give
us a degree of 5. Then let us look at its exponent. Since it has an exponent of 2, then the degree of this
DE is 2.

𝜕2 𝑧 𝜕2 𝑧
(5) 𝜕𝑦 2
− 𝜕𝑥 2 = 0

Compiled by: Kenneth D. Dugay, LPT |3


Polytechnic University of the Philippines
Santa Rosa Campus

Even if this is a partial DE, we can still apply the definitions for order here. Both of our terms are
on their second derivative, so its order is 2. But since we cannot represent our DE as a polynomial, it has
NO degree.

Note: Not all differential equation has a degree

𝑑2 𝑦 𝑑𝑦
𝑒 𝑦 𝑑𝑥 2 + 2 𝑑𝑥 = 1 We cannot represent this DE as a polynomial function because of 𝑒 𝑦 , so it has
no degree.
1
𝑑𝑦 2 1
(𝑑𝑥 ) +𝑦 =𝑥 This DE has an order of 1 but it has no degree because of the exponent .
2
Degree should be a positive integer

Activities/Assessment
Determine the order and degree of the following Differential Equations:

𝑑𝑦
(1) 𝑑𝑥 + 𝑦 = 𝑥

3
𝑑2 𝑟 𝑑𝑟 2 𝑑𝑟
(2) (𝑑𝑡 2 ) + (𝑑𝑡 ) + 𝑦 𝑑𝑡 = 0

(3) 𝑥 4 𝑦 ′′′ + 𝑥𝑦 ′′ = 𝑒 𝑥

(4) 𝑥(𝑦′′′)2 − 𝑥 2 𝑦 ′′ − 𝑥 3 (𝑦 ′ )5 − 𝑦 = 𝑥

𝑑𝑛 𝑥
(5) 𝑑𝑦 𝑛 = 𝑦 2 + 1

Compiled by: Kenneth D. Dugay, LPT |4


Polytechnic University of the Philippines
Santa Rosa Campus

Lesson 2
Solutions Leading to Differential Equations

Learning Objective
After successful completion of this module, you should be able to:

▪ identify the steps needed to transform a function to a differential equation;


▪ obtain the differential equation of a solution by elimination of arbitrary constants; and
▪ solve the differential solutions given the families of curves

Course Materials
Lecture Videos

Elimination of Arbitrary Constants

Click: Solved Problem #1 for Elimination of Arbitrary Constants: https://bit.ly/ElimConstants1

Solved Problem #2 for Elimination of Arbitrary Constants: https://bit.ly/ElimConstants2

Solved Problem #3 for Elimination of Arbitrary Constants: https://bit.ly/ElimConstants3

Solved Problem #4 for Elimination of Arbitrary Constants: https://bit.ly/ElimConstants4

Differential Equations of Families of Curves

Click: Solved Problem #1 for DE of Families of Curves: https://bit.ly/FamCurves1

Solved Problem #2 for DE of Families of Curves: https://bit.ly/FamCurves2

Solved Problem #3 for DE of Families of Curves: https://bit.ly/FamCurves3

Discussion

Elimination of Arbitrary Constants

If we are given with function, one of the simplest ways to solve for its differential equation is by
elimination of arbitrary constants. During integral calculus class, when we are solving for definite
integrals, we are used to putting that + c in our final answer. Now we must remove that constant for us to
arrive at the DE and it is possible to encounter more than one constant.

As a note, a differential equation may lead to many different solutions. So we can say that our
given function may be only one of the may solutions that can be derived from the DE. We can eliminate
our arbitrary constant/s by taking the derivative of our original function then using Algebraic methods for
simultaneous equations like elimination or substitution. Most of the time, we will be taking the derivative of
our function depending on the number of arbitrary constants. One constant means we have to take the
derivative once. If we are given with two constants, we will be solving for the derivative twice, and so on.

Compiled by: Kenneth D. Dugay, LPT |5


Polytechnic University of the Philippines
Santa Rosa Campus

Example 1. Find the differential equation whose solution is given by

𝑦 2 = 4𝑎𝑥

By Power Rule of Differentiation:


𝑑 𝑛
𝑥 = 𝑛𝑥 𝑛−1
𝑑𝑥
𝑑𝑦
2𝑦 = 4𝑎
𝑑𝑥

Solving for the value of a:

𝑦 𝑑𝑦
( ) =𝑎
2 𝑑𝑥

Substituting the value of a to our given equation:

𝑦 𝑑𝑦
𝑦 2 = 4 [ ( )] 𝑥
2 𝑑𝑥

Simplifying further:

𝑑𝑦
𝑦 2 = 2𝑥𝑦
𝑑𝑥
𝑑𝑦
𝑦 = 2𝑥
𝑑𝑥

𝑦 𝑑𝑥 − 2𝑥 𝑑𝑦 =0

Example 2. Find the differential equation whose solution is given by

𝑥𝑦 2 − 1 = 𝑐𝑦

𝑥𝑦 2 −1
= 𝑐 (1)
𝑦

By Product Rule of Differentiation:


𝑑𝑦 𝑑𝑦
𝑥(2𝑦) + 𝑦2 = 𝑐
𝑑𝑥 𝑑𝑥

2𝑥𝑦 𝑑𝑦 + 𝑦 2 𝑑𝑥 = 𝑐 𝑑𝑦

Substituting the value of c in (1):

𝑥𝑦 2 −1
2𝑥𝑦 𝑑𝑦 + 𝑦 2 𝑑𝑥 = 𝑑𝑦
𝑦

Simplifying further:

2𝑥𝑦 2 𝑑𝑦 + 𝑦 3 𝑑𝑥 = (𝑥𝑦 2 − 1)𝑑𝑦

Compiled by: Kenneth D. Dugay, LPT |6


Polytechnic University of the Philippines
Santa Rosa Campus

𝑦 3 𝑑𝑥+2𝑥𝑦 2 𝑑𝑦 − (𝑥𝑦 2 − 1)𝑑𝑦 = 0

𝑦 3 𝑑𝑥 + (2𝑥𝑦 2 − 𝑥𝑦 2 + 1) 𝑑𝑦 = 0

𝑦 3 𝑑𝑥 + (𝑥𝑦 2 + 1) 𝑑𝑦 = 0

Example 3. Find the differential equation whose solution is given by

𝑦 = 𝑐1 𝑒 −𝑥 + 𝑐2 𝑒 2𝑥

Since we have two constants, we will be taking the derivative of our function twice.

𝑦 = 𝑐1 𝑒 −𝑥 + 𝑐2 𝑒 2𝑥 (1)

𝑦′ = −𝑐1 𝑒 −𝑥 + 2𝑐2 𝑒 2𝑥 (2)

𝑦′′ = 𝑐1 𝑒 −𝑥 + 4𝑐2 𝑒 2𝑥 (3)

Eliminating 𝑐1 in (1) and (2):

𝑦 = 𝑐1 𝑒 −𝑥 + 𝑐2 𝑒 2𝑥

𝑦 ′ = −𝑐1 𝑒 −𝑥 + 2𝑐2 𝑒 2𝑥

𝑦 + 𝑦′ = 3𝑐2 𝑒 2𝑥 (4)

Eliminating 𝑐1 in (2) and (3):

𝑦′ = −𝑐1 𝑒 −𝑥 + 2𝑐2 𝑒 2𝑥

𝑦 ′′ = 𝑐1 𝑒 −𝑥 + 4𝑐2 𝑒 2𝑥

𝑦′ + 𝑦 ′′ = 6𝑐2 𝑒 2𝑥 (5)

Eliminating 𝑐2 by 2(4) − (5):

2𝑦 + 2𝑦 ′ = 6𝑐2 𝑒 2𝑥

−𝑦 ′ − 𝑦 ′′ = −6𝑐2 𝑒 2𝑥

2𝑦 + 𝑦 ′ − 𝑦′′ = 0

Rearranging the terms:

𝑦 ′′ − 𝑦 ′ − 2𝑦 = 0

Compiled by: Kenneth D. Dugay, LPT |7


Polytechnic University of the Philippines
Santa Rosa Campus

Another Solution by Determinants for System of Three Equations:

The DE of our equation can also be solved by using determinants. Let us use the solved values
of the derivatives of our function.

−𝑦 + 𝑐1 𝑒 −𝑥 + 𝑐2 𝑒 2𝑥 = 0 (1)

−𝑦′ − 𝑐1 𝑒 −𝑥 + 2𝑐2 𝑒 2𝑥 = 0 (2)

−𝑦 ′′ + 𝑐1 𝑒 −𝑥 + 4𝑐2 𝑒 2𝑥 = 0 (3)

Factoring out the common terms on each equation then putting the terms left behind as elements
of the matrix:

1 𝑐1 𝑒 −𝑥 𝑐2 𝑒 2𝑥

−𝑦 1 1
⌊ −𝑦′ −1 2⌋ = 0
−𝑦′′ 1 4

Copying the values in column 1 then putting it at column 4, and also copying the values in column
2 then putting it at column 5.

−𝑦 1 1 −𝑦 1
⌊ −𝑦′ −1 2 −𝑦′ −1 ⌋ = 0
−𝑦′′ 1 4 −𝑦′′ 1

Then multiplying the three values in a “diagonal” starting from the Row 1, Column 1 element then
adding the results to the products of the diagonals in R1C2 and R1C3 elements (follow the green lines).
For the diagonal elements in the opposite direction, we will be taking the negative of their products (follow
the orange lines)

−𝑦 1 1 −𝑦 1
⌊ −𝑦′ −1 2 −𝑦′ −1 ⌋ = 0
−𝑦′′ 1 4 −𝑦′′ 1

It will result to this:

(−𝑦)(−1)(4) + (1)(2)(−𝑦 ′′ ) + (1)(−𝑦 ′ )(1) − (1)(−1)(−𝑦 ′′ ) − (−𝑦)(2)(1) − (1)(−y ′ )(4) = 0

4𝑦 − 2𝑦 ′′ − 𝑦 ′ − 𝑦 ′′ + 2𝑦 + 4y ′ = 0

Then simplifying the equation:

−3𝑦 ′′ + 3𝑦 ′ + 6𝑦 = 0

−𝑦 ′′ + 𝑦 ′ + 2𝑦 = 0

𝑦 ′′ − 𝑦 ′ − 2𝑦 = 0

We arrive at a shorter solution to solve for the differential equation of our solution using determinants.

Compiled by: Kenneth D. Dugay, LPT |8


Polytechnic University of the Philippines
Santa Rosa Campus

Example 4. Find the differential equation whose solution is given by

𝑦 = 𝑐1 𝑒 𝑥 + 𝑐2 𝑥𝑒 𝑥

Rearranging the equation first:

−𝑦 + 𝑐1 𝑒 𝑥 + 𝑐2 𝑥𝑒 𝑥 = 0 (1)

Taking its first derivative by applying product rule of differentiation on the third term:

−𝑦 ′ + 𝑐1 𝑒 𝑥 + 𝑐2 𝑥𝑒 𝑥 + 𝑐2 𝑒 𝑥 = 0

−𝑦 ′ + 𝑐1 𝑒 𝑥 + 𝑐2 𝑒 𝑥 (𝑥 + 1) = 0 (2)

Solving for the second derivative:

−𝑦 ′′ + 𝑐1 𝑒 𝑥 + 𝑐2 𝑒 𝑥 + 𝑐2 𝑒 𝑥 (𝑥 + 1) = 0

−𝑦 ′′ + 𝑐1 𝑒 𝑥 + 𝑐2 𝑒 𝑥 (1 + 𝑥 + 1) = 0

−𝑦 ′′ + 𝑐1 𝑒 𝑥 + 𝑐2 𝑒 𝑥 (𝑥 + 2) = 0 (3)

Then taking out the common terms in (1), (2), and (3) to form the matrix, then solving:

1 𝑐1 𝑒 𝑥 𝑐2 𝑒 2𝑥

−𝑦 1 𝑥 −𝑦 1
⌊ −𝑦′ 1 𝑥+1 −𝑦′ 1⌋=0
−𝑦′′ 1 𝑥 + 2 −𝑦′′ 1

(−𝑦)(𝑥 + 2) + (−𝑦 ′′ )(𝑥 + 1) + (−𝑥𝑦 ′ ) − (−𝑥𝑦 ′′ ) − (−𝑦)(𝑥 + 1) − (−𝑦 ′ )(𝑥 + 2) = 0

Simplifying further:

𝑦 ′′ (−𝑥 − 1 + 𝑥) + 𝑦 ′ (−𝑥 + 𝑥 + 2) + 𝑦(−𝑥 − 2 + 𝑥 + 1) = 0

−𝑦 ′′ + 2𝑦 ′ − 𝑦 = 0

𝑦 ′′ − 2𝑦 ′ + 𝑦 = 0

Compiled by: Kenneth D. Dugay, LPT |9


Polytechnic University of the Philippines
Santa Rosa Campus

Families of Curves

Now that we managed to solve for the differential equation of a function, let us apply those
concepts to solve for the DE of families of curves. This section of the module requires you to recall certain
concepts in Pre-Calculus (or Analytic Geometry) for we need them to arrive at our initial function.

Example 1. Find the differential equation of the family of lines passing through the origin.

Let us start with the slope-intercept form of the equation of lines:

𝑦 = 𝑚𝑥 + 𝑏

Since the line is passing through the origin, its y-intercept (b) is
equal to zero.
Then we can reduce our equation to:

𝑦 = 𝑚𝑥

Then taking its derivative:

𝑑𝑦
=𝑚
𝑑𝑥

Substituting the value of m to our given equation:

𝑑𝑦
𝑦= 𝑥
𝑑𝑥

Now, simplifying:

𝑦𝑑𝑥 − 𝑥𝑑𝑦 = 0

Example 2. Obtain the differential equation of the family of circles with centers at the x-axis

Let us consider the standard equation of a circle at center (h,k)

(𝑥 − ℎ)2 + (𝑦 − 𝑘)2 = 𝑟 2

Since the center of our circle is at x-axis, then its y-coordinate is


always equal to zero. With that, our k=0. This will lead to:

(𝑥 − ℎ)2 + 𝑦 2 = 𝑟 2

Taking its derivative using Chain Rule and Power Rule of


Differentiation then simplying:

2(𝑥 − ℎ) + 2𝑦𝑦′ = 0

2𝑥 − 2ℎ + 2𝑦𝑦′ = 0

Compiled by: Kenneth D. Dugay, LPT | 10


Polytechnic University of the Philippines
Santa Rosa Campus

𝑥 − ℎ + 𝑦𝑦′ = 0

Now getting its second derivative:

1 + 𝑦𝑦 ′′ + 𝑦 ′ 𝑦′ = 0

𝑦𝑦 ′′ + (𝑦′)2 + 1 = 0

Example 3. Obtain the differential equation of the family of ellipses having its centers at the origin and
transverse axis x.

The standard equation for ellipses with center at the origin and
transverse axis x is:

𝑥 2 𝑦2
+ =1
𝑎2 𝑏 2

Solving for its first derivative:

2𝑥 2𝑦𝑦′
+ 2 =0
𝑎2 𝑏

𝑥 𝑦𝑦′
2
=− 2
𝑎 𝑏

1 𝑦𝑦 ′
= − (1)
𝑎2 𝑏2𝑥

Next, we solve for the second derivative:

1 𝑦𝑦 ′′ + (𝑦 ′ )2
2
=− (2)
𝑎 𝑏2
1
Substituting the values of from (1) and (2):
𝑎2

𝑦𝑦′ 𝑦𝑦 ′′ + (𝑦′)2
2
=−
𝑏 𝑥 𝑏2

Then multiplying both sides of the equation by 𝑏 2 to eliminate the constant:

𝑦𝑦′
= 𝑦𝑦 ′′ + (𝑦′)2
𝑥

𝑥[𝑦𝑦 ′′ + (𝑦 ′ )2 ] − 𝑦𝑦 ′ = 0

Compiled by: Kenneth D. Dugay, LPT | 11


Polytechnic University of the Philippines
Santa Rosa Campus

Activities/Assessment
A. Obtain the differential equation of the given solution by eliminating the arbitrary constant(s):

(1) 3𝑥 2 − 𝑥𝑦 2 = 𝑐

(2) 𝑦 = 𝛼 cos(𝑥 + 𝛽)

(3) 𝑦 = 𝑐1 𝑒 −2𝑥 + 𝑐2 𝑒 2𝑥

(4) 𝑦 = 𝑥 2 + 𝑐1 𝑒 2𝑥 + 𝑐2 𝑒 −𝑥

(5) 𝑦 = 𝑐1 𝑒 2𝑥 cos 3𝑥 + 𝑐2 𝑒 2𝑥 sin 3𝑥

B. Solve for the differential equation of the following families of curves:

(1) Family of parabolas with axis of symmetry at y-axis

(2) Straight lines with slope and x-intercept equal

(3) Family of parabolas with vertex and focus on the x-axis, opening to the right

Compiled by: Kenneth D. Dugay, LPT | 12


Polytechnic University of the Philippines
Santa Rosa Campus

Unit 2: Methods for Solving Equations of Order One

Overview
Differential Equations are interesting for there are certain considerations to think about before
solving it. One of which is the order. A huge part of our module will be talking about methods of solving
order one DE and we will start here. The first method to solve a DE of order one that we will discuss is
separation of variables. We will be employing a lot of rules and techniques of integration so make sure
that you recall the concepts from your previous subject.

A differential equation may be classified into different forms. We will then be talking about the
second form of order one differential equation which is the homogeneous equations. Our process will be
longer than our prior method because some initial steps will be added to our process of solving for the
solution.

Then we will discuss the third method to solve order one DE – Linear Coefficients in Two
Variables. But unlike the first two methods where there are certain steps to test the applicability of the
method, this form can be tested by mere observation. Even so, this is compensated by the long process
that we need to go through to solve the solution of the DE.

Lesson 1
Separation of Variables

Learning Objective
After successful completion of this module, you should be able to:

▪ identify the form of order-one differential equations;


▪ test if a differential equation is separable; and
▪ solve for the solution of a separable differential equation.

Course Materials
Lecture Video

(Click the picture for the link or type in: https://bit.ly/SepVariable)

Compiled by: Kenneth D. Dugay, LPT | 13


Polytechnic University of the Philippines
Santa Rosa Campus

Click: Solved Problem #1 for Separation of Variables: https://bit.ly/SepVariable1

Solved Problem #2 for Separation of Variables: https://bit.ly/SepVariable2

Solved Problem #3 for Separation of Variables: https://bit.ly/SepVariable3

Solved Problem #4 for Separation of Variables: https://bit.ly/SepVariable4

Solved Problem #5 for Separation of Variables: https://bit.ly/SepVariable5

Discussion

Order One Differential Equations

From our previous discussion, we talked about elimination of arbitrary constant(s) where we are
given a solution then we arrive at its DE. But onwards, we will be having a DE and we will be solving for
its solution. Here, we will be talking about order one differential equations.

Differential equation of order one may be written as:

𝑀(𝑥, 𝑦)𝑑𝑥 + 𝑁(𝑥, 𝑦)𝑑𝑦 = 0

𝑀(𝑥, 𝑦) + 𝑁(𝑥, 𝑦)𝑦′ = 0

𝑀(𝑥, 𝑦)
𝑦′ = =0
𝑁(𝑥, 𝑦)

where both M(x,y) and N(x,y) are functions of x and and y.

Notice that we only have dx and dy or y’. It is because our concern at this point in our module is
just order one DE. We will be employing different techniques and methods to solve for it. What is unique
about this type of DE is before we can solve it, we have to do different tests. We must know it what we will
be applying is appropriate to our given order one DE.

Separation of Variable

Let us start with the easiest way to solve order one DE which is the separation of variable.

Given an order one DE in the form:

𝑀(𝑥, 𝑦)𝑑𝑥 + 𝑁(𝑥, 𝑦)𝑑𝑦 = 0

It is a separable DE if it can be reduced to the form:

𝐴(𝑥)𝑑𝑥 + 𝐵(𝑦)𝑑𝑦 = 0

This means that if we can manipulate our given function so that our M(x,y) and N(x,y) will be transformed
as A(x) and B(y), then our equation is separable. To explain further, strictly a function of x must be
multiplied to dx and a function of y must be multiplied to dy.

Compiled by: Kenneth D. Dugay, LPT | 14


Polytechnic University of the Philippines
Santa Rosa Campus

Then the solution can be solved by taking the integral of the whole equation:

∫ 𝐴(𝑥)𝑑𝑥 + ∫ 𝐵(𝑦)𝑑𝑦 = 𝑐

As a practice, let us check if the following equations are separable:

𝑑𝑦 𝑥
(1) =−
𝑑𝑥 𝑦

Since our equation to be separable should follow the form 𝐴(𝑥)𝑑𝑥 + 𝐵(𝑦)𝑑𝑦 = 0, let us manipulate our
equation as follows:

𝑥
𝑑𝑦 = − 𝑑𝑥
𝑦

𝑦𝑑𝑦 = −𝑥𝑑𝑥

𝑥𝑑𝑥 + 𝑦𝑑𝑦 = 0

Since the term beside dx is only x and it is a function of x, and the term beside dy is just y which is a
function of y, then our equation follows the form 𝐴(𝑥)𝑑𝑥 + 𝐵(𝑦)𝑑𝑦 = 0. Therefore, our DE is separable.

Following the same procedure for the following separable DE:

𝑑𝑦
(2) = 𝑥 2𝑦
𝑑𝑥

1
𝑑𝑦 = 𝑥 2 𝑑𝑥
𝑦
1
𝑥 2 𝑑𝑥 − 𝑑𝑦 = 0 (DE is separable)
𝑦

𝑑𝑦
(3) =𝑦+1
𝑑𝑥
1
𝑑𝑦 = 𝑑𝑥
𝑦+1
1
𝑑𝑥 − 𝑑𝑦 = 0 (DE is separable)
𝑦+1

𝑑𝑦
(4) = 𝑥𝑦 + 𝑦
𝑑𝑥

𝑑𝑦
= 𝑦(𝑥 + 1)
𝑑𝑥
1
𝑑𝑦 = (𝑥 + 1)𝑑𝑥
𝑦
1
(𝑥 + 1)𝑑𝑥 − 𝑑𝑦 = 0 (DE is separable)
𝑦

Compiled by: Kenneth D. Dugay, LPT | 15


Polytechnic University of the Philippines
Santa Rosa Campus

The following differential equations are not separable since there are no algebraic manipulations
that can be done to reduce them in the form 𝐴(𝑥)𝑑𝑥 + 𝐵(𝑦)𝑑𝑦 = 0

𝑑𝑦
(1) =𝑥+𝑦
𝑑𝑥

𝑑𝑦 = (𝑥 + 𝑦)𝑑𝑥

𝑑𝑦 = 𝑥𝑑𝑥 + 𝑦𝑑𝑥 (DE is NOT separable)

𝑑𝑦
(2) = 𝑥𝑦 + 1
𝑑𝑥

𝑑𝑦 = (𝑥𝑦 + 1)𝑑𝑥

𝑑𝑦 = 𝑥𝑦𝑑𝑥 + 𝑑𝑥
1 1
𝑑𝑦 = 𝑥𝑑𝑥 + 𝑑𝑥 (DE is NOT separable)
𝑦 𝑦

Example 1. Obtain the general solution for the differential equation:

𝑑𝑦
= 1 + 𝑦2
𝑑𝑥

Separating the variables by making the terms multiplied to dx solely as function of x and terms multiplied
to dy solely functions of y:

𝑑𝑦 𝑑𝑥
[ = 1 + 𝑦2]
𝑑𝑥 1 + 𝑦2

𝑑𝑦
= 𝑑𝑥
1 + 𝑦2
1
Since is a function of y and dx is just multiplied to a constant 1, our DE is separable. We can now
1+𝑦 2
solve for the integral of the whole equation:

𝑑𝑦
∫ = ∫ 𝑑𝑥
1 + 𝑦2

The left-hand side of the equation is an integral leading to inverse trigonometric function:

𝑑𝑢 1 𝑢
∫ 𝑎2+𝑢2 = 𝑎 tan−1 𝑎 + 𝑐
𝑑𝑦
∫ = ∫ 𝑑𝑥
12 + 𝑦2

tan−1 𝑦 = 𝑥 + 𝑐

Compiled by: Kenneth D. Dugay, LPT | 16


Polytechnic University of the Philippines
Santa Rosa Campus

Solving for y by getting the tangent of the both sides of the equaton:

𝑦 = tan(𝑥 + 𝑐)

Example 2. Obtain the general solution for the differential equation:

(1 − 𝑥)𝑦′ = 𝑦 2

Changing the notation of y’

𝑑𝑦
(1 − 𝑥) = 𝑦2
𝑑𝑥

Separating the variables by making the terms multiplied to dx solely as function of x and terms multiplied
to dy solely functions of y:

1
[(1 − 𝑥)𝑑𝑦 = 𝑦 2 𝑑𝑥]
(1−𝑥)𝑦 2

1 1
𝑑𝑦 = 𝑑𝑥
𝑦2 1−𝑥

𝑑𝑥
𝑦 −2 𝑑𝑦 =
1−𝑥

Now that our DE is separable, we can now solve the solution by integrating the whole equation:

𝑑𝑥
∫ = ∫ 𝑦 −2 𝑑𝑦
1−𝑥

Let: 𝑢 =1−𝑥

𝑑𝑢 = −𝑑𝑥 => −𝑑𝑢 = 𝑑𝑥

−𝑑𝑢
∫ = ∫ 𝑦 −2 𝑑𝑦
𝑢

𝑑𝑢
−∫ = ∫ 𝑦 −2 𝑑𝑦
𝑢

Applying integrals leading to ln and power rule of integration:

𝑑𝑢 𝑦 𝑛+1
∫ 𝑢
= ln 𝑢 + 𝑐 ∫ 𝑦 𝑛 𝑑𝑦 = 𝑛+1
+𝑐

𝑦 −1
− ln 𝑢 = +𝑐
−1

Since 𝑢 = 1 − 𝑥
𝑦 −1
− ln|1 − 𝑥| = +𝑐
−1

Compiled by: Kenneth D. Dugay, LPT | 17


Polytechnic University of the Philippines
Santa Rosa Campus

𝑦 ln|1 − 𝑥| = 1 + 𝑐

𝑦 ln|1 − 𝑥| = 𝑐1 (adding 1 and c will still result to a constant, we let that be 𝑐1 )

𝑐1
𝑦=
ln|1 − 𝑥|

Example 3. Obtain the general solution for the differential equation:

sin𝑥 sin𝑦 𝑑𝑥 + cos𝑥 cos𝑦 𝑑𝑦 = 0

Separating the variables:

1
[𝑠𝑖𝑛𝑥 𝑠𝑖𝑛𝑦 𝑑𝑥 + 𝑐𝑜𝑠𝑥 𝑐𝑜𝑠𝑦 𝑑𝑦 = 0]
𝑠𝑖𝑛𝑦 𝑐𝑜𝑠𝑥

𝑠𝑖𝑛𝑥 𝑐𝑜𝑠𝑦
𝑑𝑥 + 𝑑𝑦 = 0
𝑐𝑜𝑠𝑥 𝑠𝑖𝑛𝑦

Solving for the solution by getting their integral:

sinx cos𝑦
∫ 𝑑𝑥 + ∫ 𝑑𝑦 = 0
cosx sin𝑦

Let: 𝑢 = cos𝑥 𝑣 = sin𝑦

𝑑𝑢 = −sin𝑥 𝑑𝑥 => −𝑑𝑢 = sin𝑥 𝑑𝑥 𝑑𝑣 = cos𝑦 𝑑𝑦

−𝑑𝑢 𝑑𝑣
∫ +∫ =0
𝑢 𝑣

𝑑𝑢 𝑑𝑣
−∫ +∫ =0
𝑢 𝑣

ln 𝑣 − ln 𝑢 = 𝑐

By Product Rule of Logarithm:


𝑣
ln 𝑢 = ln 𝑐 (Note that I let the c be ln c since the ln of a constant is still a constant)

Letting the whole equation be the exponent of e:


𝑣
𝑒 ln𝑢 = 𝑒 ln 𝑐
𝑣
=𝑐
𝑢

Bringing back the equation in terms of x and y:

sin𝑦
=𝑐
cos𝑥
sin𝑦 = 𝑐cosx

Compiled by: Kenneth D. Dugay, LPT | 18


Polytechnic University of the Philippines
Santa Rosa Campus

Example 4. Obtain the general solution for the differential equation:

𝑥𝑦 4 𝑑𝑥 + (𝑦 2 + 2)𝑒 −3𝑥 𝑑𝑦 = 0

Separating the variables:

1
[𝑥𝑦 4 𝑑𝑥 + (𝑦 2 + 2)𝑒 −3𝑥 𝑑𝑦 = 0] 𝑦 4𝑒 −3𝑥

𝑥 𝑦2 + 2
−3𝑥
𝑑𝑥 + 𝑑𝑦 = 0
𝑒 𝑦4

𝑥 𝑦2 2
𝑑𝑥 + ( 4 + 4 ) 𝑑𝑦 = 0
1 𝑦 𝑦
𝑒 3𝑥

𝑥𝑒 3𝑥 𝑑𝑥 + (𝑦 −2 + 2𝑦 −4 )𝑑𝑦 = 0

Getting the integral of the equation:

∫ 𝑥𝑒 3𝑥 𝑑𝑥 + ∫(𝑦 −2 + 2𝑦 −4 )𝑑𝑦 = 0

∫ 𝑥𝑒 3𝑥 𝑑𝑥 + ∫ 𝑦 −2 𝑑𝑦 + 2 ∫ 𝑦 −4 𝑑𝑦 = 0

Doing integration by parts: (You may use tabular integration if you are familiar with it.)

1
Let: 𝑢=𝑥 𝑑𝑣 = 𝑒 3𝑥 𝑑𝑥 => ∫ 𝑑𝑣 = ∫ 3 𝑒
3
3𝑥
𝑑𝑥

1
𝑑𝑢 = 𝑑𝑥 𝑣 = 𝑒 3𝑥
3

∫ 𝑢𝑑𝑣 = 𝑢𝑣 − ∫ 𝑣𝑑𝑢

1 1
𝑥 ( 𝑒 3𝑥 ) − ∫ 𝑒 3𝑥 𝑑𝑥
3 3
= − ∫ 𝑦 −2 𝑑𝑦 − 2 ∫ 𝑦 −4 𝑑𝑦
+𝑐 (All terms concerining 𝑑𝑦 was transferred to the right side)

1 3𝑥 1
𝑥𝑒 − ∫ 𝑒 3𝑥 𝑑𝑥 = − ∫ 𝑦 −2 𝑑𝑦 − 2 ∫ 𝑦 −4 𝑑𝑦 + 𝑐
3 3

1 3𝑥 1 1
𝑥𝑒 − ( ) ∫ 3𝑒 3𝑥 𝑑𝑥 = − ∫ 𝑦 −2 𝑑𝑦 − 2 ∫ 𝑦 −4 𝑑𝑦 + 𝑐
3 3 3

1 3𝑥 1 3𝑥
𝑥𝑒 − 𝑒 = − ∫ 𝑦 −2 𝑑𝑦 − 2 ∫ 𝑦 −4 𝑑𝑦 + 𝑐
3 9

Compiled by: Kenneth D. Dugay, LPT | 19


Polytechnic University of the Philippines
Santa Rosa Campus

Using power rule of integration on the right side of the equation:

1 3𝑥 1 3𝑥 𝑦 −1 𝑦 −3
𝑥𝑒 − 𝑒 = − − 2( )+𝑐
3 9 −1 −3

Simplifying further:

1 3𝑥 1 3𝑥 1 2
𝑥𝑒 − 𝑒 = + 3 + 𝑐
3 9 𝑦 3𝑦

9 6
3𝑥𝑒 3𝑥 − 𝑒 3𝑥 = + + 𝑐1
𝑦 𝑦3

9 6
𝑒 3𝑥 (3𝑥 − 1) = + + 𝑐1
𝑦 𝑦3

Example 5. Obtain the general solution for the differential equation:

𝑑𝑦
= 𝑦2 − 4
𝑑𝑥

Separating the variables:

𝑑𝑦 𝑑𝑥
[ = 𝑦 2 − 4] 2
𝑑𝑥 𝑦 −4

𝑑𝑦
= 𝑑𝑥
𝑦2 − 4

𝑑𝑦
= 𝑑𝑥 (1)
(𝑦 + 2)(𝑦 − 2)

Applying Partial Fractions on the left side of the equation:

1 𝐴 𝐵
= + (2)
(𝑦 + 2)(𝑦 − 2) 𝑦 + 2 𝑦 − 2

1 𝐴 𝐵
[ = + ] (𝑦 + 2)(𝑦 − 2)
(𝑦 + 2)(𝑦 − 2) 𝑦 + 2 𝑦 − 2

1 = (𝑦 − 2)𝐴 + (𝑦 + 2)𝐵

Taking the values of A and B by taking the terms starting from the highest degree:
1
𝑦1 : 0=𝐴+𝐵 𝑦0: 1 = −2𝐴 + 2𝐵 𝐵=
4

𝐴 = −𝐵 (3) 1 = −2(−𝐵) + 2𝐵

1 = 2𝐵 + 2𝐵

Compiled by: Kenneth D. Dugay, LPT | 20


Polytechnic University of the Philippines
Santa Rosa Campus

Getting the value of A from (3)

1
𝐴=−
4

Substituting the values of A and B to (2) then applying it to (1):

1 1

( 4 + 4 ) 𝑑𝑦 = 𝑑𝑥
𝑦+2 𝑦−2

Solving for the integral:

1 𝑑𝑦 1 𝑑𝑦
− ∫ + ∫ = ∫ 𝑑𝑥
4 𝑦+2 4 𝑦−2

Let: 𝑢 =𝑦+2 𝑣 =𝑦−2

𝑑𝑢 = 𝑑𝑦 𝑑𝑣 = 𝑑𝑦

1 𝑑𝑢 1 𝑑𝑣
− ∫ + ∫ = ∫ 𝑑𝑥
4 𝑢 4 𝑣

1 1
ln 𝑣 − ln 𝑢 = 𝑥 + 𝑐
4 4

ln 𝑣 − ln 𝑢 = 4𝑥 + 𝑐1

Applying quotient rule of logarithm:

𝑣
ln = 4𝑥 + 𝑐1
𝑢

Making the terms in the whole equations be the exponents of e:


𝑣
𝑒 ln𝑢 = 𝑒 4𝑥+𝑐1

Simplifying further:
𝑣
= 𝑒 4𝑥+𝑐1
𝑢

Bringing back the values of v and u and applying product rule for exponents on the right side:

𝑦−2
= 𝑒 4𝑥 𝑒 𝑐1 (𝑒 raised to a constant is still a constant. We let that be 𝑐2 )
𝑦+2

𝑦−2
= 𝑐2 𝑒 4𝑥
𝑦+2

Compiled by: Kenneth D. Dugay, LPT | 21


Polytechnic University of the Philippines
Santa Rosa Campus

Activities/Assessment
Obtain the general solution for the differential equations:

(1) 𝑦 ′ = 𝑦 2 𝑥 3

(2) (𝑥 − 1)𝑑𝑥 + 𝑥𝑦 2 𝑑𝑥 = 0

(3) 𝑥 2 𝑑𝑥 + 𝑦(𝑥 − 1)𝑑𝑦 = 0

(4) tan 2 𝑦 𝑑𝑥 = sec 2 𝑥 𝑑𝑦

(5) 𝑥𝑦 3 𝑑𝑥 + 𝑒 −𝑥 𝑑𝑦 = 0

Compiled by: Kenneth D. Dugay, LPT | 22


Polytechnic University of the Philippines
Santa Rosa Campus

Lesson 2
Homogeneous Equations

Module Objective
After successful completion of this module, you should be able to:

▪ define and identify homogeneous functions;


▪ test if a differential equation is homogeneous; and
▪ solve for the solution of a homogeneous differential equation.

Course Materials
Lecture Video

(Click the picture for the link or type in: https://bit.ly/Homogeneous)

Click: Solved Problem #1 for Homogenous Differential Equations: https://bit.ly/Homogeneous1

Solved Problem #2 for Homogenous Differential Equations: https://bit.ly/Homogeneous2

Solved Problem #3 for Homogenous Differential Equations: https://bit.ly/Homogeneous3

Solved Problem #4 for Homogenous Differential Equations: https://bit.ly/Homogeneous4

Discussions
Homogeneous Polynomials

They are polynomials all whose terms are of the same degree. Examples are as follows:

\ 3
(1) 𝑓(𝑥, 𝑦) = 𝑥 + 𝑥 𝑦 + 𝑦
2 3
(Homogeneous degree 3)

(2) 𝑓(𝑥, 𝑦) = 𝑥 2 + 𝑥𝑦 + 𝑦 2 (Homogeneous degree 2)

(3) 𝑓(𝑥, 𝑦) = 𝑥 4 + 𝑥 3 𝑦 + 𝑥𝑦 3 + 𝑦 4 (Homogeneous degree 4)

Compiled by: Kenneth D. Dugay, LPT | 23


Polytechnic University of the Philippines
Santa Rosa Campus

Degrees of the terms of a polynomial function can be easily seen by observation. We only have to
look at the exponents of the terms. If a term contains two or more variables, we can get the exponents of
each variable then add them to get the degree.

Looking at the examples above, each polynomial is homogenous since all of the terms share the
same degree. But not all the equations in this subject are polynomials so we have to define what are
homogeneous functions.

Homogeneous Functions

A function 𝑓(𝑥, 𝑦) is said to be homogenous of degree 𝑘 in 𝑥 and 𝑦 if it can be written in the form:

𝑓(𝜆𝑥, 𝜆𝑦) = 𝜆𝑘 𝑓(𝑥, 𝑦)

To test the homogeneity of a function, we change each x in the function as 𝜆𝑥 and each y as
𝜆𝑦. It is homogeneous if we can reduce it in the form above where 𝜆 is raised to a certain exponent k
(which is the degree of the homogeneous equation) then multiplied to the original function 𝑓(𝑥, 𝑦)

Example 1. Test if the following functions are homogeneous:

𝑓(𝑥, 𝑦) = 𝑥 3 + 𝑥 2 𝑦 + 𝑦 3

First, let us transform the function to 𝑓(𝜆𝑥, 𝜆𝑦). We change all the values of x to 𝜆𝑥 and y to 𝜆𝑦

𝑓(𝜆𝑥, 𝜆𝑦) = (𝜆𝑥)3 + (𝜆𝑥)2 (𝜆𝑦) + (𝜆𝑦)3

Solving algebraically

𝑓(𝜆𝑥, 𝜆𝑦) = 𝜆3 𝑥 3 + 𝜆2 𝑥 2 𝜆𝑦 + 𝜆3 𝑦 3

= 𝜆3 𝑥 3 + 𝜆3 𝑥 2 𝑦 + 𝜆3 𝑦 3

= 𝜆3 (𝑥 3 + 𝑥 2 𝑦 + 𝑦 3 )

We arrived at our given equation 𝑓(𝑥, 𝑦) multiplied to 𝜆3 , then:

𝑓(𝜆𝑥, 𝜆𝑦) = 𝜆3 𝑓(𝑥, 𝑦) (Homogeneous degree 3)

Example 2. Test if the following functions are homogeneous:

𝑥 2 + 3𝑥𝑦
𝑓(𝑥, 𝑦) =
𝑥 − 2𝑦

Following the same procedure as example 1:

(𝜆𝑥)2 + 3(𝜆𝑥)(𝜆𝑦)
𝑓(𝜆𝑥, 𝜆𝑦) =
𝜆𝑥 − 2(𝜆𝑦)

Compiled by: Kenneth D. Dugay, LPT | 24


Polytechnic University of the Philippines
Santa Rosa Campus

𝜆2 𝑥 2 + 3𝜆2 𝑥𝑦
=
𝜆𝑥 − 2𝜆𝑦

𝜆2 (𝑥 2 + 3𝑥𝑦)
=
𝜆(𝑥 − 2𝑦)

(𝑥 2 + 3𝑥𝑦)
= 𝜆1
(𝑥 − 2𝑦)

𝑓(𝜆𝑥, 𝜆𝑦) = 𝜆1 𝑓(𝑥, 𝑦) (Homogeneous degree 1)

Example 3. Test if the following functions are homogeneous:

𝑓(𝑥, 𝑦) = 𝑒 𝑥 𝑦 + 𝑦 2

Still letting our given function be 𝑓(𝜆𝑥, 𝜆𝑦):

𝑓(𝜆𝑥, 𝜆𝑦) = 𝑒 𝜆𝑥 (𝜆𝑦) + (𝜆𝑦)2

𝑓(𝜆𝑥, 𝜆𝑦) = 𝜆𝑒 𝜆𝑥 𝑦 + 𝜆2 𝑦 2

𝑓(𝜆𝑥, 𝜆𝑦) = 𝜆(𝑒 𝜆𝑥 𝑦 + 𝜆𝑦 2 ) (Not homogeneous)

Since there is no algebraic manipulation that can be done to reduce our equation to the form 𝜆𝑘 𝑓(𝑥, 𝑦),
then we can say that it is not homogeneous. Other methods for solving order one DE can be used to
solve this.

Example 4. Test if the following functions are homogeneous:

𝑥 𝑥
𝑓(𝑥, 𝑦) = ln − 𝑒 𝑦
𝑦

𝜆𝑥 𝜆𝑥
𝑓(𝜆𝑥, 𝜆𝑦) = ln − 𝑒 𝜆𝑦
𝜆𝑦
𝑥
𝑥
= ln − 𝑒 𝑦
𝑦

𝑥
𝑥
= 𝜆0 (ln − 𝑒 𝑦 )
𝑦

𝑓(𝜆𝑥, 𝜆𝑦) = 𝜆0 𝑓(𝑥, 𝑦) (Homogeneous degree 0)

Example 5. Test if the following functions are homogeneous:

𝑓(𝑥, 𝑦) = √𝑥 − 𝑦

𝑓(𝜆𝑥, 𝜆𝑦) = √𝜆𝑥 − 𝜆𝑦

Compiled by: Kenneth D. Dugay, LPT | 25


Polytechnic University of the Philippines
Santa Rosa Campus

= √𝜆(𝑥 − 𝑦)

= √𝜆 ∙ √𝑥 − 𝑦

1
= 𝜆 2 √𝑥 − 𝑦

1 1
𝑓(𝜆𝑥, 𝜆𝑦) = 𝜆2 𝑓(𝑥, 𝑦) (Homogeneous degree )
2

As we observe in the examples 4 and 5, the degree of a homogeneous equation need not to be a
counting number. This is one of the flexibilities of homogeneous equations as we solve for their solution.

Homogeneous Equations

Now that we already know how to test if an equation is indeed homogeneous, let us determine
the steps in solving homogeneous DE.

Steps in Solving Homogeneous Differential Equations:

(1) Rewrite the DE in the form:

𝑀(𝑥, 𝑦)𝑑𝑥 + 𝑁(𝑥, 𝑦)𝑑𝑦 = 0

(2) Test if the DE is homogeneous.

(3) If homogeneous, let:

𝑥 = 𝑣𝑦 if 𝑀(𝑥, 𝑦) is simpler and 𝑑𝑥 = 𝑣𝑑𝑦 + 𝑦𝑑𝑣

𝑦 = 𝑣𝑥 if 𝑁(𝑥, 𝑦) is simpler and 𝑑𝑦 = 𝑣𝑑𝑥 + 𝑥𝑑𝑣

(4) Simplify the equation so that it will lead to separable DE.

(5) Solve for the solution by integrating both sides of the equation.

Example 1. Obtain the general solution for the differential equation:

(𝑥 2 + 𝑦 2 )𝑑𝑥 + 3𝑥𝑦𝑑𝑦 = 0

Since this is a polynomial equation, its degree of 2 for all terms can be seen easily. Then we can say that
this DE is indeed homogeneous

𝑥 2 𝑑𝑥 + 𝑦 2 𝑑𝑥 + 3𝑥𝑦𝑑𝑦 = 0

Since N(x,y) is simpler compared to M(x,y), we let:

𝑦 = 𝑣𝑥

𝑑𝑦 = 𝑣𝑑𝑥 + 𝑥𝑑𝑣

Compiled by: Kenneth D. Dugay, LPT | 26


Polytechnic University of the Philippines
Santa Rosa Campus

Substituting it to all the values of y and dy, then simplifying algebraically, it will lead to a separable DE:

𝑥 2 𝑑𝑥 + (𝑣𝑥)2 𝑑𝑥 + 3𝑥(𝑣𝑥)(𝑣𝑑𝑥 + 𝑥𝑑𝑣) = 0

𝑥 2 𝑑𝑥 + 𝑣 2 𝑥 2 𝑑𝑥 + 3𝑣𝑥 2 (𝑣𝑑𝑥 + 𝑥𝑑𝑣) = 0

𝑥 2 𝑑𝑥 + 𝑣 2 𝑥 2 𝑑𝑥 + 3𝑣 2 𝑥 2 𝑑𝑥 + 3𝑣𝑥 3 𝑑𝑣 = 0

𝑥 2 𝑑𝑥 + 4𝑣 2 𝑥 2 𝑑𝑥 + 3𝑣𝑥 3 𝑑𝑣 = 0

1
[𝑥 2 (1 + 4𝑣 2 )𝑑𝑥 + 3𝑣𝑥 3 𝑑𝑣 = 0]
(1 + 4𝑣 2 )(𝑥 3 )

1 3𝑣
𝑑𝑥 + 𝑑𝑣 = 0
𝑥 (1 + 4𝑣 2 )

Now that we have a separable DE, we can now proceed with solving its solution by taking the integral of
the whole equation:

𝑑𝑥 𝑣
∫ + 3∫ 𝑑𝑣 = 0
𝑥 1 + 4𝑣 2

Let: 𝑢 = 1 + 4𝑣 2
1
𝑑𝑢 = 8𝑣𝑑𝑣 => 𝑑𝑢 = 𝑣𝑑𝑣
8

1
𝑑𝑥 𝑑𝑢
∫ + 3∫8 =0
𝑥 𝑢

𝑑𝑥 3 𝑑𝑢
∫ + ∫ =0
𝑥 8 𝑢

3
ln 𝑥 + ln 𝑢 = 𝑐
8

8ln 𝑥 + 3 ln 𝑢 = 𝑐

Using Rules of Logarithm to simplify the equation, then making them exponents of e:

ln(𝑥 8 𝑢3 ) = ln 𝑐

8 𝑢3 )
𝑒 ln(𝑥 = 𝑒 ln 𝑐

𝑥 8 (1 + 4𝑣 2 )3 = 𝑐

Since
𝑦
𝑦 = 𝑣𝑥 then 𝑣 = 𝑥

Compiled by: Kenneth D. Dugay, LPT | 27


Polytechnic University of the Philippines
Santa Rosa Campus

Bringing back our solution in terms of x and y:

𝑦 2 3
𝑥 8 [1 + 4 ( ) ] = 𝑐
𝑥
3
𝑦2
𝑥 8 [1 + 4 ] =𝑐
𝑥2
3
𝑥 2 + 4𝑦 2
𝑥8 [ ] =𝑐
𝑥2

(𝑥 2 + 4𝑦 2 )3
𝑥8 [ ]=𝑐
𝑥6

𝑥 2 (𝑥 2 + 4𝑦 2 )3 = 𝑐

Example 2. Obtain the general solution for the differential equation:

(𝑥 − 2𝑦)𝑑𝑥 + (2𝑥 + 𝑦)𝑑𝑦 = 0

𝑥𝑑𝑥 − 2𝑦𝑑𝑥 + 2𝑥𝑑𝑦 + 𝑦𝑑𝑦 = 0

For this example, both M(x, y) and N(x, y) are simple enough. You may select any of the two but I decided
to pick N(x, y) to avoid the negative sign in M(x, y). With this, we let:

𝑦 = 𝑣𝑥 𝑑𝑦 = 𝑣𝑑𝑥 + 𝑥𝑑𝑣

Substituting and simplifying it will lead to a separable DE:

𝑥𝑑𝑥 − 2(𝑣𝑥)𝑑𝑥 + 2𝑥(𝑣𝑑𝑥 + 𝑥𝑑𝑣) + 𝑣𝑥(𝑣𝑑𝑥 + 𝑥𝑑𝑣) = 0

𝑥𝑑𝑥 − 2𝑣𝑥𝑑𝑥 + 2𝑣𝑥𝑑𝑥 + 2𝑥 2 𝑑𝑣 + 𝑣 2 𝑥𝑑𝑥 + 𝑣𝑥 2 𝑑𝑣 = 0

𝑥𝑑𝑥 + 𝑣 2 𝑥𝑑𝑥 + 2𝑥 2 𝑑𝑣 + 𝑣𝑥 2 𝑑𝑣 = 0

1
[𝑥(1 + 𝑣 2 )𝑑𝑥 + 𝑥 2 (2 + 𝑣)𝑑𝑣 = 0]
(𝑥 2 )(1 + 𝑣2)

𝑑𝑥 2 + 𝑣
+ 𝑑𝑣 = 0
𝑥 1 + 𝑣2

Solving for its solution by getting the integral of the equation:

𝑑𝑥 2+𝑣
∫ +∫ 𝑑𝑣 = 0
𝑥 1 + 𝑣2

𝑑𝑥 2+𝑣
∫ +∫ 𝑑𝑣 = 0
𝑥 1 + 𝑣2

Compiled by: Kenneth D. Dugay, LPT | 28


Polytechnic University of the Philippines
Santa Rosa Campus

𝑑𝑥 𝑑𝑣 𝑣𝑑𝑣
∫ + 2∫ +∫ =0
𝑥 1 + 𝑣2 1 + 𝑣2

The second term of the equation can be solved by using the integration leading to inverse trigonometric
functions:

𝑑𝑢 1 𝑢
∫ 𝑎2 +𝑢2 = 𝑎 tan−1 𝑎 + 𝑐

𝑣𝑑𝑣
ln 𝑥 + 2tan−1 𝑣 + ∫ =𝑐
1 + 𝑣2

For our third term, we let:

𝑢 = 1 + 𝑣2
1
𝑑𝑢 = 2𝑣𝑑𝑣 => 𝑑𝑢 = 𝑣𝑑𝑣
2

1
𝑑𝑢
ln 𝑥 + 2tan 𝑣 + ∫ 2
−1
=𝑐
𝑢

1
ln 𝑥 + 2tan−1 𝑣 + ln 𝑢 = 𝑐
2

From this point onward, we are just simplifying (rules of logarithm were applied) our answer and bringing
it back in terms of x and y:

2ln 𝑥 + 4tan−1 𝑣 + ln 𝑢 = 𝑐

ln 𝑥 2 + ln 𝑢 + 4tan−1 𝑣 = 𝑐

ln|𝑥 2 𝑢| + 4tan−1 𝑣 = 𝑐

Since,

𝑢 = 1 + 𝑣2

ln|𝑥 2 (1 + 𝑣 2 )| + 4tan−1 𝑣 = 𝑐

Also earlier on, we let:


𝑦
𝑦 = 𝑣𝑥 so 𝑣 =
𝑥

𝑦2 𝑦
ln |𝑥 2 (1 + )| + 4tan−1 = 𝑐
𝑥2 𝑥

𝑥 2 + 𝑦2 𝑦
ln |𝑥 2 ( 2
)| + 4tan−1 = 𝑐
𝑥 𝑥

𝑦
ln|𝑥 2 + 𝑦 2 | + 4tan−1 =𝑐
𝑥

Compiled by: Kenneth D. Dugay, LPT | 29


Polytechnic University of the Philippines
Santa Rosa Campus

Example 3. Obtain the general solution for the differential equation:

𝑦𝑑𝑥 = (𝑥 + √𝑦 2 − 𝑥 2 ) 𝑑𝑦

𝑦𝑑𝑥 = 𝑥𝑑𝑦 + √𝑦 2 − 𝑥 2 𝑑𝑦

Since M(x, y) is simpler, we let:

𝑥 = 𝑣𝑦 𝑑𝑥 = 𝑣𝑑𝑦 + 𝑦𝑑𝑣

Simplifying our equation,

𝑦(𝑣𝑑𝑦 + 𝑦𝑑𝑣) = (𝑣𝑦)𝑑𝑦 + √𝑦 2 − (𝑣𝑦)2 𝑑𝑦

𝑣𝑦𝑑𝑦 + 𝑦 2 𝑑𝑣 = 𝑣𝑦𝑑𝑦 + √𝑦 2 − 𝑣 2 𝑦 2 𝑑𝑦

𝑦 2 𝑑𝑣 = √𝑦 2 (1 − 𝑣 2 ) 𝑑𝑦

𝑦 2 𝑑𝑣 = √𝑦 2 √ (1 − 𝑣 2 ) 𝑑𝑦

1
[𝑦 2 𝑑𝑣 = 𝑦√ (1 − 𝑣 2 ) 𝑑𝑦]
(𝑦 2 )(√1 − 𝑣 2 )

𝑑𝑣 𝑑𝑦
=
√1 − 𝑣 2 𝑦

Solving for the solution,

𝑑𝑣 𝑑𝑦
∫ =∫
√1 − 𝑣 2 𝑦

Looking at our integration in the left side, it will lead to inverse trigonometric function:

𝑑𝑢 𝑢
∫ √𝑎2−𝑢2 = sin−1 𝑎 + 𝑐

Doing the integration:

sin−1 𝑣 = ln 𝑦 + 𝑐

Since,
𝑥
𝑥 = 𝑣𝑦 𝑣=
𝑦

Then:

𝑥
sin−1 = ln 𝑦 + 𝑐
𝑦

Compiled by: Kenneth D. Dugay, LPT | 30


Polytechnic University of the Philippines
Santa Rosa Campus

Example 4. Obtain the general solution for the differential equation:

(𝑥 − 𝑦)(4𝑥 + 𝑦)𝑑𝑥 + 𝑥(5𝑥 − 𝑦)𝑑𝑦 = 0

(4𝑥 2 + 𝑥𝑦 − 4𝑥𝑦 − 𝑦 2 )𝑑𝑥 + (5𝑥 2 − 𝑥𝑦)𝑑𝑦 = 0

(4𝑥 2 − 3𝑥𝑦 − 𝑦 2 )𝑑𝑥 + (5𝑥 2 − 𝑥𝑦)𝑑𝑦 = 0

Since N(x, y) is simpler for it has less terms, we let:

𝑦 = 𝑣𝑥 𝑑𝑦 = 𝑣𝑑𝑥 + 𝑥𝑑𝑣

Substituting the values:


(4𝑥 2 − 3𝑣𝑥 2 − 𝑣 2 𝑥 2 )𝑑𝑥 + (5𝑥 2 − 𝑣𝑥 2 )(𝑣𝑑𝑥 + 𝑥𝑑𝑣) = 0

4𝑥 2 𝑑𝑥 − 3𝑣𝑥 2 𝑑𝑥 − 𝑣 2 𝑥 2 𝑑𝑥 + 5𝑣𝑥 2 𝑑𝑥 − 𝑣 2 𝑥 2 𝑑𝑥 + 5𝑥 3 𝑑𝑣 − 𝑣𝑥 3 𝑑𝑣 = 0

4𝑥 2 𝑑𝑥 + 2𝑣𝑥 2 𝑑𝑥 − 2𝑣 2 𝑥 2 𝑑𝑥 + 5𝑥 3 𝑑𝑣 − 𝑣𝑥 3 𝑑𝑣 = 0

1
[2𝑥 2 (2 + 𝑣 − 𝑣 2 )𝑑𝑥 + 𝑥 3 (5 − 𝑣)𝑑𝑣 = 0]
(2 + 𝑣 − 𝑣 2 )(𝑥 3 )

2𝑑𝑥 5−𝑣
+ 𝑑𝑣 = 0
𝑥 2 + 𝑣 − 𝑣2

Getting its integral:

𝑑𝑥 5−𝑣
2∫ +∫ 𝑑𝑣 = 0
𝑥 2 + 𝑣 − 𝑣2

Applying Partial Fractions:

5−𝑣 𝐴 𝐵
= +
(2 − 𝑣)(1 + 𝑣) 2 − 𝑣 1 + 𝑣

5−𝑣 𝐴 𝐵
[(2−𝑣)(1+𝑣) = + ] (2 − 𝑣)(1 + 𝑣)
2−𝑣 1+𝑣

5 − 𝑣 = (1 + 𝑣)𝐴 + (2 − 𝑣)𝐵

Working from the highest degree to v to its lowest degree:

𝑣1 : −1=𝐴 −𝐵

𝑣0: 5 = 𝐴 + 2𝐵 (Eliminating A by doing 𝑣 1 − 𝑣 0 )

−6 = − 3𝐵

𝐵=2

Solving for A using 𝑣1 :

−1 = 𝐴 − 2 => 𝐴=1

Compiled by: Kenneth D. Dugay, LPT | 31


Polytechnic University of the Philippines
Santa Rosa Campus

Then using the values of A and B and substituting it to the equation where we are doing our integration:

𝑑𝑥 1 2
2∫ + ∫( + ) 𝑑𝑣 = 0
𝑥 2−𝑣 1+𝑣

𝑑𝑥 𝑑𝑣 𝑑𝑣
2∫ +∫ + 2∫ =0
𝑥 2−𝑣 1+𝑣

𝑑𝑥 −𝑑𝑣 𝑑𝑣
2∫ + (−1) ∫ + 2∫ =0
𝑥 2−𝑣 1+𝑣

2 ln 𝑥 − ln|2 − 𝑣| + 2 ln|1 + 𝑣| = ln 𝑐

Now that we are done with the integration process, we only have to simplify our equation. Applying Power
Rule of Logarithm:

ln 𝑥 2 − ln|2 − 𝑣| + ln|(1 + 𝑣)2 | = ln 𝑐

Then simultaneously using Product and Quotient Riles for Logarithm:

𝑥 2 (1 + 𝑣)2
ln | | = ln 𝑐
2−𝑣

Then making both sides of the equations the exponents of base e will lead to this:

𝑥 2 (1 + 𝑣)2
=𝑐
2−𝑣

Since,
𝑦
𝑦 = 𝑣𝑥 𝑣=
𝑥

𝑦 2
𝑥 2 (1 + )
𝑥 =𝑐
𝑦
2−
𝑥

𝑥+𝑦 2
𝑥2 ( )
𝑥 =𝑐
2𝑥 − 𝑦
𝑥

(𝑥 + 𝑦)2 𝑥
𝑥2 [ ][ ]=𝑐
𝑥2 2𝑥 − 𝑦

𝑥(𝑥 + 𝑦)2 = 𝑐(2𝑥 − 𝑦)

Compiled by: Kenneth D. Dugay, LPT | 32


Polytechnic University of the Philippines
Santa Rosa Campus

Activities/Assessment
Obtain the general solution for the differential equations:

′ 𝑥−𝑦
(1) 𝑦 =
𝑦

(2) (𝑦 2 − 𝑥 2 )𝑑𝑦 = 2𝑥𝑦𝑑𝑥

(3) 𝑥𝑦𝑑𝑥 − (𝑥 + 2𝑦)2 𝑑𝑦 = 0

(4) 𝑥 2 𝑦 ′ = 4𝑥 2 + 7𝑥𝑦 + 2𝑦 2

(5) 𝑥(𝑥 2 + 𝑦 2 )2 (𝑦𝑑𝑥 − 𝑥𝑑𝑦) + 𝑦 6 𝑑𝑦 = 0

Compiled by: Kenneth D. Dugay, LPT | 33


Polytechnic University of the Philippines
Santa Rosa Campus

Lesson 3
Linear Coefficients in Two Variables

Learning Objective
After successful completion of this module, you should be able to:

▪ identify differential equations with linear coefficients in two linear variables by observing its form;
▪ reduce the differential equation with linear coefficients in two linear variables into a homogenous
equation ; and
▪ solve for the solution of the DE using methods of solving homogeneous equations.

Course Materials
Lecture Video

(Click the picture for the link or type in: http://bit.ly/LinearInTwo)

Click: Solved Problem #1 for Linear Coefficients in Two Linear Variables: http://bit.ly/LinearInTwo1

Solved Problem #2 for Linear Coefficients in Two Linear Variables: http://bit.ly/LinearInTwo2

Solved Problem #3 for Linear Coefficients in Two Linear Variables: http://bit.ly/LinearInTwo3

Discussions
As we recall from our Algebra class, two linear equations are in the form:

𝑎1 𝑥 + 𝑏1 𝑦 + 𝑐1 = 0 (1)

𝑎2 𝑥 + 𝑏2 𝑦 + 𝑐2 = 0 (2)

Then, if we let the left side of (1) be M(x, y) and (2) be N(x, y) of an order one DE, we will be
arriving at the form of the differential equation with linear coefficients in two variables:

(𝑎1 𝑥 + 𝑏1 𝑦 + 𝑐1 )𝑑𝑥 + (𝑎2 𝑥 + 𝑏2 𝑦 + 𝑐2 )𝑑𝑦 = 0

Compiled by: Kenneth D. Dugay, LPT | 34


Polytechnic University of the Philippines
Santa Rosa Campus

Examples of DE with Linear Coefficients in Two Variables:

(1) (𝑥 + 2𝑦 − 1)𝑑𝑥 + (𝑥 + 𝑦 − 3)𝑑𝑦 = 0

(2) (6𝑥 − 3𝑦 + 2)𝑑𝑥 + (2𝑥 − 𝑦 − 1)𝑑𝑦 = 0

(3) (𝑥 − 2)𝑑𝑥 + (𝑥 + 𝑦 − 1)𝑑𝑦 = 0

Notice that in (3), M(x, y) only have two terms. However, we can still consider it as linear
coefficients in two variables with the value of 𝑏1 being 0.

Steps in Solving DE with Linear Coefficients in Two Variables

(1) Transform the given DE in the form:

(𝑎1 𝑥 + 𝑏1 𝑦 + 𝑐1 )𝑑𝑥 + (𝑎2 𝑥 + 𝑏2 𝑦 + 𝑐2 )𝑑𝑦 = 0

(2) Determine for the intersection (h, k) of the two linear equations by using any methods of solving
systems of linear equations in two variables.

(3) Let the values of x and y be:

𝑥 =𝑢+ℎ 𝑑𝑥 = 𝑑𝑢

𝑦=𝑣+𝑘 𝑑𝑦 = 𝑑𝑣

where (h, k) is the intersection of the two linear equations.

(4) Simplify the equation to arrive at a homogeneous differential equation.

(5) Apply the methods to solve homogeneous equations to solve for the solution.

Example 1. Obtain the general solution for the differential equation:

(𝑥 + 𝑦 − 4)𝑑𝑥 − (𝑥 − 𝑦 + 2)𝑑𝑦 = 0

First, let us change it to the form (𝑎1 𝑥 + 𝑏1 𝑦 + 𝑐1 )𝑑𝑥 + (𝑎2 𝑥 + 𝑏2 𝑦 + 𝑐2 )𝑑𝑦 = 0,

(𝑥 + 𝑦 − 4)𝑑𝑥 + (−𝑥 + 𝑦 − 2)𝑑𝑦 = 0

Now we have our linear equations:

𝑥 + 𝑦 − 4 = 0 (1) and −𝑥 + 𝑦 − 2 = 0 (2)

Using (1):

𝑥 =4−𝑦

Substituting it to (2):

−(4 − 𝑦) + 𝑦 − 2 = 0

Compiled by: Kenneth D. Dugay, LPT | 35


Polytechnic University of the Philippines
Santa Rosa Campus

𝑦=3

Solving for x using (1) and y = 3,

𝑥=1

Then, the value of (h, k) = (1, 3)

Next, we let:

𝑥 =𝑢+ℎ => 𝑥 =𝑢+1 𝑑𝑥 = 𝑑𝑢

𝑦=𝑣+𝑘 => 𝑦=𝑣+3 𝑑𝑦 = 𝑑𝑣

Substituting these values to our given DE:

(𝑥 + 𝑦 − 4)𝑑𝑥 + (−𝑥 + 𝑦 − 2)𝑑𝑦 = 0

[(𝑢 + 1) + (𝑣 + 3) − 4]𝑑𝑢 + [−(𝑢 + 1) + (𝑣 + 3) − 2]𝑑𝑣 = 0

(𝑢 + 𝑣)𝑑𝑢 + (−𝑢 + 𝑣)𝑑𝑣 = 0

Now, our equation is homogeneous. Let us pick M(u, v) and introduce a new variable m. Let:

𝑢 = 𝑚𝑣 𝑑𝑢 = 𝑚𝑑𝑣 + 𝑣𝑑𝑚

(𝑚𝑣 + 𝑣)(𝑚𝑑𝑣 + 𝑣𝑑𝑚) + (−𝑚𝑣 + 𝑣)𝑑𝑣 = 0

𝑚2 𝑣𝑑𝑣 + 𝑚𝑣 2 𝑑𝑚 + 𝑚𝑣𝑑𝑣 + 𝑣 2 𝑑𝑚 − 𝑚𝑣𝑑𝑣 + 𝑣𝑑𝑣 = 0

𝑚2 𝑣𝑑𝑣 + 𝑣𝑑𝑣 + 𝑚𝑣 2 𝑑𝑚 + 𝑣 2 𝑑𝑚 = 0

1
[𝑣(𝑚2 + 1)𝑑𝑣 + 𝑣 2 (𝑚 + 1)𝑑𝑚 = 0]
(𝑚2 + 1)(𝑣 2 )

𝑑𝑣 𝑚 + 1
+ 2 𝑑𝑚 = 0
𝑣 𝑚 +1

Our DE is now separable. Let us take the integral of our whole equation:

𝑑𝑣 𝑚+1
∫ +∫ 2 𝑑𝑚 = 0
𝑣 𝑚 +1

𝑑𝑣 𝑚𝑑𝑚 𝑑𝑚
∫ +∫ 2 +∫ 2 =0
𝑣 𝑚 +1 𝑚 +1

𝑑𝑣 1 2𝑚𝑑𝑚 𝑑𝑚
∫ + ∫ 2 +∫ 2 =0
𝑣 2 𝑚 +1 𝑚 +1

Our third term can be solved by integration leading to inverse trigonometric functions. Then, we will arrive
at:
1
ln 𝑣 + ln|𝑚2 + 1| + tan −1 𝑚 = ln 𝑐 (I made the constant c as ln c in preparation for simplification)
2

Compiled by: Kenneth D. Dugay, LPT | 36


Polytechnic University of the Philippines
Santa Rosa Campus

1
ln 𝑣 + ln|𝑚2 + 1| + ln 𝑐 = −tan −1 𝑚 (ln c is still positive on the left side because the negative of a
2
constant is still a constant)

2 ln 𝑣 + ln|𝑚2 + 1| + ln 𝑐 = −2tan −1 𝑚

Applying Rules of Logarithm:

ln 𝑣 2 + ln|𝑚2 + 1| + ln 𝑐 = −2tan −1 𝑚

ln|(𝑣 2 )(𝑚2 + 1)𝑐| = −2tan −1 𝑚

Since,
𝑢
𝑢 = 𝑚𝑣 𝑚=
𝑣

𝑢2 𝑢
ln |(𝑣 2 )( 2
+ 1)𝑐| = −2tan −1
𝑣 𝑣

𝑢2 + 𝑣 2 𝑢
ln |(𝑣 2 )( 2
)𝑐| = −2tan −1
𝑣 𝑣

𝑢
ln|(𝑢2 + 𝑣 2 )𝑐| = −2tan −1
𝑣

But:

𝑥 =𝑢+1 => 𝑢 =𝑥−1

𝑦=𝑣+3 => 𝑣 =𝑦−3

𝑥−1
ln|[(𝑥 − 1)2 + (𝑦 − 3)2 ]𝑐| = −2tan −1
𝑦−3

Example 2. Obtain the general solution for the differential equation:

(𝑥 − 2𝑦 + 3)𝑑𝑥 + (4𝑥 + 𝑦 + 3)𝑑𝑦 = 0

Solving for the intersection of the two linear equations (by elimination or substitution), it will lead to:

(h, k) = (-1, 1)

Then we let:

𝑥 =𝑢−1 𝑑𝑥 = 𝑑𝑢

𝑦=𝑣+1 𝑑𝑦 = 𝑑𝑣

Substituting it to our given equation:

[(𝑢 − 1) − 2(𝑣 + 1) + 3]𝑑𝑢 + [4(𝑢 − 1) + (𝑣 + 1) + 3]𝑑𝑣 = 0

Compiled by: Kenneth D. Dugay, LPT | 37


Polytechnic University of the Philippines
Santa Rosa Campus

[𝑢 − 2𝑣]𝑑𝑢 + [4𝑢 + 𝑣]𝑑𝑣 = 0

Let us pick our M(u, v) and let:

𝑢 = 𝑚𝑣 𝑑𝑢 = 𝑚𝑑𝑣 + 𝑣𝑑𝑚

(𝑚𝑣 − 2𝑣)(𝑚𝑑𝑣 + 𝑣𝑑𝑚) + (4𝑚𝑣 + 𝑣)𝑑𝑣 = 0

𝑚2 𝑣𝑑𝑣 + 𝑚𝑣 2 𝑑𝑚 − 2𝑚𝑣𝑑𝑣 − 2𝑣 2 𝑑𝑚 + 4𝑚𝑣𝑑𝑣 + 𝑣𝑑𝑣 = 0

𝑚2 𝑣𝑑𝑣 + 2𝑚𝑣𝑑𝑣 + 𝑣𝑑𝑣 + 𝑚𝑣 2 𝑑𝑚 − 2𝑣 2 𝑑𝑚 = 0

1
[𝑣(𝑚2 + 2𝑚 + 1)𝑑𝑣 + 𝑣 2 (𝑚 − 2)𝑑𝑚 = 0]
(𝑚2 + 2𝑚 + 1)(𝑣 2 )

𝑑𝑣 𝑚−2
+ 2 𝑑𝑚 = 0
𝑣 𝑚 + 2𝑚 + 1

𝑑𝑣 𝑚−2
+ 𝑑𝑚 = 0
𝑣 (𝑚 + 1)2

Now that our equation is separable, let us get its integral:

𝑑𝑣 𝑚−2
∫ +∫ 𝑑𝑚 = 0
𝑣 (𝑚 + 1)2

Applying partial fractions in our second term:

𝑚−2 𝐴 𝐵
= +
(𝑚 + 1)2 𝑚 + 1 (𝑚 + 1)2

𝑚 − 2 = (𝑚 + 1)𝐴 + 𝐵

Solving the partial fraction will yield to:

𝑚1 : 𝐴 = 1

𝑚0 : − 2 = 𝐴 + 𝐵 but 𝐴 = 1 so 𝐵 = −3

Substituting the values of A and B to our integration:

𝑑𝑣 1 −3
∫ +∫[ + ] 𝑑𝑚 = 0
𝑣 𝑚 + 1 (𝑚 + 1)2

𝑑𝑣 𝑑𝑚 𝑑𝑚
∫ +∫ − 3∫ =0
𝑣 𝑚+1 (𝑚 + 1)2

Let:

𝑛 = 𝑚+1 𝑑𝑛 = 𝑑𝑚

Compiled by: Kenneth D. Dugay, LPT | 38


Polytechnic University of the Philippines
Santa Rosa Campus

𝑑𝑣 𝑑𝑛
∫ +∫ − 3 ∫ 𝑛−2 𝑑𝑛 = 0
𝑣 𝑛

ln 𝑣 + ln 𝑛 + 3𝑛−1 = ln 𝑐

ln 𝑣 + ln|𝑚 + 1| + ln 𝑐 = −3(𝑚 + 1)−1

−3
ln|𝑣(𝑚 + 1)𝑐| =
𝑚+1

Since,
𝑢
𝑢 = 𝑚𝑣 𝑚=
𝑣

𝑢 −3
ln |𝑣( + 1)𝑐| = 𝑢
𝑣 +1
𝑣

𝑢+𝑣 −3
ln |𝑣 ( ) 𝑐| = 𝑢 + 𝑣
𝑣
𝑣

−3𝑣
ln|(𝑢 + 𝑣)𝑐| =
𝑢+𝑣

But,

𝑥 =𝑢−1 𝑢 =𝑥+1

𝑦=𝑣+1 𝑣 =𝑦−1

−3(𝑦 − 1)
ln|(𝑥 + 1 + 𝑦 − 1)𝑐| =
𝑥+1+𝑦−1

(x + y) ln|(𝑥 + 𝑦)𝑐| = −3(𝑦 − 1)

Example 3. Obtain the general solution for the differential equation:

(2𝑥 − 5𝑦 + 3)𝑑𝑥 − (2𝑥 + 4𝑦 − 6)𝑑𝑦 = 0

(2𝑥 − 5𝑦 + 3)𝑑𝑥 + (−2𝑥 − 4𝑦 + 6)𝑑𝑦 = 0

Solving for the intersection, it will yield to h =1 and k = 1.

Then we let:

𝑥 =𝑢+1 𝑑𝑥 = 𝑑𝑢

𝑦=𝑣+1 𝑑𝑦 = 𝑑𝑣

Substituting these to our given equation:

[2(𝑢 + 1) − 5(𝑣 + 1) + 3]𝑑𝑢 + [−2(𝑢 + 1) − 4(𝑣 + 1) + 6]𝑑𝑣 = 0

Compiled by: Kenneth D. Dugay, LPT | 39


Polytechnic University of the Philippines
Santa Rosa Campus

(2𝑢 − 5𝑣 )𝑑𝑢 + (−2𝑢 − 4𝑣)𝑑𝑣 = 0

Let us pick our M(u, v) and let:

𝑢 = 𝑚𝑣 𝑑𝑢 = 𝑚𝑑𝑣 + 𝑣𝑑𝑚

(2𝑚𝑣 − 5𝑣)(𝑚𝑑𝑣 + 𝑣𝑑𝑚) + (−2𝑚𝑣𝑑𝑣 − 4𝑣𝑑𝑣) = 0

2𝑚2 𝑣𝑑𝑣 + 2𝑚𝑣 2 𝑑𝑚 − 5𝑚𝑣𝑑𝑣 − 5𝑣 2 𝑑𝑚 − 2𝑚𝑣𝑑𝑣 − 4𝑣𝑑𝑣 = 0

2𝑚2 𝑣𝑑𝑣 − 7𝑚𝑣𝑑𝑣 − 4𝑣𝑑𝑣 + 2𝑚𝑣 2 𝑑𝑚 − 5𝑣 2 𝑑𝑚 = 0

1
[𝑣(2𝑚2 − 7𝑚 − 4)𝑑𝑣 + 𝑣 2 (2𝑚 − 5)𝑑𝑚 = 0]
(2𝑚2 − 7𝑚 − 4)(𝑣 2 )

𝑑𝑣 2𝑚 − 5
+ 2
𝑑𝑚 = 0
𝑣 2𝑚 − 7𝑚 − 4

𝑑𝑣 2𝑚 − 5
+ 𝑑𝑚 = 0
𝑣 (2𝑚 + 1)(𝑚 − 4)

Solving for the integral of the whole equation:

𝑑𝑣 2𝑚 − 5
∫ +∫ 𝑑𝑚 = 0
𝑣 (2𝑚 + 1)(𝑚 − 4)

Applying partial fractions in our second term:

2𝑚 − 5 𝐴 𝐵
= +
(2𝑚 + 1)(𝑚 − 4) 2𝑚 + 1 𝑚 − 4

2𝑚 − 5 = (𝑚 − 4)𝐴 + (2𝑚 + 1)𝐵

𝑚1 : 2 = 𝐴 + 2𝐵

𝑚0 : − 5 = −4𝐴 + 𝐵

1 4
𝐵=3 𝐴=3

Substituting the values of A and B:

4 1
𝑑𝑣 3
∫ +∫( + 3 ) 𝑑𝑚 = 0
𝑣 2𝑚 + 1 𝑚 − 4

𝑑𝑣 4 𝑑𝑚 1 𝑑𝑚
∫ + ∫ + ∫ =0
𝑣 3 2𝑚 + 1 3 𝑚 − 4

𝑑𝑣 4 1 2𝑑𝑚 1 𝑑𝑚
∫ + ( )∫ + ∫ =0
𝑣 3 2 2𝑚 + 1 3 𝑚 − 4

Compiled by: Kenneth D. Dugay, LPT | 40


Polytechnic University of the Philippines
Santa Rosa Campus

2 1
ln 𝑣 + ln|2𝑚 + 1| + ln|𝑚 − 4| = ln 𝑐
3 3

3 ln 𝑣 + 2 ln|2𝑚 + 1| + ln|𝑚 − 4| = ln 𝑐

Applying Rules of Logarithm:

ln|𝑣 3 (2𝑚 + 1)2 (𝑚 − 4)| = ln 𝑐

3 (2𝑚+1)2 (𝑚−4)|
𝑒 ln|𝑣 = 𝑒 ln 𝑐

𝑣 3 (2𝑚 + 1)2 (𝑚 − 4) = 𝑐

Since,
𝑢
𝑢 = 𝑚𝑣 𝑚=
𝑣

2
2𝑢 𝑢
𝑣 3 ( + 1) ( − 4) = 𝑐
𝑣 𝑣

2𝑢 + 𝑣 2 𝑢 − 4𝑣
𝑣3 ( ) ( )=𝑐
𝑣 𝑣

(2𝑢 + 𝑣)2 𝑢 − 4𝑣
𝑣3 ( )( )=𝑐
𝑣2 𝑣

(2𝑢 + 𝑣)2 (𝑢 − 4𝑣) = 𝑐

But,

𝑥 =𝑢+1 𝑢 =𝑥−1

𝑦=𝑣+1 𝑣 =𝑦−1

[2(𝑥 − 1) + (𝑦 − 1)]2 [𝑥 − 1 − 4(𝑦 − 1)] = 𝑐

(2𝑥 + 𝑦 − 3)2 (𝑥 − 4𝑦 + 3) = 𝑐

Activities/Assessment
Obtain the general solution for the differential equations:

(1) (𝑥 + 2𝑦 − 1)𝑑𝑥 + (𝑥 + 𝑦 − 3)𝑑𝑦 = 0

(2) (6𝑥 − 3𝑦 + 2)𝑑𝑥 + (2𝑥 − 𝑦 − 1)𝑑𝑦 = 0

(3) (𝑥 − 2)𝑑𝑥 + (𝑥 + 𝑦 − 1)𝑑𝑦 = 0

(4) (𝑥 − 𝑦 + 1)𝑑𝑥 + (𝑥 + 4𝑦 + 1)𝑑𝑦 = 0

Compiled by: Kenneth D. Dugay, LPT | 41


Polytechnic University of the Philippines
Santa Rosa Campus

Unit 3: Equations involving Integrating Factors

Overview
For this part of our module, we will be taking a different approach when it comes to solving order
one differential equations. We will again do a test but now to check if an equation is “exact”. A concept in
Differential Calculus which is partial derivatives is also needed. Then we will address the question, “What
if the DE that we are solving is not exact?” Here, we will also be talking about some conditions and when
they are fulfilled, we can make a non-exact differential equation as exact.

As we go deeper in solving for the solution of order one differential equations, there are still
unique forms of equations that we are yet to cover. We will move away from our typical starting form
which is 𝑀𝑑𝑥 + 𝑁𝑑𝑦 = 0, and move on to another form which is the linear equations. Afterwards, we will
discuss a special linear form which is the Bernoulli equations.

Finally, we will be discussing the last method to solve the solution of an order one differential
equation. Previous methods require the equation to be in some form or to adhere to a specific test. Here,
we will be looking for ways to solve our DE when it failed those conditions. Our mastery of the concepts in
this subject will be tested for we need to have some foresight as to what manipulations should be done so
that our equation will be solvable using the methods already discussed in this module.

Lesson 1
Exact Equations

Learning Objective
After successful completion of this module, you should be able to:

▪ recall the use partial derivatives;


▪ test the exactness of an order one differential equation; and
▪ solve for the solution of an exact differential equation.

Course Materials
Lecture Video

(Click the picture for the link or type in: http://bit.ly/ExactDE)

Compiled by: Kenneth D. Dugay, LPT | 42


Polytechnic University of the Philippines
Santa Rosa Campus

Recall: Partial Derivatives Lecture: http://bit.ly/PartialDerive

Click: Solved Problem #1 for Exact Differential Equations: http://bit.ly/ExactDE1

Solved Problem #2 for Exact Differential Equations: http://bit.ly/ExactDE2

Solved Problem #3 for Exact Differential Equations: https://bit.ly/ExactDE3

Solved Problem #4 for Exact Differential Equations: https://bit.ly/ExactDE4

Discussions

Partial Derivatives

A partial is derivative is done by taking the derivative of a certain variable while considering other
variables as constant. We take the derivative of a function, say f, with multiple variables with respect to a
specific variable. It is represented as:

𝜕𝑓
(Partial Derivative of function f with respect to x)
𝜕𝑥

𝜕𝑓
(Partial Derivative of function f with respect to y)
𝜕𝑦

𝜕𝑓
(Partial Derivative of function f with respect to z)
𝜕𝑧

Example. Evaluate the following partial derivatives.

𝜕𝑓 𝜕𝑓
a) b)
𝜕𝑥 𝜕𝑦

𝑓(𝑥, 𝑦) = 𝑥 2 𝑦 + 5𝑥𝑦 + 𝑦 2

𝜕𝑓 𝜕
a) = 𝜕𝑥 (𝑥2 𝑦 + 5𝑥𝑦 + 𝑦2 )
𝜕𝑥

For this example, we treat y as a constant. Then taking the derivative with respect to x will result to:

𝜕𝑓
= 2𝑥𝑦 + 5𝑦
𝜕𝑥

Notice that the third term 𝑦 2 becomes zero. It is because when taking the partial derivative with respect to
x, other variables are considered to be constants. Hence, the derivative of a constant is just equal to zero.

Compiled by: Kenneth D. Dugay, LPT | 43


Polytechnic University of the Philippines
Santa Rosa Campus

𝜕𝑓 𝜕
b) = 𝜕𝑦 (𝑥2 𝑦 + 5𝑥𝑦 + 𝑦2 )
𝜕𝑦

Now we will be treating x as a constant then solve for the derivative with respect to y. Then our answer
will be:

𝜕𝑓
= 𝑥 2 + 5𝑥 + 2𝑦
𝜕𝑦

This short review about partial derivatives was included here in preparation for the main topic for
this part of our module. Now that we are done with our preliminary discussion, let us talk about exact
equations.

Exact Differential Equations

Given a DE of order one in the form:

𝑀𝑑𝑥 + 𝑁𝑑𝑦 = 0

where M and N are functions of x and y. A differential equation is exact if it satisfied the condition:

𝜕𝑀 𝜕𝑁
=
𝜕𝑦 𝜕𝑥

Then if the equation satisfies the condition for exactness, we can say that the following are true for the
solution F:

𝜕𝐹 𝜕𝐹
=𝑀 and =𝑁
𝜕𝑥 𝜕𝑦

Example 1. Test the exactness of the following differential equations:

(3𝑥 2 + 𝑦 2 )𝑑𝑥 + 2𝑥𝑦𝑑𝑦 = 0

𝜕𝑀 𝜕
= (3𝑥 2 + 𝑦 2 ) = 2𝑦
𝜕𝑦 𝜕𝑦

𝜕𝑁 𝜕
= (2𝑥𝑦) = 2𝑦
𝜕𝑥 𝜕𝑥
𝜕𝑀 𝜕𝑁
Since both and are equal to 2y, then the differential equation is exact!
𝜕𝑦 𝜕𝑥

Compiled by: Kenneth D. Dugay, LPT | 44


Polytechnic University of the Philippines
Santa Rosa Campus

Example 2. Test the exactness of the following differential equations:

𝑥𝑒 𝑥𝑦 𝑑𝑥 + 𝑦𝑒 𝑥𝑦 𝑑𝑦 = 0

Let us start with taking the partial derivative of M with respect to y:

𝜕𝑀 𝜕
= (𝑥𝑒 𝑥𝑦 )
𝜕𝑦 𝜕𝑦

Since x is treated as a constant, we only have to take the derivative of 𝑒 𝑥𝑦

𝜕𝑀
= 𝑥(𝑥𝑒 𝑥𝑦 ) = 𝑥 2 𝑒 𝑥𝑦
𝜕𝑦

Now doing the partial derivatives of N with respect to x:

𝜕𝑁 𝜕
= (𝑦𝑒 𝑥𝑦 𝑑𝑦)
𝜕𝑥 𝜕𝑥

𝜕𝑁
= 𝑦(𝑦𝑒 𝑥𝑦 ) = 𝑦 2 𝑒 𝑥𝑦
𝜕𝑥
𝜕𝑀 𝜕𝑁
Since is not equal to , then our differential equation is NOT exact.
𝜕𝑦 𝜕𝑥

Steps in Solving for the Solution of Exact Differential Equations

(1) Transform the given DE in the form:

𝑀𝑑𝑥 + 𝑁𝑑𝑦 = 0

(2) Test the differential equation for exactness. Exact DE should follow the condition that:

𝜕𝑀 𝜕𝑁
=
𝜕𝑦 𝜕𝑥

Note that these are true for an exact DE:

𝜕𝐹 𝜕𝐹
=𝑀 and =𝑁
𝜕𝑥 𝜕𝑦

𝜕𝐹
(3) Take
𝜕𝑥
= 𝑀, then multiply both sides by 𝜕𝑥 to arrive at 𝜕𝐹 = 𝑀𝜕𝑥. Solve for the integral partially
with respect to x (treating y as a constant) to arrive at the value of F.

∫ 𝑑𝐹 = ∫ 𝑀𝑑𝑥 + 𝜙(𝑦)

Notice that we added 𝜙(𝑦) as the constant of integration since we are taking the integral partially.

𝜕𝐹
(5) Now that we have the value of F, solve for . Doing this will make our 𝜙(𝑦) be 𝜙′(𝑦)
𝜕𝑦

Compiled by: Kenneth D. Dugay, LPT | 45


Polytechnic University of the Philippines
Santa Rosa Campus

𝜕𝐹 𝜕𝐹
(6) Then let us equate our given =𝑁 to our solved to have the value of 𝜙′(𝑦) solely as function
𝜕𝑦 𝜕𝑦
of y.

(7) Solve for the value of 𝜙(𝑦) by taking the integral of our previously solved 𝜙′(𝑦) now as a function of y
then substitute it to our equation of F to arrive at our solution.

𝜕𝐹
As a final note, you can also solve for the DE by taking = 𝑁 at the start of the equation. You just have
𝜕𝑦
to interchange the roles of x and y in your solution.

Example 1. Obtain the general solution for the differential equation:

(𝑥 + 2𝑦)𝑑𝑥 + (2𝑥 + 𝑦)𝑑𝑦 = 0

Testing for exactness:

𝜕𝑀 𝜕
= (𝑥 + 2𝑦) = 2
𝜕𝑦 𝜕𝑦

𝜕𝑁 𝜕
= (2𝑥 + 𝑦) = 2
𝜕𝑥 𝜕𝑥
𝜕𝑀 𝜕𝑁
Since = , then our DE is exact.
𝜕𝑦 𝜕𝑥

Now let us take:

𝜕𝐹
=𝑀
𝜕𝑥

𝜕𝐹 = 𝑀𝜕𝑥

∫ 𝑑𝐹 = ∫(𝑥 + 2𝑦)𝑑𝑥 + 𝜙(𝑦) (where 𝜙(𝑦) is the constant of integration)

Taking its integral with respect to x while treating the other variable(s) as constant:

𝑥2
𝐹= + 2𝑥𝑦 + 𝜙(𝑦) (1)
2

Notice that we included 𝜙(𝑦) to compensate for the fact that we are getting the integral partially. Next, we
have to solve the value of 𝜙(𝑦) to complete the solution. To do that, let us take the derivative of F partially
now with respect to y:

𝜕𝐹
= 2𝑥 + 𝜙′(𝑦)
𝜕𝑦

𝜕𝐹
But
𝜕𝑦
= 𝑁 = 2𝑥 + 𝑦, so equating this to our previous equation,

2𝑥 + 𝜙 ′ (𝑦) = 2𝑥 + 𝑦

2𝑥 + 𝜙 ′ (𝑦) = 2𝑥 + 𝑦

Compiled by: Kenneth D. Dugay, LPT | 46


Polytechnic University of the Philippines
Santa Rosa Campus

𝜙 ′ (𝑦) = 𝑦

Then taking its integral to solve for the value of 𝜙(𝑦),

∫ 𝜙 ′ (𝑦) = ∫ 𝑦 𝑑𝑦

𝑦2
𝜙(𝑦) =
2
Substituting it to (1), our equation with F:

𝑥2 𝑦2
𝐹= + 2𝑥𝑦 +
2 2

Finally, our solution including our constant c is:

𝑥2 𝑦2
+ 2𝑥𝑦 + =𝑐
2 2

𝑥 2 + 4𝑥𝑦 + 2𝑦 2 = 𝑐

Example 2. Obtain the general solution for the differential equation:

(𝑦 2 − 2𝑥𝑦 + 6𝑥)𝑑𝑥 − (𝑥 2 − 2𝑥𝑦 + 2)𝑑𝑦 = 0

(𝑦 2 − 2𝑥𝑦 + 6𝑥)𝑑𝑥 + (−𝑥 2 + 2𝑥𝑦 − 2)𝑑𝑦 = 0

Testing for exactness:

𝜕𝑀 𝜕 2
= (𝑦 − 2𝑥𝑦 + 6𝑥) = 2𝑦 − 2𝑥
𝜕𝑦 𝜕𝑦

𝜕𝑁 𝜕
= (−𝑥 2 + 2𝑥𝑦 − 2) = −2𝑥 + 2𝑦 = 2𝑦 − 2𝑥
𝜕𝑥 𝜕𝑥
𝜕𝐹
Our DE is exact. Now we take =𝑀
𝜕𝑥

∫ 𝑑𝐹 = ∫ 𝑀𝑑𝑥 + 𝜙(𝑦)

𝐹 = ∫(𝑦 2 − 2𝑥𝑦 + 6𝑥)𝑑𝑥 + 𝜙(𝑦)

Treating y as a constant,

𝐹 = 𝑦 2 ∫ 𝑑𝑥 − 2𝑦 ∫ 𝑥𝑑𝑥 + 6 ∫ 𝑥𝑑𝑥 + 𝜙(𝑦)

𝑥2 𝑥2
𝐹 = 𝑦 2 (𝑥) − 2𝑦 ( ) + 6 ( ) + 𝜙(𝑦)
2 2

Compiled by: Kenneth D. Dugay, LPT | 47


Polytechnic University of the Philippines
Santa Rosa Campus

𝐹 = 𝑥𝑦 2 − 𝑥 2 𝑦 + 3𝑥 2 + 𝜙(𝑦) (1)

Now taking its partial derivative with respect to y:

𝜕𝐹 𝜕
= (𝑥𝑦 2 − 𝑥 2 𝑦 + 3𝑥 2 + 𝜙(𝑦))
𝜕𝑦 𝜕𝑦

𝜕𝐹
= 2𝑥𝑦 − 𝑥 2 + 𝜙′(𝑦)
𝜕𝑦

But:

𝜕𝐹
= 𝑁 = −𝑥 2 + 2𝑥𝑦 − 2
𝜕𝑦

𝜕𝐹 𝜕𝐹
Then = ,
𝜕𝑦 𝜕𝑦

2𝑥𝑦 − 𝑥 2 + 𝜙 ′ (𝑦) = −𝑥 2 + 2𝑥𝑦 − 2

𝜙 ′ (𝑦) = −2

Solving for its integral:

∫ 𝜙 ′ (𝑦) = ∫ −2𝑑𝑦

𝜙(𝑦) = −2𝑦

Substituting it to (1),

𝐹 = 𝑥𝑦 2 − 𝑥 2 𝑦 + 3𝑥 2 − 2𝑦

Our solution is:

𝑥𝑦 2 − 𝑥 2 𝑦 + 3𝑥 2 − 2𝑦 = 𝑐

Example 3. Obtain the general solution for the differential equation:

(𝑦 cos 𝑥 + 2𝑥𝑒 𝑦 ) + (sin 𝑥 + 𝑥 2 𝑒 𝑦 − 1)𝑦 ′ = 0

Testing for exactness:

𝜕𝑀 𝜕
= (𝑦 cos 𝑥 + 2𝑥𝑒 𝑦 ) = cos 𝑥 + 2𝑥𝑒 𝑦
𝜕𝑦 𝜕𝑦

𝜕𝑁 𝜕
= (sin 𝑥 + 𝑥 2 𝑒 𝑦 − 1) = cos 𝑥 + 2𝑥𝑒 𝑦
𝜕𝑥 𝜕𝑥

Compiled by: Kenneth D. Dugay, LPT | 48


Polytechnic University of the Philippines
Santa Rosa Campus

𝜕𝐹
Now that are DE is exact, let us take =𝑀
𝜕𝑥

∫ 𝑑𝐹 = ∫ 𝑀𝑑𝑥 + 𝜙(𝑦)

𝐹 = ∫(𝑦 cos 𝑥 + 2𝑥𝑒 𝑦 )𝑑𝑥 + 𝜙(𝑦)

Treating y as a constant,

𝐹 = 𝑦 ∫ cos 𝑥 𝑑𝑥 + 2𝑒 𝑦 ∫ 𝑥 𝑑𝑥 + 𝜙(𝑦)

𝐹 = 𝑦 sin 𝑥 + 𝑥 2 𝑒 𝑦 + 𝜙(𝑦) (1)

Solving for the partial derivative with respect to y:

𝜕𝐹 𝜕
= (𝑦 sin 𝑥 + 𝑥 2 𝑒 𝑦 + 𝜙(𝑦))
𝜕𝑦 𝜕𝑦

𝜕𝐹
= sin 𝑥 + 𝑥 2 𝑒 𝑦 + 𝜙′(𝑦)
𝜕𝑦

But for exact equations,

𝜕𝐹
= 𝑁 = sin 𝑥 + 𝑥 2 𝑒 𝑦 − 1
𝜕𝑦

Then,

sin 𝑥 + 𝑥 2 𝑒 𝑦 + 𝜙 ′ (𝑦) = sin 𝑥 + 𝑥 2 𝑒 𝑦 − 1

𝜙 ′ (𝑦) = −1

∫ 𝜙 ′ (𝑦) = ∫(−1)𝑑𝑦

𝜙(𝑦) = −𝑦

Substituting back to (1), we arrive at our solution:

sin 𝑥 + 𝑥 2 𝑒 𝑦 − 𝑦 = 𝑐

Example 4. Obtain the general solution for the differential equation:

(cos 2𝑦 − 3𝑥 2 𝑦 2 )𝑑𝑥 + (cos 2𝑦 − 2𝑥 sin 2𝑦 − 2𝑥 3 𝑦)𝑑𝑦 = 0

Testing for exactness:

𝜕𝑀 𝜕
= (cos 2𝑦 − 3𝑥 2 𝑦 2 ) = −2sin 2𝑦 − 6𝑥 2 𝑦
𝜕𝑦 𝜕𝑦

Compiled by: Kenneth D. Dugay, LPT | 49


Polytechnic University of the Philippines
Santa Rosa Campus

𝜕𝑁 𝜕
= (cos 2𝑦 − 2𝑥 sin 2𝑦 − 2𝑥 3 𝑦) = −2sin 2𝑦 − 6𝑥 2 𝑦
𝜕𝑥 𝜕𝑥
𝜕𝐹
Taking =𝑀
𝜕𝑥

∫ 𝑑𝐹 = ∫ 𝑀𝑑𝑥 + 𝜙(𝑦)

𝐹 = ∫(cos 2𝑦 − 3𝑥 2 𝑦 2 )𝑑𝑥 + 𝜙(𝑦)

Solving for the integral while considering y as a constant:

𝐹 = cos 2𝑦 ∫ 𝑑𝑥 − 3𝑦 2 ∫ 𝑥 2 𝑑𝑥 + 𝜙(𝑦)

𝐹 = 𝑥 cos 2𝑦 − 𝑥 3 𝑦 2 + 𝜙(𝑦) (1)

Solving for the partial derivative with respect to y:

𝜕𝐹 𝜕
= (𝑥 cos 2𝑦 − 𝑥 3 𝑦 2 + 𝜙(𝑦))
𝜕𝑦 𝜕𝑦

𝜕𝐹
= −2𝑥 sin 2𝑦 − 2𝑥 3 𝑦 + 𝜙′(𝑦)
𝜕𝑦

But for exact equations,

𝜕𝐹
= 𝑁 = cos 2𝑦 − 2𝑥 sin 2𝑦 − 2𝑥 3 𝑦
𝜕𝑦

Then,

−2𝑥 sin 2𝑦 − 2𝑥 3 𝑦 + 𝜙 ′ (𝑦) = cos 2𝑦 − 2𝑥 sin 2𝑦 − 2𝑥 3 𝑦

𝜙 ′ (𝑦) = cos 2𝑦

∫ 𝜙 ′ (𝑦) = ∫ cos 2𝑦 𝑑𝑦

1
𝜙(𝑦) = ∫ 2 cos 2𝑦 𝑑𝑦
2

1
𝜙(𝑦) = sin 2𝑦
2

Then our solution by substituting 𝜙(𝑦) to (1) is:


1
𝑥 cos 2𝑦 − 𝑥 3 𝑦 2 + sin 2𝑦 = 𝑐
2

2𝑥 cos 2𝑦 − 2𝑥 3 𝑦 2 + sin 2𝑦 = 𝑐

Compiled by: Kenneth D. Dugay, LPT | 50


Polytechnic University of the Philippines
Santa Rosa Campus

Activities/Assessment
Obtain the general solution for the differential equations:

(1) (2𝑥𝑦 − 3𝑥 2 )𝑑𝑥 + (𝑥 2 + 2𝑦)𝑑𝑦 = 0

(2) (2𝑥𝑦)𝑑𝑥 + (𝑦 2 + 𝑥 2 )𝑑𝑦 = 0

(3) 𝑥𝑒 𝑥𝑦 𝑑𝑥 + 𝑦𝑒 𝑥𝑦 𝑑𝑦 = 0
𝑥
(4) (cos 𝑥 + ln 𝑦)𝑑𝑥 + (𝑦 + 𝑒 𝑦 ) 𝑑𝑦 = 0

𝑥+𝑦
(5) (𝑥 + tan−1 𝑦)𝑑𝑥 + 1+𝑦 2 𝑑𝑦 = 0

Compiled by: Kenneth D. Dugay, LPT | 51


Polytechnic University of the Philippines
Santa Rosa Campus

Lesson 2
Integrating Factors by Formula

Learning Objective
After successful completion of this module, you should be able to:

▪ test if a non-exact differential equation can be made exact;


▪ determine the possible integrating factor of non-exact differential equations; and
▪ solve for the solution of differential equations using integrating factors.

Course Materials
Lecture Video

(Click the picture for the link or type in: https://bit.ly/IntegFactor)

Click: Solved Problem #1 for Integrating Factor: https://bit.ly/IntegFactor1

Solved Problem #2 for Integrating Factor: https://bit.ly/IntegFactor2

Solved Problem #3 for Integrating Factor: https://bit.ly/IntegFactor3

Solved Problem #4 for Integrating Factor: https://bit.ly/IntegFactor4

Discussions

Integrating Factor

An integrating factor can be multiplied to a non-exact DE to transform it to an exact DE. But not
all differential equations have an integrating factor. To determine if an integrating factor can be drawn,
given a DE in the form:

𝑀𝑑𝑥 + 𝑁𝑑𝑦 = 0

one of the two following conditions must be met:

1 𝜕𝑀 𝜕𝑁
𝑁
( 𝜕𝑦 − 𝜕𝑥 ) = 𝑓(𝑥) and its integrating factor, 𝐼 , is equal to: 𝐼 = 𝑒 ∫ 𝑓(𝑥)𝑑𝑥

Compiled by: Kenneth D. Dugay, LPT | 52


Polytechnic University of the Philippines
Santa Rosa Campus

or:

1 𝜕𝑀 𝜕𝑁
( 𝜕𝑦 − 𝜕𝑥 ) = 𝑔(𝑦) and its integrating factor, 𝐼 , is equal to: 𝐼 = 𝑒 − ∫ 𝑔(𝑦)𝑑𝑦
𝑀

where 𝑓(𝑥) is solely a function of x, and 𝑔(𝑦) is solely a function of y. Also, do not neglect negative sign
in the exponent of the integrating factor for 𝑔(𝑦) since it is a part of the formula.

Example 1. Solve for the integrating factor of the given differential equations:

𝑦 2 𝑑𝑥 + 𝑥𝑦𝑑𝑦 = 0

𝜕𝑀 𝜕 2 𝜕𝑁 𝜕
= 𝑦 = 2𝑦 = 𝑥𝑦 = 𝑦
𝜕𝑦 𝜕𝑦 𝜕𝑥 𝜕𝑥

Our DE is not exact. Let us now test if we can get an integrating factor.

1 𝜕𝑀 𝜕𝑁 1 𝑦 1
( − )= (2𝑦 − 𝑦) = =
𝑁 𝜕𝑦 𝜕𝑥 𝑥𝑦 𝑥𝑦 𝑥

1
𝑓(𝑥) =
𝑥

Since the result is already a function of x, we do not need to proceed with our second formula. Solving for
𝐼:

𝐼 = 𝑒∫ 𝑓(𝑥)𝑑𝑥
1
𝐼 = 𝑒∫𝑥𝑑𝑥
𝑑𝑥
𝐼 = 𝑒∫ 𝑥

𝐼 = 𝑒 ln 𝑥

𝐼=𝑥

Example 2. Solve for the integrating factor of the given differential equations:

(2𝑥𝑦 2 − 2𝑦)𝑑𝑥 + (3𝑥 2 𝑦 − 4𝑥)𝑑𝑦 = 0

𝜕𝑀 𝜕 𝜕𝑁 𝜕
= (2𝑥𝑦 2 − 2𝑦) = 4𝑥𝑦 − 2 = (3𝑥 2 𝑦 − 4𝑥) = 6𝑥𝑦 − 4
𝜕𝑦 𝜕𝑦 𝜕𝑥 𝜕𝑥

Doing our first test:

1 𝜕𝑀 𝜕𝑁 1
( − )= 2 [4𝑥𝑦 − 2 − (6𝑥𝑦 − 4)]
𝑁 𝜕𝑦 𝜕𝑥 3𝑥 𝑦 − 4𝑥

Compiled by: Kenneth D. Dugay, LPT | 53


Polytechnic University of the Philippines
Santa Rosa Campus

−2𝑥𝑦 + 2
=
3𝑥 2 𝑦 − 4𝑥

2(−𝑥𝑦 + 1)
=
𝑥(3𝑥𝑦 − 4)

Since we cannot represent it solely as function of x, let us proceed with our second test.

1 𝜕𝑀 𝜕𝑁 1
( − )= 2
[4𝑥𝑦 − 2 − (6𝑥𝑦 − 4)]
𝑀 𝜕𝑦 𝜕𝑥 2𝑥𝑦 − 2𝑦

−2𝑥𝑦 + 2
=
2𝑥𝑦 2 − 2𝑦

−2(𝑥𝑦 − 1)
=
2𝑦(𝑥𝑦 − 1)

1 𝜕𝑀 𝜕𝑁 1
( − )=−
𝑀 𝜕𝑦 𝜕𝑥 𝑦

1
𝑔(𝑦) = −
𝑦

We were able to solve for a function of y using the second formula for integrating factor. We can now
solve for 𝐼:

𝐼 = 𝑒− ∫ 𝑔(𝑦)𝑑𝑦
− ∫ −𝑦1 𝑑𝑦
𝐼 =𝑒 = 𝑒ln 𝑦 = 𝑦
𝑑𝑦
∫𝑦
𝐼=𝑒

𝐼 = 𝑒 ln 𝑦

𝐼=𝑦

Steps in Solving for Non-Exact DE using Integrating Factor

(1) Transform the given DE in the form:

𝑀𝑑𝑥 + 𝑁𝑑𝑦 = 0

(2) Test if the DE has an integrating factor for it to be an exact DE using:

1 𝜕𝑀 𝜕𝑁
( − ) = 𝑓(𝑥)
𝑁 𝜕𝑦 𝜕𝑥

1 𝜕𝑀 𝜕𝑁
( − ) = 𝑔(𝑦)
𝑀 𝜕𝑦 𝜕𝑥

Compiled by: Kenneth D. Dugay, LPT | 54


Polytechnic University of the Philippines
Santa Rosa Campus

(3) If any two of the conditions were satisfied, solve for the integrating factor by:

𝐼 = 𝑒 ∫ 𝑓(𝑥)𝑑𝑥

𝐼 = 𝑒 − ∫ 𝑔(𝑦)𝑑𝑦

(4) Multiply the integrating factor, 𝐼 , to the given equation. It will result to an exact differential equation.

(5) Follow the steps in solving for the solutions of exact DE.

Example 1. Obtain the general solution for the differential equation:

(𝑥 2 + 𝑦 2 + 1)𝑑𝑥 + 𝑥(𝑥 − 2𝑦)𝑑𝑦 = 0

(𝑥 2 + 𝑦 2 + 1)𝑑𝑥 + (𝑥 2 − 2𝑥𝑦)𝑑𝑦 = 0

Testing for exactness:

𝜕𝑀 𝜕 2 𝜕𝑁 𝜕 2
= (𝑥 + 𝑦 2 + 1) = 2𝑦 = (𝑥 − 2𝑥𝑦) = 2𝑥 − 2𝑦
𝜕𝑦 𝜕𝑦 𝜕𝑥 𝜕𝑥

Since they are not exact, let us look for the integrating factor.

𝜕𝑀 𝜕𝑁
− = 2𝑦 − (2𝑥 − 2𝑦) = −2𝑥 + 4𝑦
𝜕𝑦 𝜕𝑥

1 𝜕𝑀 𝜕𝑁 1
( − )= (−2𝑥 + 4𝑦)
𝑁 𝜕𝑦 𝜕𝑥 𝑥(𝑥 − 2𝑦)

−2(𝑥 − 2𝑦)
=
𝑥(𝑥 − 2𝑦)

1 𝜕𝑀 𝜕𝑁 −2
( − )= = 𝑓(𝑥)
𝑁 𝜕𝑦 𝜕𝑥 𝑥

Since it is already a function of x, let us apply use the formula to solve the integrating factor

𝐼 = 𝑒 ∫ 𝑓(𝑥)𝑑𝑥
−2 𝑑𝑥 −2
𝐼 = 𝑒 ∫ 𝑥 𝑑𝑥 = 𝑒 −2 ∫ 𝑥 = 𝑒 −2 ln 𝑥 = 𝑒 ln 𝑥 = 𝑥 −2

Now let us multiply the integrating factor 𝐼 to the given equation:

[(𝑥 2 + 𝑦 2 + 1)𝑑𝑥 + (𝑥 2 − 2𝑥𝑦)𝑑𝑦 = 0] 𝐼

[(𝑥 2 + 𝑦 2 + 1)𝑑𝑥 + (𝑥 2 − 2𝑥𝑦)𝑑𝑦 = 0] 𝑥 −2

(1 + 𝑥 −2 𝑦 2 + 𝑥 −2 )𝑑𝑥 + (1 − 2𝑥 −1 𝑦)𝑑𝑦 = 0

Compiled by: Kenneth D. Dugay, LPT | 55


Polytechnic University of the Philippines
Santa Rosa Campus

This equation is now exact. The following steps are done to solve the solution of an exact differential
equation:

𝜕𝐹
=𝑀
𝜕𝑥

∫ 𝑑𝐹 = ∫ 𝑀 𝑑𝑥 + 𝜙(𝑦)

𝐹 = ∫(1 + 𝑥 −2 𝑦 2 + 𝑥 −2 )𝑑𝑥 + 𝜙(𝑦)

𝐹 = ∫ 𝑑𝑥 + 𝑦 2 ∫ 𝑥 −2 𝑑𝑥 + ∫ 𝑥 −2 𝑑𝑥 + 𝜙(𝑦)

𝐹 = 𝑥 − 𝑥 −1 𝑦 2 − 𝑥 −1 + 𝜙(𝑦)

Taking the partial derivative of F with respect to y:

𝜕𝐹 𝜕
= [𝑥 − 𝑥 −1 𝑦 2 − 𝑥 −1 + 𝜙(𝑦)]
𝜕𝑦 𝜕𝑦

𝜕𝐹
= −2𝑥 −1 𝑦 + 𝜙′(𝑦)
𝜕𝑦

𝜕𝐹
= 𝑁 = 1 − 2𝑥 −1 𝑦
𝜕𝑦

−2𝑥 −1 𝑦 + 𝜙 ′ (𝑦) = 1 − 2𝑥 −1 𝑦

𝜙 ′ (𝑦) = 1

Taking the integral of the equation to solve 𝜙(𝑦)

∫ 𝜙 ′ (𝑦) = ∫(1)𝑑𝑦

𝜙(𝑦) = 𝑦

Then our solution is:

𝑥 − 𝑥 −1 𝑦 2 − 𝑥 −1 + 𝑦 = 𝑐 or 𝑥 2 − 𝑦 2 + 𝑥𝑦 − 1 = 𝑐𝑥

Example 2. Obtain the general solution for the differential equation:

𝑦(4𝑥 + 𝑦)𝑑𝑥 − 2(𝑥 2 − 𝑦)𝑑𝑦 = 0

(4𝑥𝑦 + 𝑦 2 )𝑑𝑥 + (−2𝑥 2 + 2𝑦)𝑑𝑦 = 0

Compiled by: Kenneth D. Dugay, LPT | 56


Polytechnic University of the Philippines
Santa Rosa Campus

Testing for exactness:

𝜕𝑀 𝜕 𝜕𝑁 𝜕
= (4𝑥𝑦 + 𝑦 2 ) = 4𝑥 + 2𝑦 = (−2𝑥 2 + 2𝑦) = −4𝑥
𝜕𝑦 𝜕𝑦 𝜕𝑥 𝜕𝑥

Proceeding with the test for integrating factor:

𝜕𝑀 𝜕𝑁
− = 4𝑥 + 2𝑦 − (−4𝑥) = 8𝑥 + 2𝑦 = 2(4𝑥 + 𝑦)
𝜕𝑦 𝜕𝑥

1 𝜕𝑀 𝜕𝑁 1
( − )= [2(2𝑥 + 4)]
𝑁 𝜕𝑦 𝜕𝑥 −2(𝑥 2 − 𝑦)

1 𝜕𝑀 𝜕𝑁 2(2𝑥 + 4)
( − )=
𝑁 𝜕𝑦 𝜕𝑥 −2(𝑥 2 − 𝑦)

Since it is not solely a function of x, let us proceed with the second formula for the test of integrating
factor:

1 𝜕𝑀 𝜕𝑁 1
( − )= [2(4𝑥 + 𝑦)]
𝑀 𝜕𝑦 𝜕𝑥 𝑦(4𝑥 + 𝑦)

1 𝜕𝑀 𝜕𝑁 2(4𝑥 + 𝑦) 2
( − )= = = 𝑔(𝑦)
𝑀 𝜕𝑦 𝜕𝑥 𝑦(4𝑥 + 𝑦) 𝑦

Now that it is solely a function of y, we may now obtain the integrating factor:

𝐼 = 𝑒 − ∫ 𝑔(𝑦)𝑑𝑦
2 𝑑𝑦
− ∫ 𝑑𝑦 −2 ∫ −2
𝐼=𝑒 𝑦 =𝑒 𝑦 = 𝑒 −2 ln 𝑦 = 𝑒 ln 𝑦 = 𝑦 −2

Then, multiplying the integrating factor to our given equation:

[(4𝑥𝑦 + 𝑦 2 )𝑑𝑥 + (−2𝑥 2 + 2𝑦)𝑑𝑦 = 0] 𝐼

[(4𝑥𝑦 + 𝑦 2 )𝑑𝑥 + (−2𝑥 2 + 2𝑦)𝑑𝑦 = 0] 𝑦 −2

(4𝑥𝑦 −1 + 1)𝑑𝑥 + (−2𝑥 2 𝑦 −2 + 2𝑦 −1 )𝑑𝑦 = 0

Solving for the solution of the now exact DE:

𝜕𝐹
=𝑀
𝜕𝑥

∫ 𝑑𝐹 = ∫(4𝑥𝑦 −1 + 1) 𝑑𝑥 + 𝜙(𝑦)

𝐹 = 2𝑥 2 𝑦 −1 + 𝑥 + 𝜙(𝑦)

Compiled by: Kenneth D. Dugay, LPT | 57


Polytechnic University of the Philippines
Santa Rosa Campus

𝜕𝐹
= −2𝑥 2 𝑦 −2 + 𝜙′(𝑦)
𝜕𝑦

𝜕𝐹
= 𝑁 = −2𝑥 2 𝑦 −2 + 2𝑦 −1
𝜕𝑦

−2𝑥 2 𝑦 −2 + 𝜙′(𝑦) = −2𝑥 2 𝑦 −2 + 2𝑦 −1

𝜙′(𝑦) = 2𝑦 −1

∫ 𝜙′(𝑦) = ∫ 2𝑦 −1 𝑑𝑦

𝑑𝑦
𝜙(𝑦) = 2 ∫
𝑦

𝜙(𝑦) = 2 ln 𝑦 = ln 𝑦 2

The solution of our DE is:

2𝑥 2 𝑦 −1 + 𝑥 + ln 𝑦 2 = ln 𝑐

2𝑥 2 𝑦 −1 + 𝑥 + ln 𝑦 2 + ln 𝑐 = 0

2𝑥 2 + 𝑥𝑦 + 𝑦 ln 𝑦 2 + 𝑦 ln 𝑐 = 0

2𝑥 2 + 𝑥𝑦 + 𝑦(ln 𝑦 2 + ln 𝑐) = 0

2𝑥 2 + 𝑥𝑦 + 𝑦 ln 𝑐𝑦 2 = 0

Example 3. Obtain the general solution for the differential equation:

𝑦 ′ = 2𝑥𝑦 − 𝑥

𝑑𝑦
= 2𝑥𝑦 − 𝑥
𝑑𝑥

(2𝑥𝑦 − 𝑥)𝑑𝑥 + (−𝑑𝑦) = 0

Testing for exactness:

𝜕𝑀 𝜕 𝜕𝑁 𝜕
= (2𝑥𝑦 − 𝑥) = 2𝑥 = (−1) = 0
𝜕𝑦 𝜕𝑦 𝜕𝑥 𝜕𝑥

𝜕𝑀 𝜕𝑁
− = 2𝑥 − 0 = 2𝑥
𝜕𝑦 𝜕𝑥

Testing for the integrating factor:

1 𝜕𝑀 𝜕𝑁 1
( − )= (2𝑥) = −2𝑥 = 𝑓(𝑥)
𝑁 𝜕𝑦 𝜕𝑥 −1

Compiled by: Kenneth D. Dugay, LPT | 58


Polytechnic University of the Philippines
Santa Rosa Campus

𝐼 = 𝑒 ∫ 𝑓(𝑥)𝑑𝑥
2
𝐼 = 𝑒 ∫ −2𝑥𝑑𝑥 = 𝑒 −𝑥

Multiplying 𝐼 to the given equation:


2
[(2𝑥𝑦 − 𝑥)𝑑𝑥 + (−𝑑𝑦) = 0]𝑒 −𝑥
2 2 2
(2𝑥𝑦𝑒 −𝑥 − 𝑥𝑒 −𝑥 )𝑑𝑥 + (−𝑒 −𝑥 )𝑑𝑦 = 0

𝜕𝐹
Taking = 𝑁 and interchanging x and y in the solution for exact DE since N is much simpler:
𝜕𝑦

∫ 𝑑𝐹 = ∫ 𝑁 𝑑𝑦 + 𝜙(𝑥)

2
𝐹 = ∫ −𝑒 −𝑥 𝑑𝑦 + 𝜙(𝑥)

Since we take x as a constant:

2
𝐹 = −𝑒 −𝑥 ∫ 𝑑𝑦 + 𝜙(𝑥)

2
𝐹 = −𝑦𝑒 −𝑥 + 𝜙(𝑥)

Taking the partial derivative of F with respect to x:

𝜕𝐹 2
= 2𝑥𝑦𝑒 −𝑥 + 𝜙′(𝑥)
𝜕𝑥

𝜕𝐹 2 2
= 𝑀 = 2𝑥𝑦𝑒 −𝑥 − 𝑥𝑒 −𝑥
𝜕𝑥
2 2 2
2𝑥𝑦𝑒 −𝑥 + 𝜙 ′ (𝑥) = 2𝑥𝑦𝑒 −𝑥 − 𝑥𝑒 −𝑥
2
𝜙 ′ (𝑥) = −𝑥𝑒 −𝑥

Solving for 𝜙(𝑥) by taking the integral of the equation:

2
∫ 𝜙 ′ (𝑥) = ∫ −𝑥𝑒 −𝑥 𝑑𝑥

1 2
𝜙(𝑥) = ∫ −2𝑥𝑒 −𝑥 𝑑𝑥
2

1 2
𝜙(𝑥) = 𝑒 −𝑥
2

Compiled by: Kenneth D. Dugay, LPT | 59


Polytechnic University of the Philippines
Santa Rosa Campus

Then our solution is:

2 1 2
𝑐 = −𝑦𝑒 −𝑥 + 𝑒 −𝑥
2
2
Simplifying by multiplying both sides of the equation by 𝑒 𝑥 :

1 2
−𝑦 + = 𝑐𝑒 𝑥
2

2 1
𝑦 = 𝑐𝑒 𝑥 +
2

Example 4. Obtain the general solution for the differential equation:

𝑦 2 cos 𝑥 𝑑𝑥 + (4 + 5𝑦 sin 𝑥)𝑑𝑦 = 0

Testing for exactness:

𝜕𝑀 𝜕 2 𝜕𝑁 𝜕
= (𝑦 cos 𝑥) = 2𝑦 cos 𝑥 = (4 + 5𝑦 sin 𝑥) = 5𝑦 cos 𝑥
𝜕𝑦 𝜕𝑦 𝜕𝑥 𝜕𝑥

𝜕𝑀 𝜕𝑁
− = 2𝑦 cos 𝑥 − 5𝑦 cos 𝑥 0 = −3𝑦 cos 𝑥
𝜕𝑦 𝜕𝑥

Testing for the existence of an integrating factor:

1 𝜕𝑀 𝜕𝑁 −3𝑦 cos 𝑥
( − )=
𝑁 𝜕𝑦 𝜕𝑥 4 + 5𝑦 sin 𝑥

Since this is not a function of x,

1 𝜕𝑀 𝜕𝑁 −3𝑦 cos 𝑥 −3
( − )= 2 = = 𝑔(𝑦)
𝑀 𝜕𝑦 𝜕𝑥 𝑦 cos 𝑥 𝑦

Since it is already a function of y,

−3
−∫ 𝑑𝑦
𝐼=𝑒 𝑦

𝑑𝑦
3∫ 3
𝐼=𝑒 𝑦 = 𝑒 3 ln 𝑦 = 𝑒 ln 𝑦 = 𝑦 3

Multiplying the integrating factor to the given equation:

[𝑦 2 cos 𝑥 𝑑𝑥 + (4 + 5𝑦 sin 𝑥)𝑑𝑦 = 0]𝑦 3

𝑦 5 cos 𝑥 𝑑𝑥 + (4𝑦 3 + 5𝑦 4 sin 𝑥)𝑑𝑦 = 0

Compiled by: Kenneth D. Dugay, LPT | 60


Polytechnic University of the Philippines
Santa Rosa Campus

Solving for the solution of the exact DE:

∫ 𝑑𝐹 = 𝑀 𝑑𝑥 + 𝜙(𝑦)

𝐹 = ∫ 𝑦 5 cos 𝑥 𝑑𝑥 + 𝜙(𝑦)

𝐹 = 𝑦 5 sin 𝑥 + 𝜙(𝑦)

𝜕𝐹
= 5𝑦 4 sin 𝑥 + 𝜙′(𝑦)
𝜕𝑦

𝜕𝐹
= 𝑁 = 4𝑦 3 + 5𝑦 4 sin 𝑥
𝜕𝑦

5𝑦 4 sin 𝑥 + 𝜙 ′ (𝑦) = 4𝑦 3 + 5𝑦 4 sin 𝑥

𝜙 ′ (𝑦) = 4𝑦 3

∫ 𝜙 ′ (𝑦) = ∫ 4𝑦 3 𝑑𝑦

𝜙(𝑦) = 𝑦 4

Our solution is:

𝑦 5 sin 𝑥 + 𝑦 4 = 𝑐

Activities/Assessment
Obtain the general solution for the differential equations:

(1) 𝑥𝑑𝑦 = (𝑥𝑦 2 − 𝑦)𝑑𝑥

(2)𝑦(2𝑥 − 𝑦 + 1)𝑑𝑥 + 𝑥(3𝑥 − 4𝑦 + 3)𝑑𝑦 = 0

(3) (4𝑥𝑦 + 3𝑦 2 − 𝑥)𝑑𝑥 + 𝑥(𝑥 + 2𝑦)𝑑𝑦 = 0

(4) 𝑦(𝑥 + 𝑦 + 1)𝑑𝑥 + 𝑥(𝑥 + 3𝑦 + 2)𝑑𝑦 = 0

(5) (𝑥 + 𝑦) sin 𝑦 𝑑𝑥 + (𝑥 sin 𝑦 + cos 𝑦 )𝑑𝑦 = 0

Compiled by: Kenneth D. Dugay, LPT | 61


Polytechnic University of the Philippines
Santa Rosa Campus

Lesson 3
Linear Equations

Learning Objective
After successful completion of this module, you should be able to:

▪ transform a differential equation to its linear form; and


▪ solve for the solution of a differential equation in linear form.

Course Materials
Lecture Video

(Click the picture for the link or type in: https://bit.ly/LinearEq)

Click: Solved Problem #1 for Linear Equations: https://bit.ly/LinearEq1

Solved Problem #2 for Linear Equations: https://bit.ly/LinearEq2

Solved Problem #3 for Linear Equations: https://bit.ly/LinearEq3

Solved Problem #4 for Linear Equations: https://bit.ly/LinearEq4

Discussions
Linear Equations

Given an order one differential equation, a linear equation is in the form:

𝑑𝑦
+ 𝑃(𝑥)𝑦 = 𝑄(𝑥) (Linear in y)
𝑑𝑥

𝑑𝑥
+ 𝑃(𝑦)𝑥 = 𝑄(𝑦) (Linear in x)
𝑑𝑦

The integrating factor 𝐼 of the linear equation is given by:

𝐼 = 𝑒 ∫ 𝑃(𝑥)𝑑𝑥

Compiled by: Kenneth D. Dugay, LPT | 62


Polytechnic University of the Philippines
Santa Rosa Campus

We may solve it by multiplying the integrating factor to the given equation then obtaining the
solution of the now exact equation. This method is good but for this discussion, we can solve the solution
easily by applying the formulas given below:

𝑦𝑒 ∫ 𝑃(𝑥)𝑑𝑥 = ∫ 𝑄(𝑥)𝑒 ∫ 𝑃(𝑥)𝑑𝑥 𝑑𝑥 (Linear in y)

𝑥𝑒 ∫ 𝑃(𝑦)𝑑𝑦 = ∫ 𝑄(𝑦)𝑒 ∫ 𝑃(𝑦)𝑑𝑦 𝑑𝑦 (Linear in x)

Now, let us proceed to transforming our differential equation into its linear form.

Example 1. Transform the following differential equations in its linear form then identify its 𝑃 and 𝑄.

𝑑𝑦 = (𝑥 − 3𝑦)𝑑𝑥

Let us first check if it is linear in y:

𝑑𝑦
= 𝑥 − 3𝑦
𝑑𝑥

𝑑𝑦
+ 3𝑦 = 𝑥
𝑑𝑥

Since we already have a function of x multiplied to y in our second term in the left side of the equation,
and a function of x in the rights side, then our equation is linear in y with the form:

𝑑𝑦
+ 𝑃(𝑥)𝑦 = 𝑄(𝑥)
𝑑𝑥

Then our 𝑃 and 𝑄 are:

𝑃(𝑥) = 3

𝑄(𝑥) = 𝑥

Example 2. Transform the following differential equations in its linear form then identify its 𝑃 and 𝑄.

𝑦𝑑𝑥 + (3𝑥 − 𝑥𝑦 + 2)𝑑𝑦 = 0

Transforming it leading to linear in y will NOT yield to a form for linear equations as shown below:

𝑑𝑦 𝑦
+ =0
𝑑𝑥 3𝑥 − 𝑥𝑦 + 2

Let us try transforming in the form of linear in x:

𝑑𝑥
𝑦 + 3𝑥 − 𝑥𝑦 + 2 = 0
𝑑𝑦

Compiled by: Kenneth D. Dugay, LPT | 63


Polytechnic University of the Philippines
Santa Rosa Campus

𝑑𝑥
+ 3𝑥𝑦 −1 − 𝑥 + 2𝑦 −1 = 0
𝑑𝑦

𝑑𝑥
+ (3𝑦 −1 − 1)𝑥 = −2𝑦 −1
𝑑𝑦

Now it is in the form:

𝑑𝑥
+ 𝑃(𝑦)𝑥 = 𝑄(𝑦)
𝑑𝑦

Hence, it is linear in x with the values of 𝑃 and 𝑄 as:

𝑃(𝑦) = 3𝑦 −1 − 1

𝑄(𝑦) = −2𝑦 −1

Steps in Solving Linear Differential Equations

(1) Transform the given differential equation into either of the forms:

𝑑𝑦
+ 𝑃(𝑥)𝑦 = 𝑄(𝑥) (Linear in y)
𝑑𝑥

𝑑𝑥
+ 𝑃(𝑦)𝑥 = 𝑄(𝑦) (Linear in x)
𝑑𝑦

(2) Identify the values of 𝑃 and 𝑄

(3) Solve for the solution of the linear DE by applying the appropriate formula below:

𝑦𝑒 ∫ 𝑃(𝑥)𝑑𝑥 = ∫ 𝑄(𝑥)𝑒 ∫ 𝑃(𝑥)𝑑𝑥 𝑑𝑥 (Linear in y)

𝑥𝑒 ∫ 𝑃(𝑦)𝑑𝑦 = ∫ 𝑄(𝑦)𝑒 ∫ 𝑃(𝑦)𝑑𝑦 𝑑𝑦 (Linear in x)

Example 1. Obtain the general solution for the differential equations:

(𝑥 4 + 2𝑦)𝑑𝑥 − 𝑥𝑑𝑦 = 0

Transforming the given equation to linear in y:

𝑑𝑦
−𝑥 + 𝑥 4 + 2𝑦 = 0
𝑑𝑥

𝑑𝑦 2
+ (− ) 𝑦 = 𝑥 3
𝑑𝑥 𝑥

Compiled by: Kenneth D. Dugay, LPT | 64


Polytechnic University of the Philippines
Santa Rosa Campus

Now that we have a linear equation in y. Obtaining 𝑃(𝑥) and 𝑄(𝑥):

2
𝑃(𝑥) = − 𝑄(𝑥) = 𝑥 3
𝑥

Solution 1: Using Integrating Factor of the Linear Equation

Let us first solve for the solution of our differential equation by getting the integrating factor. Then we will
multiply it to our given equation so that we can make it an exact equation. The integrating factor of a
linear equation is defined by:

𝐼 = 𝑒 ∫ 𝑃(𝑥)𝑑𝑥
−2 𝑑𝑥 −2
𝐼 = 𝑒 ∫ 𝑥 𝑑𝑥 = ∫ 𝑒 −2 ∫ 𝑥 = ∫ 𝑒 −2 ln 𝑥 = ∫ 𝑒 ln 𝑥

𝐼 = 𝑥 −2

Multiplying 𝐼 to our linear equation:

𝑑𝑦 2
[ + (− ) 𝑦 = 𝑥 3 ] 𝑥 −2
𝑑𝑥 𝑥

𝑑𝑦
𝑥 −2 − 2𝑥 −3 𝑦 − 𝑥 = 0
𝑑𝑥

𝑥 −2 𝑑𝑦 − (2𝑥 −3 𝑦 + 𝑥)𝑑𝑥 = 0

(2𝑥 −3 𝑦 + 𝑥)𝑑𝑥 − 𝑥 −2 𝑑𝑦 = 0

Testing for exactness:

𝜕𝑀 𝜕 𝜕𝑁 𝜕
= (2𝑥 −3 𝑦 + 𝑥) = 2𝑥 −3 = (−𝑥 −2 ) = 2𝑥 −3
𝜕𝑦 𝜕𝑦 𝜕𝑥 𝜕𝑥

Now that our DE is already exact, we can now use the steps in solving for the solution of exact equations.
Picking 𝑁 for it is simpler:

∫ 𝑑𝐹 = 𝑁 𝑑𝑦 + 𝜙(𝑥)

𝐹 = ∫ −𝑥 −2 𝑑𝑦 + 𝜙(𝑥)

𝐹 = −𝑥 −2 𝑦 + 𝜙(𝑥)

𝜕𝐹
= 2𝑥 −3 𝑦 + 𝜙′(𝑥)
𝜕𝑥

𝜕𝐹
= 𝑀 = 2𝑥 −3 𝑦 + 𝑥
𝜕𝑥

2𝑥 −3 𝑦 + 𝜙′(𝑥) = 2𝑥 −3 𝑦 + 𝑥

Compiled by: Kenneth D. Dugay, LPT | 65


Polytechnic University of the Philippines
Santa Rosa Campus

𝜙 ′ (𝑥) = 𝑥

∫ 𝜙 ′ (𝑥) = ∫ 𝑥 𝑑𝑥

𝑥2
𝜙(𝑥) =
2

Our solution is:

𝑥2
−𝑥 −2 𝑦 + =𝑐
2

𝑥2
+ 𝑐 = 𝑥 −2 𝑦
2

(Take note that I still took the positive of c, since the negative of a constant is still a constant)

𝑥2
[ + 𝑐 = 𝑥 −2 𝑦] 𝑥 2
2

𝑥4
𝑦= + 𝑐𝑥 2
2

Solution 2: Direct Application of Formula for the Solution of Linear Equation

Solving for the solution,

𝑦𝑒 ∫ 𝑃(𝑥)𝑑𝑥 = ∫ 𝑄(𝑥)𝑒 ∫ 𝑃(𝑥)𝑑𝑥 𝑑𝑥

Let us solve for 𝑒 ∫ 𝑃(𝑥)𝑑𝑥 first:

2 𝑑𝑥 −2
𝑒 ∫ 𝑃(𝑥)𝑑𝑥 = 𝑒 ∫ −𝑥𝑑𝑥 = 𝑒 −2 ∫ 𝑥 = 𝑒 −2 ln 𝑥 = 𝑒 ln 𝑥

𝑒 ∫ 𝑃(𝑥)𝑑𝑥 = 𝑥 −2

Substituting it to our previous equation:

𝑦(𝑥 −2 ) = ∫ 𝑥 3 (𝑥 −2 )𝑑𝑥 + 𝑐

𝑥 −2 𝑦 = ∫ 𝑥𝑑𝑥 + 𝑐

𝑥2
𝑥 −2 𝑦 = +𝑐
2

𝑥4
𝑦= + 𝑐𝑥 2
2

Compiled by: Kenneth D. Dugay, LPT | 66


Polytechnic University of the Philippines
Santa Rosa Campus

Observe that we arrived at the same solution using the two methods but multiplying the
integrating factor then solving for the exact equations (Solution 1) will take a lot of time. So for our
succeeding problems in this module involving linear equations, we will just directly apply the formula
(Solution 2).

Example 2. Obtain the general solution for the differential equations:

(𝑥𝑦 2 + 𝑥 2 )𝑑𝑦 + 𝑥𝑦𝑑𝑥 = 0

By observing the given equation, it will not be linear in y. Then transforming it as linear in x:

𝑑𝑥
𝑥𝑦 + 𝑥𝑦 2 + 𝑥 2 = 0
𝑑𝑦

𝑑𝑥 1
+ ( ) 𝑥 = −𝑦
𝑑𝑦 𝑦

Using our linear equation in x:

1
𝑃(𝑦) = 𝑦 𝑄(𝑦) = −𝑦

Solving for the solution using the formula:

𝑥𝑒 ∫ 𝑃(𝑦)𝑑𝑦 = ∫ 𝑄(𝑦)𝑒 ∫ 𝑃(𝑦)𝑑𝑦 𝑑𝑦

𝑑𝑦
∫𝑦
𝑒 ∫ 𝑃(𝑦)𝑑𝑦 = 𝑒 = 𝑒 ln 𝑦

𝑒 ∫ 𝑃(𝑦)𝑑𝑦 = −𝑦

𝑥𝑦 = ∫ −𝑦(𝑦)𝑑𝑦 + 𝑐

𝑥𝑦 = − ∫ 𝑦 2 𝑑𝑦 + 𝑐

𝑦3
𝑥𝑦 = − +𝑐
3

3𝑥𝑦 + 𝑦 3 = 𝑐

Compiled by: Kenneth D. Dugay, LPT | 67


Polytechnic University of the Philippines
Santa Rosa Campus

Example 3. Obtain the general solution for the differential equations:

(3𝑥𝑦 + 3𝑦 − 4)𝑑𝑥 + (𝑥 + 1)2 𝑑𝑦 = 0

Setting it up as linear in y:

𝑑𝑦
(𝑥 + 1)2 + 3𝑦(𝑥 + 1) − 4 = 0
𝑑𝑥

𝑑𝑦 3 4
+( )𝑦 =
𝑑𝑥 𝑥+1 (𝑥 + 1)2

3 4
𝑃(𝑥) = 𝑥+1 𝑄(𝑥) = (𝑥+1)2

Solving for the solution:

𝑦𝑒 ∫ 𝑃(𝑥)𝑑𝑥 = ∫ 𝑄(𝑥)𝑒 ∫ 𝑃(𝑥)𝑑𝑥 𝑑𝑥

3 𝑑𝑥
𝑑𝑥 3
𝑒 ∫ 𝑃(𝑥)𝑑𝑥 = 𝑒 ∫𝑥+1 = 𝑒 3 ∫𝑥+1 = 𝑒 3 ln(𝑥+1) = 𝑒 ln(𝑥+1)

𝑒 ∫ 𝑃(𝑥)𝑑𝑥 = (𝑥 + 1)3

Substituting it to our formula:

4
𝑦(𝑥 + 1)3 = ∫ ( ) (𝑥 + 1)3 𝑑𝑥 + 𝑐
(𝑥 + 1)2

𝑦(𝑥 + 1)3 = 4 ∫(𝑥 + 1)𝑑𝑥 + 𝑐

𝑥2
𝑦(𝑥 + 1)3 = 4 ( ) + 4𝑥 + 𝑐
2

𝑦(𝑥 + 1)3 = 2𝑥 2 + 4𝑥 + 2 + 𝑐 (we let the arbitrary constant c be 2+c)

𝑦(𝑥 + 1)3 = 2(𝑥 2 + 2𝑥 + 1) + 𝑐

𝑦(𝑥 + 1)3 = 2(𝑥 + 1)2 + 𝑐

Solving for y by multiplying the whole equation by (𝑥 + 1)−3

𝑦 = 2(𝑥 + 1)−1 + 𝑐(𝑥 + 1)−3

Compiled by: Kenneth D. Dugay, LPT | 68


Polytechnic University of the Philippines
Santa Rosa Campus

Example 4. Obtain the general solution for the differential equations:

𝑦 ′ = 𝑥 − 2𝑦 cot 2𝑥

This equation is obviously linear in y. Transforming the equation:

𝑑𝑦
+ (2 cot 2𝑥)𝑦 = 𝑥
𝑑𝑥

𝑃(𝑥) = 2 cot 2𝑥 𝑄(𝑥) = 𝑥

𝑦𝑒 ∫ 𝑃(𝑥)𝑑𝑥 = ∫ 𝑄(𝑥)𝑒 ∫ 𝑃(𝑥)𝑑𝑥 𝑑𝑥

Since our integration process require some effort, let us take ∫ 𝑃(𝑥)𝑑𝑥 first

∫ 𝑃(𝑥)𝑑𝑥 = ∫ 2 cot 2𝑥 𝑑𝑥

cos 2𝑥
= ∫2 𝑑𝑥
sin 2𝑥

let: 𝑤 = sin 2𝑥 𝑑𝑤 = 2 cos 2𝑥 𝑑𝑥

𝑑𝑤
=∫ = ln 𝑤
𝑤

∫ 𝑃(𝑥)𝑑𝑥 = ln (sin 2𝑥)

𝑒 ∫ 𝑃(𝑥)𝑑𝑥 = 𝑒 ln(sin 2𝑥)

𝑒 ∫ 𝑃(𝑥)𝑑𝑥 = sin 2𝑥

Working on our solution:

𝑦 sin 2𝑥 = ∫ 𝑥 sin 2𝑥 𝑑𝑥 + 𝑐

Doing integration by parts on the right side of our equation:

let: 𝑢=𝑥 𝑑𝑣 = sin 2𝑥 𝑑𝑥


1
𝑑𝑢 = 𝑑𝑥 𝑣 = − cos 2𝑥
2

𝑥 1
𝑦 sin 2𝑥 = − cos 2𝑥 − ∫ − cos 2𝑥 𝑑𝑥 + 𝑐
2 2

𝑥 1
𝑦 sin 2𝑥 = − cos 2𝑥 + ∫ cos 2𝑥 𝑑𝑥 + 𝑐
2 2

Compiled by: Kenneth D. Dugay, LPT | 69


Polytechnic University of the Philippines
Santa Rosa Campus

𝑥 1 1
𝑦 sin 2𝑥 = − cos 2𝑥 + ( ) ∫ 2 cos 2𝑥 𝑑𝑥 + 𝑐
2 2 2

𝑥 1
𝑦 sin 2𝑥 = − cos 2𝑥 + sin 2𝑥 + 𝑐
2 4

4𝑦 sin 2𝑥 = −2𝑥 cos 2𝑥 + sin 2𝑥 + 𝑐

Activities/Assessment
Obtain the general solution for the differential equations:

(1) 𝑑𝑦 = (𝑥 − 3𝑦)𝑑𝑥

(2) (𝑥 − 2𝑦)𝑦 ′ = 𝑦 + 2(𝑥 − 2)3

(3) 𝑦 ′ + 𝑦 cot 𝑥 = 5𝑒 cos 𝑥

(4) 𝑥 ln 𝑦 𝑑𝑥 + (𝑥 − ln 𝑦)𝑑𝑦 = 0

(5) 𝑦 ′ = csc 𝑥 − 𝑦 cot 𝑥

Compiled by: Kenneth D. Dugay, LPT | 70


Polytechnic University of the Philippines
Santa Rosa Campus

Lesson 4
Bernoulli Equations

Learning Objective
After successful completion of this module, you should be able to:

▪ identify a Bernoulli equation based on its form;


▪ transform a Bernoulli equation into a linear equation; and
▪ solve for the solution of a Bernoulli equation.

Course Materials
Lecture Video

(Click the picture for the link or type in: https://bit.ly/BernoulliEq)

Click: Solved Problem #1 for Bernoulli Equations: https://bit.ly/ BernoulliEq1

Solved Problem #2 for Bernoulli Equations: https://bit.ly/ BernoulliEq2

Solved Problem #3 for Bernoulli Equations: https://bit.ly/ BernoulliEq3

Discussions
Bernoulli Equations

Bernoulli equation is a generalized case of the linear form. It contains a certain expression in
terms of the dependent variable which makes it distinct from the linear form.

A Bernoulli Equation is in the form:

𝑑𝑦
+ 𝑃(𝑥)𝑦 = 𝑦 𝑛 𝑄(𝑥)
𝑑𝑥

We will be doing some algebraic manipulations and apply some derivative concepts in our
Bernoulli equations. Our goal is to make it linear in form. Here are the steps that we will be taking to do it:

Compiled by: Kenneth D. Dugay, LPT | 71


Polytechnic University of the Philippines
Santa Rosa Campus

Steps in Solving Bernoulli Equations

(1) Transform the given equation to a Bernoulli equation in the form:

𝑑𝑦
+ 𝑃(𝑥)𝑦 = 𝑦 𝑛 𝑄(𝑥)
𝑑𝑥

(2) Multiply both sides of the equation to 𝑦 −𝑛 :

𝑑𝑦
𝑦 −𝑛 + 𝑃(𝑥)𝑦 −𝑛+1 = 𝑄(𝑥)
𝑑𝑥

(3) Let:

𝑧 = 𝑦 −𝑛+1

𝑑𝑧 = (−𝑛 + 1)𝑦 −𝑛 𝑑𝑦

𝑑𝑧
𝑦 −𝑛 𝑑𝑦 =
1−𝑛

(4) Substitute the values of 𝑦 −𝑛+1 and 𝑦 −𝑛 to our Bernoulli equation then it will be linear in form.

1 𝑑𝑧
( )+
1−𝑛 𝑑𝑥
𝑃(𝑥)𝑧 = 𝑄(𝑥) (linear in z)

(5) Solve for the solution using the formula for DE in linear form:

𝑧𝑒 ∫ 𝑃(𝑥)𝑑𝑥 = ∫ 𝑄(𝑥)𝑒 ∫ 𝑃(𝑥)𝑑𝑥 𝑑𝑥

Example 1. Obtain the general solution for the differential equation:

𝑑𝑦
− 𝑦 = 𝑥𝑦 2
𝑑𝑥

Our given equation is already a Bernoulli equation. Now let us multiple both sides of our equation by 𝑦 −2

𝑑𝑦
𝑦 −2 𝑑𝑥 − 𝑦 −1 = 𝑥 (1)

Let:

𝑧 = 𝑦 −1

𝑑𝑧 = −𝑦 −2 𝑑𝑦 => −𝑑𝑧 = 𝑦 −2 𝑑𝑦

Substituting these to (1):

−𝑑𝑧
−𝑧=𝑥
𝑑𝑥

Compiled by: Kenneth D. Dugay, LPT | 72


Polytechnic University of the Philippines
Santa Rosa Campus

𝑑𝑧
+ 𝑧 = −𝑥
𝑑𝑥

𝑃(𝑥) = 1 𝑄(𝑥) = −𝑥

Now that it is already linear in z, let us solve for its solution using:

𝑧𝑒 ∫ 𝑃(𝑥)𝑑𝑥 = ∫ 𝑄(𝑥)𝑒 ∫ 𝑃(𝑥)𝑑𝑥 𝑑𝑥

𝑧𝑒 ∫ 𝑑𝑥 = ∫ −𝑥𝑒 ∫ 𝑑𝑥 𝑑𝑥

𝑧𝑒 𝑥 = − ∫ 𝑥𝑒 𝑥 𝑑𝑥 + 𝑐

Doing integration by parts.

Let: 𝑢=𝑥 𝑑𝑣 = 𝑒 𝑥 𝑑𝑥

𝑑𝑢 = 𝑑𝑥 𝑣 = 𝑒𝑥

𝑧𝑒 𝑥 = − [𝑥𝑒 𝑥 − ∫ 𝑒 𝑥 𝑑𝑥] + 𝑐

𝑧𝑒 𝑥 = −𝑥𝑒 𝑥 + 𝑒 𝑥 + 𝑐

𝑧 = −𝑥 + 1 + 𝑐𝑒 −𝑥

Bringing it back in terms of x and y

𝑦 −1 = −𝑥 + 1 + 𝑐𝑒−𝑥

Example 2. Obtain the general solution for the differential equation:

(𝑥 2 + 𝑦 2 )𝑑𝑥 − 𝑥𝑦𝑑𝑦 = 0

First, let’s transform it into the form of a Bernoulli Equation.

𝑑𝑦
−𝑥𝑦 + 𝑥 2 + 𝑦2 = 0
𝑑𝑥

𝑑𝑦 1
+ (− ) 𝑦 = 𝑥𝑦 −1
𝑑𝑥 𝑥

Now that it is a Bernoulli equation, we multiply both sides by y:

𝑑𝑦 1
𝑦 + (− ) 𝑦 2 = 𝑥
𝑑𝑥 𝑥

Compiled by: Kenneth D. Dugay, LPT | 73


Polytechnic University of the Philippines
Santa Rosa Campus

Let: 𝑧 = 𝑦2
1
𝑑𝑧 = 2𝑦𝑑𝑦 => 𝑦𝑑𝑦 = 𝑑𝑧
2

Substituting it to our worked Bernoulli equation:

1 𝑑𝑧 1
( ) + (− ) 𝑧 = 𝑥
2 𝑑𝑥 𝑥

𝑑𝑧 2
+ (− ) 𝑧 = 2𝑥
𝑑𝑥 𝑥
2
𝑃(𝑥) = − 𝑥 𝑄(𝑥) = 2𝑥

Now that it is already linear in z, let us solve for its solution using:

𝑧𝑒 ∫ 𝑃(𝑥)𝑑𝑥 = ∫ 𝑄(𝑥)𝑒 ∫ 𝑃(𝑥)𝑑𝑥 𝑑𝑥

−2 𝑑𝑥 −2
)𝑑𝑥
𝑒 ∫ 𝑃(𝑥)𝑑𝑥 = 𝑒 ∫( 𝑥 = 𝑒 −2 ∫ 𝑥 = 𝑒 −2 ln 𝑥 = 𝑒 ln 𝑥

𝑒 ∫ 𝑃(𝑥)𝑑𝑥 = 𝑥 −2

Working on our solution:

𝑧𝑥 −2 = ∫ 2𝑥(𝑥 −2 )𝑑𝑥 + 𝑐

𝑑𝑥
𝑥 −2 𝑧 = 2 ∫ +𝑐
𝑥

𝑥 −2 𝑧 = 2 ln 𝑥 + ln 𝑐

𝑥 −2 𝑦 2 = ln 𝑐𝑥 2

𝑦 2 = 𝑥 2 ln 𝑐𝑥 2

Example 3. Obtain the general solution for the differential equation:

𝑑𝑦
+ 𝑦 = 𝑦 2 sin 𝑥
𝑑𝑥

Our equation is already in the Bernoulli form. Let us now multiply it to 𝑦 −2

𝑑𝑦
𝑦 −2 + 𝑦 −1 = sin 𝑥
𝑑𝑥

Let: 𝑧 = 𝑦 −1

𝑑𝑧 = −𝑦 −2 𝑑𝑦 => 𝑦 −2 𝑑𝑦 = −𝑑𝑧

Compiled by: Kenneth D. Dugay, LPT | 74


Polytechnic University of the Philippines
Santa Rosa Campus

𝑑𝑧
− + 𝑧 = sin 𝑥
𝑑𝑥

𝑑𝑧
+ (−1)𝑧 = −sin 𝑥
𝑑𝑥

𝑃(𝑥) = −1 𝑄(𝑥) = −sin 𝑥

Now solving for the solution of now linear DE:

𝑧𝑒 ∫ 𝑃(𝑥)𝑑𝑥 = ∫ 𝑄(𝑥)𝑒 ∫ 𝑃(𝑥)𝑑𝑥 𝑑𝑥

𝑧𝑒 ∫ −𝑑𝑥 = ∫ −sin 𝑥 𝑒 ∫ −𝑑𝑥 𝑑𝑥

𝑧𝑒 −𝑥 = − ∫ sin 𝑥 𝑒 −𝑥 𝑑𝑥 + 𝑐

Working on ∫ sin 𝑥 𝑒 −𝑥 𝑑𝑥 using integration by parts:

∫ sin 𝑥 𝑒 −𝑥 𝑑𝑥 = − sin 𝑥 𝑒 −𝑥 + ∫ cos 𝑥 𝑒 −𝑥 𝑑𝑥

∫ sin 𝑥 𝑒 −𝑥 𝑑𝑥 = − sin 𝑥 𝑒 −𝑥 − cos 𝑥 𝑒 −𝑥 − ∫ sin 𝑥 𝑒 −𝑥 𝑑𝑥

∫ sin 𝑥 𝑒 −𝑥 𝑑𝑥 + ∫ sin 𝑥 𝑒 −𝑥 𝑑𝑥 = −𝑒 −𝑥 (sin 𝑥 + cos 𝑥)

2 ∫ sin 𝑥 𝑒−𝑥𝑑𝑥 = −𝑒−𝑥 (sin 𝑥 + cos 𝑥)

1
∫ sin 𝑥 𝑒 −𝑥 𝑑𝑥 = − 𝑒 −𝑥 (sin 𝑥 + cos 𝑥)
2

Going back to our solution for our DE:

1
𝑧𝑒 −𝑥 = − (− 𝑒 −𝑥 (sin 𝑥 + cos 𝑥)) + 𝑐
2

1 −𝑥
𝑦 −1 𝑒−𝑥 = 𝑒 (sin 𝑥 + cos 𝑥) + 𝑐
2
1
𝑦 −1 = (sin 𝑥 + cos 𝑥) + 𝑐𝑒𝑥
2
2𝑦 −1 = sin 𝑥 + cos 𝑥 + 𝑐𝑒𝑥

Compiled by: Kenneth D. Dugay, LPT | 75


Polytechnic University of the Philippines
Santa Rosa Campus

Example 4. Obtain the general solution for the differential equation:

𝑥𝑑𝑦 − 𝑦𝑑𝑥 − 𝑥𝑦 3 𝑑𝑥 − 𝑥𝑦 3 ln 𝑥 𝑑𝑥 = 0

Making our given DE a Bernoulli equation:

𝑑𝑦
𝑥 − 𝑦 − 𝑥𝑦 3 − 𝑥𝑦 3 ln 𝑥 = 0
𝑑𝑥

𝑑𝑦 1
+ (− ) 𝑦 = 𝑦 3 + 𝑦 3 ln 𝑥
𝑑𝑥 𝑥

𝑑𝑦 1
+ (− ) 𝑦 = 𝑦 3 (1 + ln 𝑥)
𝑑𝑥 𝑥

Now that it is already a Bernoulli equation, let us multiply our equation by 𝑦 −3

𝑑𝑦 1
𝑦 −3 + (− ) 𝑦 −2 = 1 + ln 𝑥
𝑑𝑥 𝑥

Let: 𝑧 = 𝑦 −2

1
𝑑𝑧 = −2𝑦 −3 𝑑𝑦 => 𝑦 −3 𝑑𝑦 = − 2 𝑑𝑧

1 𝑑𝑧 1
(− ) + (− ) 𝑧 = 1 + ln 𝑥
2 𝑑𝑥 𝑥

𝑑𝑧 2
+ ( ) 𝑧 = −2(1 + ln 𝑥)
𝑑𝑥 𝑥
2
𝑃(𝑥) = 𝑥 𝑄(𝑥) = −2(1 + ln 𝑥)

Arriving at this linear equation, we will then get its solution using:

𝑧𝑒 ∫ 𝑃(𝑥)𝑑𝑥 = ∫ 𝑄(𝑥)𝑒 ∫ 𝑃(𝑥)𝑑𝑥 𝑑𝑥

2 𝑑𝑥 2
)𝑑𝑥
𝑒 ∫ 𝑃(𝑥)𝑑𝑥 = 𝑒 ∫(𝑥 = 𝑒 2 ∫ 𝑥 = 𝑒 2 ln 𝑥 = 𝑒 ln 𝑥

𝑒 ∫ 𝑃(𝑥)𝑑𝑥 = 𝑥 2

𝑧𝑥 2 = ∫ −2(1 + ln 𝑥)𝑥 2 𝑑𝑥 + 𝑐

.
𝑧𝑥 2 = −2 ∫(1 + ln 𝑥)𝑥 2 𝑑𝑥 + 𝑐

Compiled by: Kenneth D. Dugay, LPT | 76


Polytechnic University of the Philippines
Santa Rosa Campus

Doing integration by parts:

Let: 𝑢 = 1 + ln 𝑥 𝑑𝑣 = 𝑥 2 𝑑𝑥

1 𝑥3
𝑑𝑢 = 𝑥 𝑑𝑥 𝑣= 3

𝑥3 1
𝑧𝑥 2 = −2 [ (1 + ln 𝑥) − ∫ 𝑥 2 𝑑𝑥] + 𝑐
3 3

𝑥3 1 𝑥3
𝑧𝑥 2 = −2 [ (1 + ln 𝑥) − ( )] + 𝑐
3 3 3

2𝑥 3 2𝑥 3
𝑧𝑥 2 = − (1 + ln 𝑥) + +𝑐
3 9

Simplifying our solution:

2𝑥 2𝑥
𝑧=− (1 + ln 𝑥) + + 𝑐𝑥 −2
3 9

9𝑧 = −6𝑥(1 + ln 𝑥) + 2𝑥 + 𝑐𝑥 −2

9𝑦 −2 = −6𝑥(1 + ln 𝑥) + 2𝑥 + 𝑐𝑥 −2

Activities/Assessment
Obtain the general solution for the differential equations:

𝑑𝑦 𝑦
(1) + 𝑥 = 𝑥𝑦 2
𝑑𝑥

(2) 2𝑥 3 𝑦 ′ = 𝑦(𝑦 2 + 𝑥 2 )
1
3
(3) 𝑦 ′ − 𝑥 𝑦 = 𝑥 4 𝑦 3

(4) 𝑦 ′ tan 𝑥 sin 2𝑦 = sin2 𝑥 + cos2 𝑦

(5) 𝑦(𝑥 tan 𝑥 + ln 𝑦)𝑑𝑥 + tan 𝑥 𝑑𝑦 = 0

Compiled by: Kenneth D. Dugay, LPT | 77


Polytechnic University of the Philippines
Santa Rosa Campus

Lesson 5
Substitution Method

Learning Objective
After successful completion of this module, you should be able to:

▪ transform the equation by substitution to adhere to the conditions needed for a method in solving
order one differential equation to be applicable; and
▪ solve for the solution of a differential equation by substitution.

Course Materials
Lecture Videos

Click: Solved Problem #1 for Substitution Method: https://bit.ly/SubstitutionDE1

Solved Problem #2 for Substitution Method: https://bit.ly/SubstitutionDE2

Solved Problem #3 for Substitution Method: https://bit.ly/SubstitutionDE3

Solved Problem #4 for Substitution Method: https://bit.ly/SubstitutionDE4

Discussions
Substitution Method

Substitution method is used when the previous methods of solving for order one differential
equation does not apply. It means that we have to manipulate our equation in a way that it can be solved
using any of the previous methods. It is done by changing some part of the equation as other variables
(let u be equal to some terms). It is good if we are able arrive at a solvable equation right away. This
method requires us to think ahead and do some trials and see what manipulation works. Here, there are
no laid out steps to solve the equation so we are free to navigate it.

Example 1. Obtain the general solution for the differential equation:

− sin 𝑦 (𝑥 + sin 𝑦)𝑑𝑥 + 2𝑥 2 cos 𝑦 𝑑𝑦 = 0

This equation will not satisfy the conditions needed by our prior methods. This is why we have to do
something first. Let us observe those with the variable y. We have sin 𝑦 in 𝑀 while its derivative cos 𝑦 𝑑𝑦 is
in 𝑁. We can start working here by:

Let: 𝑢 = sin 𝑦 𝑑𝑢 = cos 𝑦 𝑑𝑦

−𝑢(𝑥 + 𝑢)𝑑𝑥 + 2𝑥 2 𝑑𝑢 = 0

Compiled by: Kenneth D. Dugay, LPT | 78


Polytechnic University of the Philippines
Santa Rosa Campus

Since we are left with two variables (one can be the independent while the other is dependent), we can
say that our substitution is successful. But we still have to look if a method to solve for its solution can be
applied.

−𝑢𝑥𝑑𝑥 − 𝑢2 𝑑𝑥 + 2𝑥 2 𝑑𝑢 = 0

Notice that every term has a degree two. Then our new equation is homogeneous. Since the terms
beside 𝑑𝑢 is simpler,

Let: 𝑢 = 𝑣𝑥 𝑑𝑢 = 𝑣𝑑𝑥 + 𝑥𝑑𝑣

−(𝑣𝑥)𝑥𝑑𝑥 − (𝑣𝑥)2 𝑑𝑥 + 2𝑥 2 (𝑣𝑑𝑥 + 𝑥𝑑𝑣) = 0

−𝑣𝑥 2 𝑑𝑥 − 𝑣 2 𝑥 2 𝑑𝑥 + 2𝑣𝑥 2 𝑑𝑥 + 2𝑥 3 𝑑𝑣 = 0

𝑣𝑥 2 𝑑𝑥 − 𝑣 2 𝑥 2 𝑑𝑥 + 2𝑥 3 𝑑𝑣 = 0

𝑥 2 (𝑣 − 𝑣 2 )𝑑𝑥 + 2𝑥 3 𝑑𝑣 = 0

𝑑𝑥 2𝑑𝑣
+ =0
𝑥 𝑣 − 𝑣2

𝑑𝑥 2𝑑𝑣
+ =0
𝑥 𝑣(1 − 𝑣)

Doing Partial Fractions:

2 𝐴 𝐵
= +
𝑣(1 − 𝑣) 𝑣 1 − 𝑣

1 = (1 − 𝑣)𝐴 + 𝐵𝑣

𝑣 1 : 0 = −𝐴 + 𝐵 => 𝐴=𝐵

𝑣0: 𝐵 = 2

𝐴=2

𝑑𝑥 2 2
+( + ) 𝑑𝑣 = 0
𝑥 𝑣 1−𝑣

Taking the integral of the whole equation:

𝑑𝑥 𝑑𝑣 𝑑𝑣
∫ + 2∫ + 2∫ =𝑐
𝑥 𝑣 1−𝑣

𝑑𝑥 𝑑𝑣 −𝑑𝑣
∫ + 2∫ − 2∫ =𝑐
𝑥 𝑣 1−𝑣

ln 𝑥 + 2 ln 𝑣 − 2 ln(1 − 𝑣) = ln 𝑐

Compiled by: Kenneth D. Dugay, LPT | 79


Polytechnic University of the Philippines
Santa Rosa Campus

Applying Rules of Logarithm:

ln 𝑥 + ln 𝑣 2 − ln(1 − 𝑣)2 = ln 𝑐

𝑥𝑣 2
ln = ln 𝑐
(1 − 𝑣)2

𝑥𝑣 2
ln 2
𝑒 (1−𝑣) = 𝑒 ln 𝑐

𝑥𝑣 2
=𝑐
(1 − 𝑣)2

Since,
𝑢
𝑢 = 𝑣𝑥 𝑣=𝑥

𝑢 2
𝑥( )
𝑥 =𝑐
𝑢 2
(1 − )
𝑥

Simplifying the equation:

𝑢2 (𝑥 − 𝑢)2
𝑥( ) = 𝑐 [ ]
𝑥2 𝑥2

𝑥𝑢2 = 𝑐(𝑥 − 𝑢)2

But,

𝑢 = sin 𝑦

𝑥 sin2 𝑦 = 𝑐(𝑥 − sin 𝑦)2

Example 2. Obtain the general solution for the differential equation:

𝑦(𝑥 tan 𝑥 + ln 𝑦)𝑑𝑥 + tan 𝑥 𝑑𝑦 = 0

𝑑𝑦
There is nothing that we can do with tan 𝑥 in 𝑀 and 𝑁. Looking at ln 𝑦, its derivative is
𝑦
. We can
1
produce it by multiplying both sides of our equation by
𝑦

𝑑𝑦
(𝑥 tan 𝑥 + ln 𝑦)𝑑𝑥 + tan 𝑥 =0
𝑦

Compiled by: Kenneth D. Dugay, LPT | 80


Polytechnic University of the Philippines
Santa Rosa Campus

Now that the derivative of ln 𝑦 exists on our second term,

𝑑𝑦
Let: 𝑢 = ln 𝑦 𝑑𝑢 = 𝑦

(𝑥 tan 𝑥 + 𝑢)𝑑𝑥 + tan 𝑥 𝑑𝑢 = 0

𝑑𝑢
tan 𝑥 + 𝑥 tan 𝑥 + 𝑢 = 0
𝑑𝑥

𝑑𝑢 𝑢
+𝑥+ =0
𝑑𝑥 tan 𝑥

𝑑𝑢
+ 𝑢 cot 𝑥 = −𝑥
𝑑𝑥

Now our equation is linear in u:

𝑃(𝑥) = cot 𝑥 𝑄(𝑥) = −𝑥

𝑢𝑒 ∫ 𝑃(𝑥)𝑑𝑥 = ∫ 𝑄(𝑥)𝑒 ∫ 𝑃(𝑥)𝑑𝑥 𝑑𝑥

𝑢𝑒 ∫ cot 𝑥𝑑𝑥 = ∫ −𝑥𝑒 ∫ cot 𝑥𝑑𝑥 𝑑𝑥

𝑢𝑒 ln sin 𝑥 = ∫ −𝑥𝑒 ln sin 𝑥 𝑑𝑥 + 𝑐

𝑢 sin 𝑥 = ∫ −𝑥 sin 𝑥 𝑑𝑥 + 𝑐

Doing integration by parts:

Let: 𝑢 = −𝑥 𝑑𝑣 = sin 𝑥 𝑑𝑥

𝑑𝑢 = −𝑑𝑥 𝑣 = − cos 𝑥

𝑢 sin 𝑥 = 𝑥 cos 𝑥 − ∫ cos 𝑥 𝑑𝑥 + 𝑐

𝑢 sin 𝑥 = 𝑥 cos 𝑥 − sin 𝑥 + 𝑐

Since,

𝑢 = ln 𝑦

ln 𝑦 sin 𝑥 = 𝑥 cos 𝑥 − sin 𝑥 + 𝑐

Compiled by: Kenneth D. Dugay, LPT | 81


Polytechnic University of the Philippines
Santa Rosa Campus

Example 3. Obtain the general solution for the differential equation:

𝑑𝑦
= (−5𝑥 + 𝑦)2 − 4
𝑑𝑥

For this equation, we will be taking a different approach. We will take the more complicated terms for us
to simplify the equation hoping that it will satisfy the conditions for our previous methods.

Let: 𝑢 = −5𝑥 + 𝑦

𝑑𝑢 𝑑𝑦 𝑑𝑦 𝑑𝑢
𝑑𝑥
= −5 + 𝑑𝑥 => 𝑑𝑥
= 𝑑𝑥 + 5

𝑑𝑢
+ 5 = 𝑢2 − 4
𝑑𝑥

𝑑𝑢 = (𝑢2 − 9)𝑑𝑥

𝑑𝑢
= 𝑑𝑥
𝑢2 − 9

𝑑𝑢
= 𝑑𝑥
(𝑢 − 3)(𝑢 + 3)

Now that our equation is separable, let us solve for its solution:

𝑑𝑢
∫ = ∫ 𝑑𝑥
(𝑢 − 3)(𝑢 + 3)

Using partial fractions:

1 𝐴 𝐵
= +
(𝑢 − 3)(𝑢 + 3) 𝑢 − 3 𝑢 + 3

1 = (𝑢 + 3)𝐴 + (𝑢 − 3)𝐵

1
𝑢1 : 0 = 𝐴 + 𝐵 𝑢0 : 1 = 3𝐴 − 3𝐵 𝐴=
6

𝐴 = −𝐵 1 = 3(−𝐵) − 3𝐵

1
𝐵=−
6

Substituting the values of A and B:

1 1

∫( 6 + 6 ) 𝑑𝑢 = ∫ 𝑑𝑥
𝑢−3 𝑢+3

1 𝑑𝑢 1 𝑑𝑢
∫ − ∫ = ∫ 𝑑𝑥
6 𝑢−3 6 𝑢+3

1 1
ln(𝑢 − 3) − ln(𝑢 + 3) = 𝑥 + 𝑐
6 6

Compiled by: Kenneth D. Dugay, LPT | 82


Polytechnic University of the Philippines
Santa Rosa Campus

Simplifying the equation:

ln(𝑢 − 3) − ln(𝑢 + 3) = 6𝑥 + 𝑐

𝑢−3
ln = 6𝑥 + 𝑐
𝑢+3
𝑢−3
𝑒 ln𝑢+3 = 𝑒 6𝑥+𝑐

𝑢−3
= 𝑒 6𝑥 𝑒 𝑐
𝑢+3

𝑢−3
= 𝑐𝑒 6𝑥
𝑢+3

Solving for u:

𝑢 − 3 = 𝑢𝑐𝑒 6𝑥 + 3𝑐𝑒 6𝑥

𝑢 − 𝑢𝑐𝑒 6𝑥 = 3𝑐𝑒 6𝑥 + 3

𝑢(1 − 𝑐𝑒 6𝑥 ) = 3(𝑐𝑒 6𝑥 + 1)

3(𝑐𝑒 6𝑥 + 1)
𝑢=
1 − 𝑐𝑒 6𝑥

But,

𝑢 = −5𝑥 + 𝑦

3(𝑐𝑒 6𝑥 + 1)
−5𝑥 + 𝑦 =
1 − 𝑐𝑒 6𝑥

3(𝑐𝑒 6𝑥 + 1)
𝑦= + 5𝑥
1 − 𝑐𝑒 6𝑥

Example 4. Obtain the general solution for the differential equation:

𝑦 ′ = 1 + 6𝑥𝑒 𝑥−𝑦

𝑑𝑦
= 1 + 6𝑥𝑒 𝑥−𝑦
𝑑𝑥

For this equation, it will be difficult for us to work through the variables in the exponent. To simplify this,

Let: 𝑢 =𝑥−𝑦

𝑑𝑢 𝑑𝑦 𝑑𝑦 𝑑𝑢
𝑑𝑥
= 1 − 𝑑𝑥 =>
𝑑𝑥
= 1 − 𝑑𝑥

𝑑𝑢
1− = 1 + 6𝑥𝑒 𝑢
𝑑𝑥

Compiled by: Kenneth D. Dugay, LPT | 83


Polytechnic University of the Philippines
Santa Rosa Campus

𝑑𝑢
= −6𝑥𝑒 𝑢
𝑑𝑥

𝑑𝑢 = −6𝑥𝑒 𝑢 𝑑𝑥

𝑒 −𝑢 𝑑𝑢 = −6𝑥𝑑𝑥

Since our equation is separable, we can get its solution:

∫ 𝑒 −𝑢 𝑑𝑢 = ∫ −6𝑥𝑑𝑥

− ∫ −𝑒 −𝑢 𝑑𝑢 = −6 ∫ 𝑥𝑑𝑥

−𝑒 −𝑢 = −3𝑥 2 + 𝑐

Since,

𝑢 =𝑥−𝑦

−𝑒 −(𝑥−𝑦) = −3𝑥 2 + 𝑐

𝑒 𝑦−𝑥 = 3𝑥 2 + 𝑐

ln 𝑒 𝑦−𝑥 = ln(3𝑥 2 + 𝑐)

𝑦 − 𝑥 = ln(3𝑥 2 + 𝑐)

𝑦 = 𝑥 + ln(3𝑥 2 + 𝑐)

Activities/Assessment
Obtain the general solution for the differential equations:

(1) 𝑦 ′ = √𝑥 + 𝑦 − 1

(2) 𝑦 ′ = (9𝑥 + 4𝑦 + 1)2

(3) (𝑒 4𝑦 − 𝑥)𝑑𝑥 = 4𝑒 4𝑦 (𝑒 4𝑦 + 𝑘𝑥)𝑑𝑦

(4) cos 𝑦 sin 2𝑥 𝑑𝑥 + (cos 2 𝑦 − cos 2 𝑥)𝑑𝑦 = 0

(5) cos 𝑦 (𝑥 + cos 𝑦)𝑑𝑥 + 2𝑥 2 sin 𝑦 𝑑𝑦 = 0

Compiled by: Kenneth D. Dugay, LPT | 84


Polytechnic University of the Philippines
Santa Rosa Campus

Unit 4: Applications of Differential Equations of Order One

Overview
Now that we are done with the methods to solve order one differential equations, we can now
apply these in real-life scenarios. Differential equations are concerned about change as discussed in the
first topic of this module. Then we can observe these changes more closely. Most of the time, our
variables are taken with respect to time but there are still many applications in Physics, Engineering and
even Business that we can explore.

Module Objective
After successful completion of this module, you should be able to:

▪ define and identify the properties of differential operators; and


▪ apply the properties of differential operators to solve differential equations.

Course Materials
Lecture Videos

Newton’s Law of Cooling

Click: Solved Problem #1 for Newton’s Law of Cooling: https://bit.ly/NewtonCool1

Solved Problem #2 for Newton’s Law of Cooling: https://bit.ly/NewtonCool2

Growth and Decay

Click: Solved Problem #1 for Substitution Method: https://bit.ly/GrowthDecay1

Solved Problem #2 for Substitution Method: https://bit.ly/GrowthDecay2

Solved Problem #2 for Substitution Method: https://bit.ly/GrowthDecay2

RL Circuits

Click: Solved Problem #1 for Substitution Method: https://bit.ly/RLinDE1

Compiled by: Kenneth D. Dugay, LPT | 85


Polytechnic University of the Philippines
Santa Rosa Campus

Discussions

Newton’s Law of Cooling

Newton’s Law of Cooling states that “the rate of change of the temperature of a cooling body is
proportional to the difference between the temperature of the body and the constant temperature of the
medium surrounding the body.”

This proportionality can be represented by:

𝑑𝑇
∝ (𝑇 − 𝑀)
𝑑𝑡
𝑑𝑇
Considering a proportionality constant −𝑘 (it is negative to represent as a cooling process)
𝑑𝑡

𝑑𝑇
= −k(𝑇 − 𝑀)
𝑑𝑡

where: 𝑇 = 𝑡𝑒𝑚𝑝𝑒𝑟𝑎𝑡𝑢𝑟𝑒 𝑜𝑓 𝑡ℎ𝑒 𝑏𝑜𝑑𝑦

𝑡 = 𝑡𝑖𝑚𝑒

𝑀 = 𝑐𝑜𝑛𝑠𝑡𝑎𝑛𝑡 𝑡𝑒𝑚𝑝𝑒𝑟𝑎𝑡𝑢𝑟𝑒 𝑜𝑓 𝑡ℎ𝑒 𝑚𝑒𝑑𝑖𝑢𝑚 𝑠𝑢𝑟𝑟𝑜𝑢𝑛𝑑𝑖𝑛𝑔 𝑏𝑜𝑑𝑦

𝑘 = 𝑝𝑟𝑜𝑝𝑜𝑟𝑡𝑖𝑜𝑛𝑎𝑙𝑖𝑡𝑦 𝑐𝑜𝑛𝑠𝑡𝑎𝑛𝑡

Deriving the general formula for Newton’s Law of Cooling,

𝑑𝑇
= −𝑘(𝑇 − 𝑀)
𝑑𝑡

𝑑𝑇
= −𝑘𝑑𝑡
𝑇−𝑀

𝑑𝑇
∫ = ∫ −𝑘𝑑𝑡
𝑇−𝑀

𝑑𝑇
∫ = −𝑘 ∫ 𝑑𝑡
𝑇−𝑀

ln(𝑇 − 𝑀) = −𝑘𝑡 + 𝑐

𝑒 ln(𝑇−𝑀) = 𝑒 −𝑘𝑡+𝑐

𝑇 − 𝑀 = 𝑒 −𝑘𝑡 𝑒 𝑐

𝑇 = 𝑐𝑒 −𝑘𝑡 + 𝑀

Considering the body temperature T as a function of time t:

𝑇(𝑡) = 𝑐𝑒 −𝑘𝑡 + 𝑀

This is our general formula for Newton’s Law of Cooling.

Compiled by: Kenneth D. Dugay, LPT | 86


Polytechnic University of the Philippines
Santa Rosa Campus

Example 1. At 12:00 midnight, with the temperature inside the warehouse at 70°F and the outside
temperature at 20°F, the furnace breaks down. Two hours later the temperature in the building has fallen
to 50°F. Determine when the temperatures in the warehouse fall to 40°F.

Given: 𝑇(0) = 70°F 𝑇(2) = 50°F

𝑀 = 20°F

Solve: 𝑡 𝑎𝑡 𝑇 = 40°F

Solution:

Starting at our derived formula for Newton’s Law of Cooling:

𝑇(𝑡) = 𝑐𝑒 −𝑘𝑡 + 𝑀

At 𝑡 = 0:

𝑇(0) = 𝑐𝑒 −𝑘𝑡 + 𝑀

70 = c𝑒−𝑘(0) + 20

c𝑒 0 = 70 − 20

𝑐 = 50

Then at 𝑡 = 2

𝑇(2) = 𝑐𝑒 −𝑘𝑡 + 𝑀

50 = 50𝑒 −𝑘(2) + 20

30 = 50𝑒 −2𝑘

30
𝑒 −2𝑘 =
50

3
ln 𝑒 −2𝑘 = ln
5
3
−2𝑘 = ln
5

1 3
𝑘 = − ln = 0.2554
2 5

Now solving for the time when the temperature is 40°F:

𝑇(40) = 𝑐𝑒 −𝑘𝑡 + 𝑀

40 = 50𝑒 −0.2554𝑡 + 20

Compiled by: Kenneth D. Dugay, LPT | 87


Polytechnic University of the Philippines
Santa Rosa Campus

50𝑒 −0.2554𝑡 = 20

20
𝑒 −0.2554𝑡 =
50
2
ln 𝑒 −0.2554𝑡 = ln
5
2
−0.2554𝑡 = ln
5
1 2
𝑡 = (− ) ln
0.2554 5

𝑡 = 3.5877 ℎ𝑜𝑢𝑟𝑠 or approximately 3 hours and 35.262 minutes.

∴ the warehouse will be 40°F after 3.5877 hours or 3 hours and 35.262 minutes.

Example 2. A body at unknown temperature is placed in a room which is held at a constant temperature
of 30°F. If after 10 minutes the temperature of the body is 0°F, and 15°F after 20 minutes, find the
unknown initial temperature.

Given: 𝑀 = 30°F 𝑇(10) = 0°F

𝑇(20) = 15°F

Solve: 𝑇(0)

Solution:

𝑇(𝑡) = 𝑐𝑒 −𝑘𝑡 + 𝑀

At 𝑡 = 0,

𝑇(0) = 𝑐𝑒 −𝑘𝑡 + 𝑀

𝑇(0) = 𝑐𝑒 −𝑘(0) + 30

𝑇(0) = 𝑐 + 30 (1)

At 𝑡 = 10,

𝑇(10) = 𝑐𝑒 −𝑘𝑡 + 𝑀

0 = 𝑐𝑒 −𝑘(10) + 30 => 𝑐𝑒 −10𝑘 = −30 (2)

Compiled by: Kenneth D. Dugay, LPT | 88


Polytechnic University of the Philippines
Santa Rosa Campus

At 𝑡 = 20,

𝑇(20) = 𝑐𝑒 −𝑘𝑡 + 𝑀

15 = 𝑐𝑒 −𝑘(20) + 30

𝑐𝑒 −20𝑘 = −15 (3)

Dividing (2) and (3):

𝑐𝑒 −10𝑘 −30
=
𝑐𝑒 −20𝑘 −15

𝑒 10𝑘 = 2

ln 𝑒 10𝑘 = ln 2

10𝑘 = ln 2

1
𝑘= ln 2
10

𝑘 = 0.0693

Solving for the value of 𝑐 using (2):

𝑐𝑒 −10(0.0693) = −30

−30
𝑐=
𝑒 0.693

𝑐 = −15.0022

Substituting the values to (1):

𝑇(0) = 𝑐 + 30

𝑇(0) = −15.0022 + 30

𝑇(0) = 14.9978°F

∴ the initial temperature of the body is 14.9978°F.

Compiled by: Kenneth D. Dugay, LPT | 89


Polytechnic University of the Philippines
Santa Rosa Campus

Growth and Decay

The rate of change of a substance is proportional to the amount of the substance. This can be
represented as:

𝑑𝑥
∝𝑥
𝑑𝑡

Considering a proportionality constant 𝑘,

𝑑𝑥
= 𝑘𝑥 (growth equation)
𝑑𝑡

𝑑𝑥
= −𝑘𝑥 (decay equation)
𝑑𝑡

where: 𝑥 = 𝑎𝑚𝑜𝑢𝑛𝑡 𝑜𝑓 𝑠𝑢𝑏𝑠𝑡𝑎𝑛𝑐𝑒

𝑡 = 𝑡𝑖𝑚𝑒

𝑘 = 𝑝𝑟𝑜𝑝𝑜𝑟𝑡𝑖𝑜𝑛𝑎𝑙𝑖𝑡𝑦 𝑐𝑜𝑛𝑠𝑡𝑎𝑛𝑡

Take note that 𝑘 is positive at the growth equation while it is negative at the decay equation.

Example 1. A culture initially has a number 𝑁0 of bacteria. At 𝑡 = 1 hour, the number of bacteria is
3
measured to be 𝑁0 . If the rate of growth is proportional to the number of bacteria present, determine the
2
time necessary for the number of bacteria to be a) double, b) triple.
3
Given: 𝑥(0) = 𝑁0 𝑥(1) = 𝑁0
2

Solve: 𝑡 𝑓𝑜𝑟 2𝑁0 and 𝑡 𝑓𝑜𝑟 3𝑁0

Solution:

Starting with the growth equation,

𝑑𝑥
= 𝑘𝑥
𝑑𝑡

𝑑𝑥
= 𝑘𝑑𝑡
𝑥

𝑑𝑥
∫ = ∫ 𝑘𝑑𝑡
𝑥

ln 𝑥 = 𝑘𝑡 + ln 𝑐

𝑒 ln 𝑥 = 𝑒 𝑘𝑡+ln 𝑐

𝑥 = 𝑒 𝑘𝑡 𝑒 ln 𝑐

𝑥 = 𝑐𝑒 𝑘𝑡

Compiled by: Kenneth D. Dugay, LPT | 90


Polytechnic University of the Philippines
Santa Rosa Campus

Applying the derived equation to our current problem:

𝑥(𝑡) = 𝑐𝑒 𝑘𝑡

At 𝑡 = 0:

𝑥(0) = 𝑐𝑒 𝑘𝑡

𝑁0 = 𝑐𝑒 𝑘(0)

𝑐 = 𝑁0

At 𝑡 = 1:

𝑥(1) = 𝑐𝑒 𝑘𝑡

3
𝑁 = 𝑁0 𝑒 𝑘(1)
2 0

Dividing both sides by 𝑁0

3
= 𝑒𝑘
2
3
ln = ln 𝑒 𝑘
2
3
𝑘 = ln = 0.4055
2

a) 𝑥 = 2𝑁0

𝑥(𝑡) = 𝑐𝑒 𝑘𝑡

2𝑁0 = 𝑁0 𝑒 0.4055𝑡

2 = 𝑒 0.4055𝑡

ln 2 = ln 𝑒 0.4055𝑡

ln 2 = 0.4055𝑡

ln 2
𝑡=
0.4055

𝑡 = 1.7094 ℎ𝑜𝑢𝑟𝑠

b) 𝑥 = 3𝑁0

𝑥(𝑡) = 𝑐𝑒 𝑘𝑡

3𝑁0 = 𝑁0 𝑒 0.4055𝑡

Compiled by: Kenneth D. Dugay, LPT | 91


Polytechnic University of the Philippines
Santa Rosa Campus

3 = 𝑒 0.4055𝑡

ln 3 = ln 𝑒 0.4055𝑡

ln 3 = 0.4055𝑡

ln 3
𝑡=
0.4055

𝑡 = 2.7093 ℎ𝑜𝑢𝑟𝑠

∴ it will take 1.7094 hours for the bacteria to double and 2.7093 hours for it to triple.

Example 2. A certain radioactive material is known to decay at a rate proportional to the amount present.
If initially there is 50 milligrams of the material present and after two hours, it is observed that the material
has lost 20% of its original mass, find a) an expression for the mass of the material remaining at any time,
b) the mass of the material after 5 hours; and c) the time at which material has decayed to one half of its
initial mass.

Given: 𝑥(0) = 50 𝑚𝑔 𝑥(2) = 50 − 0.20(50) = 40 𝑚𝑔

Solve: 𝑥(𝑡), 𝑥(5) , 𝑡 𝑜𝑓 0.5𝑥(0)

Solution:

Using the decay equation,

𝑑𝑥
= −𝑘𝑥
𝑑𝑡

𝑑𝑥
= −𝑘𝑑𝑡
𝑥

𝑑𝑥
∫ = ∫ 𝑘𝑑𝑡
𝑥

ln 𝑥 = −𝑘𝑡 + ln 𝑐

ln 𝑥 − ln 𝑐 = −𝑘𝑡

𝑥
ln = −𝑘𝑡
𝑐
𝑥
𝑒 ln 𝑐 = 𝑒 −𝑘𝑡

𝑥
= 𝑒 −𝑘𝑡
𝑐

𝑥 = 𝑐𝑒 −𝑘𝑡

Compiled by: Kenneth D. Dugay, LPT | 92


Polytechnic University of the Philippines
Santa Rosa Campus

At 𝑡 = 0:

𝑥(0) = 𝑐𝑒 −𝑘𝑡

50 = 𝑐𝑒 −𝑘(0)

𝑐 = 50

At 𝑡 = 2:

𝑥(2) = 𝑐𝑒 −𝑘𝑡

40 = 50𝑒 −𝑘(2)

40
𝑒 −2𝑘 =
50
4
ln 𝑒 −2𝑘 = ln
5

4
−2𝑘 = ln
5
1 4
𝑘 = − ln
2 5

𝑘 = 0.1116

a) The expression at any time 𝑡 can be represented as:

𝑥(𝑡) = 𝑐𝑒 −𝑘𝑡

𝑥(𝑡) = 50𝑒 −0.1116𝑡

b) The mass of the material after 5 hours:

𝑥(5) = 𝑐𝑒 −𝑘𝑡

𝑥(5) = 50𝑒 −0.1116(5)

𝑥(5) = 28.6176 𝑚𝑔

c) The time when the material is half its initial mass:

0.5𝑥(0) = 𝑐𝑒 −𝑘𝑡

0.5(50) = 50𝑒 −0.1116𝑡

0.5 = 𝑒 −0.1116𝑡

Compiled by: Kenneth D. Dugay, LPT | 93


Polytechnic University of the Philippines
Santa Rosa Campus

ln 𝑒 −0.1116𝑡 = ln 0.5

−0.1116𝑡 = ln 0.5

ln 0.5
𝑡=
−0.1116

𝑡 = 6.211 ℎ𝑜𝑢𝑟𝑠

∴ the expression for the mass of the material remaining at any time 𝑡 is 𝑥(𝑡) = 50𝑒 −0.1116𝑡 , the mass of the
material after 5 hours is 28.6176 mg, and it will take 6.211 hours for the material to decay to half of its
original mass.

Example 3. The population of a certain country is known to increase at a rate proportional to the number
of people presently living in the country. If after two years the population has doubled, and after three
years the population is 20,000, find the number of people initially living in the country.

Given: Let 𝑥0 be the initial number of people in the country:

𝑥(2) = 2𝑥0 𝑥(3) = 20,000

Solve: 𝑥(0)

Solution:

From our previous equations, solving the growth equation:

𝑑𝑥
= 𝑘𝑥
𝑑𝑡

𝑑𝑥
= 𝑘𝑑𝑡
𝑥
𝑑𝑥
∫ = ∫ 𝑘𝑑𝑡
𝑥

ln 𝑥 = 𝑘𝑡 + ln 𝑐

𝑒 ln 𝑥 = 𝑒 𝑘𝑡+ln 𝑐

𝑥 = 𝑒 𝑘𝑡 𝑒 ln 𝑐

𝑥 = 𝑐𝑒 𝑘𝑡

At 𝑡 = 0:

𝑥(0) = 𝑐𝑒 𝑘𝑡

𝑥0 = 𝑐𝑒 𝑘(0)

𝑐 = 𝑥0

Compiled by: Kenneth D. Dugay, LPT | 94


Polytechnic University of the Philippines
Santa Rosa Campus

At 𝑡 = 2:

𝑥(2) = 𝑐𝑒 𝑘𝑡

2𝑥0 = 𝑥0 𝑒 2𝑘

𝑒 2𝑘 = 2

ln 𝑒 2𝑘 = ln 2

2𝑘 = ln 2

ln 2
𝑘= = 0.3466
2

At 𝑡 = 3:

𝑥(3) = 𝑐𝑒 𝑘𝑡

20 000 = 𝑥0 𝑒 (0.3466)(3)

20 000
𝑥0 =
𝑒 1.0398

𝑥0 = 7 070.5076 ≈ 7 071 𝑝𝑒𝑜𝑝𝑙𝑒

∴ there are 7 071 people initially living in the country

Compiled by: Kenneth D. Dugay, LPT | 95


Polytechnic University of the Philippines
Santa Rosa Campus

Series RL Circuits (Additional application for ECE students)

Given this series RL circuit, a constant voltage 𝑉 is applied when the switch is closed. As per Ohm’s Law,
the voltage 𝑉𝑅 across the resistor 𝑅 with the current 𝑖 is:

𝑉𝑅 = 𝑖𝑅

and the voltage 𝑉𝐿 across the inductor 𝐿 is given by:

𝑑𝑖
𝑉𝐿 = 𝐿
𝑑𝑡

Applying Kirchhoff’s Voltage Law across the circuit, we arrive at the differential equation for a series RL
circuit:

𝑑𝑖
𝑉 = 𝑖𝑅 + 𝐿
𝑑𝑡

Example 1. An RL circuit has 𝑉 = 5 𝑉, a resistance of 50 𝛺, and an inductance of 1 𝐻. If the switch is


initially open, a) find the current of the circuit at any time 𝑡, and b) the current at 𝑡 = 0.05 𝑠.

Given: 𝑉 = 5 𝑉 𝑅 = 50 𝛺

𝐿 = 1𝐻 𝑖(0) = 0 𝐴

Solve: 𝑖(𝑡), 𝑖(0.05)

Solution:

a) Starting with our differential equation for series RL circuit:

𝑑𝑖
𝑉 = 𝑖𝑅 + 𝐿
𝑑𝑡

𝑑𝑖
5 = 50𝑖 + (1)
𝑑𝑡

𝑑𝑖
+ 50𝑖 = 5
𝑑𝑡

Our equation is now linear in 𝑖

𝑃(𝑡) = 50 𝑄(𝑡) = 5

Compiled by: Kenneth D. Dugay, LPT | 96


Polytechnic University of the Philippines
Santa Rosa Campus

Solving for the linear DE:

𝑖𝑒 ∫ 𝑃(𝑡)𝑑𝑡 = ∫ 𝑄(𝑡)𝑒 ∫ 𝑃(𝑡)𝑑𝑡 𝑑𝑡

𝑖𝑒 ∫ 50𝑑𝑡 = ∫ 5𝑒 ∫ 50𝑑𝑡 𝑑𝑡

𝑖𝑒 50𝑡 = 5 ∫ 𝑒 50𝑡 𝑑𝑡 + 𝑐

1
𝑖𝑒 50𝑡 = 5 ( ) ∫ 50𝑒 50𝑡 𝑑𝑡 + 𝑐
50

𝑖𝑒 50𝑡 = 0.10𝑒 50𝑡 + 𝑐

Dividing both sides by 𝑒 50𝑡 :

𝑖 = 0.10 + 𝑐𝑒 −50𝑡 (1)

Solving for 𝑐, at 𝑡 = 0, 𝑖 = 0 (switch was initially open)

0 = 0.10 + 𝑐𝑒 −50(0)

𝑐 = −0.10

Substituting the value of 𝑐 to (1):

𝑖 = 0.10 + (−0.10)𝑒 −50𝑡

𝑖(𝑡) = 0.10(1 − 𝑒 −50𝑡 )

b) Solving for 𝑖(0.05)

𝑖(0.05) = 0.10(1 − 𝑒 −50(0.05) )

𝑖(0.05) = 0.0918 𝐴

∴ the current of the circuit at any time 𝑡 can be represented as 𝑖(𝑡) = 0.10(1 − 𝑒 −50𝑡 ) and the current at
0.05 second is 0.0918 A.

Compiled by: Kenneth D. Dugay, LPT | 97


Polytechnic University of the Philippines
Santa Rosa Campus

Activities/Assessment
Solve for the value(s) being asked in the following problems:

(1) A thermometer reading 75°F is taken out where the temperature is 20°F. The reading is 30°F after 4
minutes. Find a) the temperature reading 7 minutes after the thermometer was brought outside, and b)
the time taken for the reading to drop from 75°F to within half a degree of the air temperature (20.5°F)

(2) A body at temperature 0°F is placed in a room whose temperature is kept at 100°F. If after 10 minutes
the temperature of the body is 25°F, find a) the time required for the body to reach a temperature of 50°F,
and b) the temperature of the body after 20 minutes.

(3) If the population of a country doubles in 50 years, in how many years will it triple under the assumption
that the rate of increase is proportional to the number of inhabitants.

(4) A certain radioactive material is known to decay at a rate proportional to the amount present. If after
one hour it is observed that 10% of the material has decayed, find the half-life of the material.

(5) The amount 𝑆, to which an original principal amount 𝑆0 will be after 𝑡 years, is represented by the
differential equation where 𝑟 is the rate of interest when compounded continuously:

𝑑𝑆
= 𝑟𝑆
𝑑𝑡

Determine when an amount of ₱10,000 that is invested at a rate of 5% compounded continuously will
become ₱20,000.

*(6) [For ECE students only] A series RL circuit with R = 50 Ω and L = 10 H has a constant voltage
V = 100 V applied at t = 0 by the closing of a switch. Find a) the equation for 𝑖 at any time 𝑡, and b) the
current at t = 0.5 s.

Compiled by: Kenneth D. Dugay, LPT | 98


Polytechnic University of the Philippines
Santa Rosa Campus

Unit 5: Differential Operators

Overview
This last unit of our module will now give you a broader tool to solve for differential equations that
is not limited to order one. We will be introducing the concept of differential operators for us to solve the
solution of higher order differential equations. First, we will be talking about its properties so that we can
be familiar in the use of this new operator.

There are different forms that we need to consider in solving equations involving differential
operators. We will be considering different cases of DE and know what the proper approach is to solve for
its solution. Our process will not be as labor intensive as our previous discussions, but it will require us to
know new properties and theorems.

Lesson 1
Properties of Differential Operators

Module Objective
After successful completion of this module, you should be able to:

▪ define and identify the properties of differential operators; and


▪ apply the theorem for exponential shift to solve differential equations.

Course Materials
Lecture Videos
Click: Solved Problem #1 for Exponential Shift: https://bit.ly/ExpShift1

Solved Problem #2 for Exponential Shift: https://bit.ly/ExpShift2

Discussions
Differential Operators

The differential operator 𝐷 is another notation for differentiation that we will use. Given the
derivative of 𝑦 with respect to 𝑥, we can represent it as:

𝑑𝑦
First Derivative: 𝐷𝑦 = 𝑑𝑥

𝑑2 𝑦
Second Derivative: 𝐷2 𝑦 = 𝑑𝑥 2

Compiled by: Kenneth D. Dugay, LPT | 99


Polytechnic University of the Philippines
Santa Rosa Campus

𝑑3 𝑦
Third Derivative: 𝐷3 𝑦 = 𝑑𝑥 3

In general, we can represent our differential operator as:

𝑑𝑛 𝑦
𝐷𝑛 𝑦 =
𝑑𝑥 𝑛

for a dependent variable 𝑦 and independent variable 𝑥.

For us to be more familiar with our new notation, let us use it with the notation that we are familiar with.

𝑑𝑦
1) (𝑥𝐷 + 2)𝑦 = 𝑥𝐷𝑦 + 2𝑦 = 𝑥 + 2𝑦
𝑑𝑥

𝑑2 𝑦
2) (𝐷2 − 1)𝑦 = 𝐷2 𝑦 − 𝑦 = −𝑦
𝑑𝑥 2

𝑑2𝑦 𝑑𝑦 𝑑𝑦
3) (𝑥𝐷 + 3)(𝐷 − 2)𝑦 = 𝑥𝐷 2 𝑦 − 2𝑥𝐷𝑦 + 3𝐷𝑦 − 6𝑦 = 𝑥 2
− 2𝑥 +3 − 6𝑦
𝑑𝑥 𝑑𝑥 𝑑𝑥

Properties of Differential Operators

Property 1. 𝑓(𝐷) = 𝑎0 𝐷𝑛 + 𝑎1 𝐷𝑛−1 + ⋯ + 𝑎𝑛−1 𝐷 + 𝑎𝑛

If 𝐷 denotes the operator which is a differentiation with respect to 𝑥, then we will have the
equation above which is the linear operator of order 𝑛.

Property 2. 𝐷 𝑘 𝑒 𝑚𝑥 = 𝑚𝑘 𝑒 𝑚𝑥

Let us demonstrate the property for 𝑚 = 3

𝐷3 𝑒 𝑚𝑥 = 𝐷2 (𝐷𝑒 𝑚𝑥 )

Taking the derivative with respect to 𝑥:

𝐷3 𝑒 𝑚𝑥 = 𝐷2 (𝑚𝑒 𝑚𝑥 )

= 𝐷(𝐷𝑚𝑒 𝑚𝑥 )

= 𝐷𝑚2 𝑒 𝑚𝑥

𝐷3 𝑒 𝑚𝑥 = 𝑚3 𝑒 𝑚𝑥

Then the property is true for 𝑚 = 3.

Compiled by: Kenneth D. Dugay, LPT | 100


Polytechnic University of the Philippines
Santa Rosa Campus

Property 3. 𝑓(𝐷)𝑒 𝑚𝑥 = 𝑒 𝑚𝑥 𝑓(𝑚)

Proof:

Using Property 1 for 𝑓(𝐷):

𝑓(𝐷)𝑒 𝑚𝑥 = (𝑎0 𝐷𝑛 + 𝑎1 𝐷𝑛−1 + ⋯ + 𝑎𝑛−1 𝐷 + 𝑎𝑛 )𝑒𝑚𝑥

𝑓(𝐷)𝑒 𝑚𝑥 = 𝑎0 (𝐷𝑛 𝑒𝑚𝑥 ) + 𝑎1 (𝐷 𝑛−1 𝑒𝑚𝑥 ) + ⋯ + 𝑎𝑛−1 (𝐷 𝑒𝑚𝑥 ) + 𝑎𝑛 𝑒𝑚𝑥

Using Property 2 for terms inside the parentheses:

𝑓(𝐷)𝑒 𝑚𝑥 = 𝑎0 𝑚𝑛 𝑒𝑚𝑥 + 𝑎1 𝑚𝑛−1 𝑒𝑚𝑥 + ⋯ + 𝑎𝑛−1 𝑚𝑒𝑚𝑥 + 𝑎𝑛 𝑒𝑚𝑥

𝑓(𝐷)𝑒 𝑚𝑥 = 𝑒𝑚𝑥 (𝑎0 𝑚𝑛 + 𝑎1 𝑚𝑛−1 + ⋯ + 𝑎𝑛−1 𝑚 + 𝑎𝑛 )

Observe that we can treat 𝑚 inside the parenthesis as a polynomial function 𝑓(𝑚) the same way we
applied it to 𝐷 in Property 1:

𝑓(𝐷)𝑒 𝑚𝑥 = 𝑒𝑚𝑥 𝑓(𝑚) ■

Property 4. If 𝑚 is a root of the auxiliary equation defined by 𝑓(𝑚) = 0, then

𝑓(𝐷)𝑒 𝑚𝑥 = 0

Note than the auxiliary equation is obtained by replacing 𝐷 by 𝑚 in 𝑓(𝐷). This gives 𝑒 𝑚𝑥 as a
solution for the homogeneous equation 𝑓(𝐷)𝑦 = 0 => 𝑓(𝐷)𝑒 𝑚𝑥 = 0.

Property 5. If 𝑓(𝐷) is a polynomial in terms of the operator 𝐷, then:

(𝐷 − 𝑎)𝑒 𝑎𝑥 𝑦 = 𝑒 𝑎𝑥 𝐷𝑦

This is called the exponential shift as expanded in the following theorem:

Exponential Shift

Theorem 1. From the Property 5 of Differential Operators, let 𝐹(𝐷) be a polynomial in 𝐷, then:

𝑓(𝐷 − 𝑎)(𝑒 𝑎𝑥 𝑦) = 𝑒 𝑎𝑥 𝑓(𝐷)𝑦

and can be generalized into higher orders as:

(𝐷 − 𝑎)𝑛 𝑒 𝑎𝑥 𝑦 = 𝑒 𝑎𝑥 𝐷𝑛 𝑦

Compiled by: Kenneth D. Dugay, LPT | 101


Polytechnic University of the Philippines
Santa Rosa Campus

Example 1. Use exponential shift to solve the differential equation:

(𝐷 + 2)3 𝑦 = 0

First, let us multiply the equation by 𝑒 raised to the 𝑥 times the constant added to the differential operator.
This is done to prepare the equation for the exponential shift. Doing that, multiplying the equation by 𝑒 2𝑥 :

𝑒 2𝑥 (𝐷 + 2)3 𝑦 = 0

Using our exponential shift equation:

(𝐷 − 𝑎)𝑛 𝑒 𝑎𝑥 𝑦 = 𝑒 𝑎𝑥 𝐷 𝑛 𝑦

𝑒 𝑎𝑥 𝐷𝑛 𝑦 = (𝐷 − 𝑎)𝑛 𝑒 𝑎𝑥 𝑦

Now our 𝑎 = 2 and 𝐷 + 2 ⟶ 𝐷

𝑒 2𝑥 (𝐷 + 2)3 𝑦 = [(𝐷 + 2) − 2]3 (𝑒 2𝑥 𝑦) = 0

𝐷3 (𝑒 2𝑥 𝑦) = 0

Integrating three times for the order of the operator is 3:

𝐷3 (𝑒 2𝑥 𝑦) = 0

𝐷2 (𝑒 2𝑥 𝑦) = 𝑐1

𝐷(𝑒 2𝑥 𝑦) = 𝑐1 𝑥 + 𝑐2

𝑒 2𝑥 𝑦 = 𝑐1 𝑥 2 + 𝑐2 𝑥 + 𝑐3

𝑦 = 𝑒 −2𝑥 (𝑐1 𝑥 2 + 𝑐2 𝑥 + 𝑐3 )

Example 2. Use exponential shift to solve the differential equation:

(𝐷 − 5)4 𝑦 = 0

Multiplying by 𝑒 −5𝑥 :

𝑒 −5𝑥 (𝐷 − 5)4 𝑦 = 0

Applying the exponential shift equation with our 𝑎 = −5 and 𝐷 − 5 ⟶ 𝐷

𝑒 𝑎𝑥 𝐷𝑛 𝑦 = (𝐷 − 𝑎)𝑛 𝑒 𝑎𝑥 𝑦

𝑒 −5𝑥 (𝐷 − 5)4 = [(𝐷 − 5) − (−5)]4 (𝑒 −5𝑥 𝑦) = 0

(𝐷)4 (𝑒 −5𝑥 𝑦) = 0

Compiled by: Kenneth D. Dugay, LPT | 102


Polytechnic University of the Philippines
Santa Rosa Campus

Integrating four times:

𝐷4 (𝑒 −5𝑥 𝑦) = 0

𝐷3 (𝑒 −5𝑥 𝑦) = 𝑐1

𝐷2 (𝑒 −5𝑥 𝑦) = 𝑐1 𝑥 + 𝑐2

𝐷(𝑒 −5𝑥 𝑦) = 𝑐1 𝑥 2 + 𝑐2 𝑥 + 𝑐3

𝑒 −5𝑥 𝑦 = 𝑐1 𝑥 3 + 𝑐2 𝑥 2 + 𝑐3 𝑥 + 𝑐4

𝑦 = 𝑒 5𝑥 (𝑐1 𝑥 3 + 𝑐2 𝑥 2 + 𝑐3 𝑥 + 𝑐4 )

Activities/Assessment
Use exponential shift to solve the differential equations:

(1) (𝐷 − 9)2 𝑦 = 0

(2) (𝐷 + 4)3 𝑦 = 0

(3) (𝐷 − 3)4 𝑦 = 0

(4)(𝐷 + 1)5 𝑦 = 0

(5) (2𝐷 − 1)2 𝑦 = 0

Compiled by: Kenneth D. Dugay, LPT | 103


Polytechnic University of the Philippines
Santa Rosa Campus

Lesson 2
Solutions of Homogeneous Linear Equations with Real Roots

Module Objective
After successful completion of this module, you should be able to:

▪ identify the real roots of an auxiliary equation; and


▪ solve for the solution of general homogeneous linear equations with real roots.

Course Materials
Lecture Videos

Distinct Real Roots of Auxiliary Equations

Click: Solved Problem #1 for Distinct Real Roots of Auxiliary Equations: https://bit.ly/DistinctReal1

Solved Problem #2 for Distinct Real Roots of Auxiliary Equations: https://bit.ly/DistinctReal2

Solved Problem #3 for Distinct Real Roots of Auxiliary Equations: https://bit.ly/DistinctReal3

Repeated Real Roots of Auxiliary Equations

Click: Solved Problem #1 for Repeated Real Roots of Auxiliary Equations: https://bit.ly/RepReal1

Solved Problem #2 for Repeated Real Roots of Auxiliary Equations: https://bit.ly/RepReal2

Discussions

The form of a general linear homogeneous equation with constant coefficient is given by:

𝑑𝑛 𝑦 𝑑 𝑛−1 𝑦 𝑑𝑦
𝑎0 𝑛
+ 𝑎1 𝑛−1
+ ⋯ + 𝑎𝑛−1 + 𝑎𝑛 𝑦 = 0
𝑑𝑥 𝑑𝑥 𝑑𝑥

This form may be written as 𝑓(𝐷)𝑦 = 0 where 𝑓(𝐷) is a linear differential operator as stated in Property 1
of differential operators.

For example, we can define order two linear homogeneous equation as:

𝑎𝑦 ′′ + 𝑏𝑦 ′ + 𝑐𝑦 = 0

or in differential operator form:

𝐷2 𝑦 + 𝐷𝑦 + 𝑐𝑦 = 0

Compiled by: Kenneth D. Dugay, LPT | 104


Polytechnic University of the Philippines
Santa Rosa Campus

Distinct Real Roots of Auxiliary Equation

Let the roots of the auxiliary equation 𝑓(𝑚) = 0 be 𝑚1 , 𝑚2 , … , 𝑚𝑘 . Then these will produce 𝑘
solutions as:

𝑦1 = 𝑒 𝑚1 𝑥 , 𝑦2 = 𝑒 𝑚2 𝑥 , … , 𝑦𝑘 = 𝑒 𝑚𝑘 𝑥

Then the solution, denoted by 𝑦𝑐 , of the general linear homogeneous equations with 𝑚1 , 𝑚2 , … , 𝑚𝑘
roots of the auxiliary equation is:

𝑦𝑐 = 𝑐1 𝑒𝑚1𝑥 + 𝑐2 𝑒𝑚2𝑥 + ⋯ + 𝑐3 𝑒𝑚3𝑥

Example 1. Solve for the general solution of the differential equation:

𝑦 ′′ + 𝑦 ′ − 2𝑦 = 0

Changing it to its differential operator form:

𝐷2 𝑦 + 𝐷𝑦 − 2𝑦 = 0

then as an auxiliary equation 𝑓(𝑚),

𝑓(𝑚) = 𝑚2 + 𝑚 − 2 = 0

(𝑚 + 2)(𝑚 − 1) = 0

The roots are:

𝑚1 = −2 and 𝑚2 = 1

Now the solutions 𝑦𝑘 = 𝑒 𝑚𝑘 𝑥 are:

𝑦1 = 𝑒 −2𝑥 and 𝑦2 = 𝑒 𝑚2𝑥

Following the general linear homogeneous equations, the solution of our differential equation is:

𝑦𝑐 = 𝑐1 𝑒 −2𝑥 + 𝑐2 𝑒 𝑥

Example 2. Solve for the general solution of the differential equation:

𝑦 ′′′ − 𝑦 ′′ − 2𝑦′ = 0

𝐷3 𝑦 − 𝐷2 𝑦 − 2𝐷𝑦 = 0

As an auxiliary equation 𝑓(𝑚),

𝑓(𝑚) = 𝑚3 − 𝑚2 − 2𝑚 = 0

Compiled by: Kenneth D. Dugay, LPT | 105


Polytechnic University of the Philippines
Santa Rosa Campus

Solving for the roots:

(𝑚)(𝑚 − 2)(𝑚 + 1) = 0

The roots are:

𝑚1 = −1 ; 𝑚2 = 0 and 𝑚3 = 2

(Note that I changed the arrangements of the roots from lowest to highest. I just did it to organize
the roots so we will arrive at a uniform final answer for easier checking of output)

Now the solutions 𝑦𝑘 = 𝑒 𝑚𝑘 𝑥 are:

𝑦1 = 𝑒 −𝑥 ; 𝑦2 = 𝑒 (0)𝑥 = 1 ; 𝑦3 = 𝑒 2𝑥

Following the general linear homogeneous equations, the solution of our differential equation is:

𝑦𝑐 = 𝑐1 𝑒 −𝑥 + 𝑐2 + 𝑐3 𝑒 2𝑥

Example 3. Solve for the general solution of the differential equation:

(𝐷2 + 2𝐷 − 1)𝑦 = 0

𝑓(𝑚) = 𝑚2 + 2𝑚 − 1 = 0

Since we cannot solve its roots by trial and error, let us use quadratic equation:

−2 ± √(2)2 − 4(1)(−1)
𝑚=
2(1)

−2 ± √8 −2 ± √4(2) −2 2√2
= = = ±
2 2 2 2

𝑚1 = −1 + √2 and 𝑚2 = −1 − √2

Now the solutions 𝑦𝑘 = 𝑒 𝑚𝑘 𝑥 are:

𝑦1 = 𝑒 (−1+√2)𝑥 ; 𝑦2 = 𝑒 (−1−√2)𝑥

The general solution is:

𝑦𝑐 = 𝑐1 𝑒 (−1+√2)𝑥 + 𝑐2 𝑒 (−1−√2)𝑥

Compiled by: Kenneth D. Dugay, LPT | 106


Polytechnic University of the Philippines
Santa Rosa Campus

Repeated Real Roots of Auxiliary Equation

In our previous section, we talked about the general solution of a homogeneous linear equations
with distinct roots. Now, we will consider the case wherein the roots are repeated.

To solve for the solution of auxiliar equations with repeated roots, we will be using the concept of
exponential shift as discussed in the previous module. For example, the root will be repeated twice:

𝑒 −𝑎𝑥 (𝐷 − 𝑎)2 𝑦 = 0

Applying exponential shift:

𝐷2 (𝑒 −𝑎𝑥 𝑦) = 0

To get its solution, we take the derivative twice:

𝐷2 (𝑒 𝑎𝑥 𝑦) = 𝐷[𝐷(𝑒 −𝑎𝑥 𝑦)] = 0

𝐷(𝑒 −𝑎𝑥 𝑦) = 𝑐1

𝑒 −𝑎𝑥 𝑦 = 𝑐1 𝑥 + 𝑐2

Interchanging 𝑐1 and 𝑐2 for us to observe easier the pattern of increasing exponent of 𝑥

𝑦 = 𝑐1 𝑒 𝑎𝑥 + 𝑐2 𝑥𝑒 𝑎𝑥

Notice that the number of repeated roots, now represented as the degree of the differential operator after
the exponential shift, determines the number of arbitrary constants in the solution. We already did this in
our previous lesson. Then the variable 𝑥 𝑘−1 multiplied to the arbitrary constant where 𝑘 is the degree of
the differential operator.

As an example, following the pattern above, the solution of a differential operator of degree three is equal
to:

𝑦 = 𝑐1 𝑒 𝑎𝑥 + 𝑐2 𝑥𝑒 𝑎𝑥 + 𝑐3 𝑥 2 𝑒 𝑎𝑥

Now let us apply this concept in differential operators with distinct and repeated root at the same time.

Example 1. Solve for the general solution of the differential equation:

𝑑 3 𝑦 𝑑 2 𝑦 𝑑𝑦
− − +𝑦 =0
𝑑𝑥 3 𝑑𝑥 2 𝑑𝑥

𝐷3 𝑦 − 𝐷2 𝑦 − 𝐷𝑦 + 𝑦 = 0

𝑓(𝑚) = 𝑚3 − 𝑚2 − 𝑚 + 1 = 0

Compiled by: Kenneth D. Dugay, LPT | 107


Polytechnic University of the Philippines
Santa Rosa Campus

Solving for the roots:

𝑚2 (𝑚 − 1) − (𝑚 − 1) = 0

We grouped the equation this way to factor out 𝑚 − 1

(𝑚2 − 1)(𝑚 − 1) = 0

(𝑚 + 1)(𝑚 − 1)(𝑚 − 1) = 0

(𝑚 + 1)(𝑚 − 1)2 = 0

The roots are:

𝑚1 = −1 ; 𝑚2 = 1 and 𝑚3 = 1

Notice that our root 1 was repeated twice. Then, our solutions are:

𝑦1 = 𝑒 −𝑥 ; 𝑦2 = 𝑒 𝑥 = 1 ; 𝑦3 = 𝑥𝑒 𝑥

We included a variable 𝑥 in our 𝑦3 for our root 1 is repeated. Then our general solution is:

𝑦𝑐 = 𝑐1 𝑒 −𝑥 + 𝑐2 𝑒 𝑥 + 𝑐3 𝑥𝑒 𝑥

Example 2. Solve for the general solution of the differential equation:

(4𝐷4 − 4𝐷3 − 23𝐷 2 + 12𝐷 + 36)𝑦 = 0

Then the auxiliary equation is:

4𝑚4 − 4𝑚3 − 23𝑚2 + 12𝑚 + 36 = 0

Since we cannot see the roots immediately, let us apply synthetic division:

4 −4 −23 12 36 2
8 8 −30 −36
4 4 −15 −18 0
Applying another synthetic division:

4 4 −15 18 2
8 24 −18
4 12 9 0
Now, the roots of the remaining terms can be solved using trial and error:

(𝑚 − 2)(𝑚 − 2)(4𝑚2 + 12𝑚 + 9) = 0

(𝑚 − 2)2 (2𝑚 + 3)(2𝑚 + 3) = 0

(𝑚 − 2)2 (2𝑚 + 3)2 = 0

Compiled by: Kenneth D. Dugay, LPT | 108


Polytechnic University of the Philippines
Santa Rosa Campus

The roots of the auxiliary equation are:

3 3
𝑚1 = − 2 ; 𝑚2 = − 2 ; 𝑚3 = 2 ; and 𝑚4 = 2

Then our solutions are:


3 3
𝑦1 = 𝑒 −2𝑥 ; 𝑦2 = 𝑥𝑒 −2𝑥 ; 𝑦3 = 𝑒 2𝑥 ; 𝑦4 = 𝑥𝑒 2𝑥

Finally, our general solution will be:

3 3
𝑦𝑐 = 𝑐1 𝑒 −2𝑥 + 𝑐2 𝑥𝑒 −2𝑥 + 𝑐3 𝑒 2𝑥 + 𝑐4 𝑥𝑒 2𝑥

Activities/Assessment
Solve for the general solution of the differential equations:

(1) (𝐷2 − 5𝐷 + 4)𝑦 = 0

(2) (6𝐷2 − 5𝐷 − 6)𝑦 = 0

(3) (𝐷3 − 4𝐷2 + 4𝐷)𝑦 = 0

(4)(𝐷4 − 𝐷3 )𝑦 = 0

(5) (4𝐷 4 + 4𝐷3 − 3𝐷2 − 2𝐷 + 1)𝑦 = 0

Compiled by: Kenneth D. Dugay, LPT | 109


Polytechnic University of the Philippines
Santa Rosa Campus

Lesson 3
Solutions of Homogeneous Linear Equations with
Imaginary Roots

Module Objective
After successful completion of this module, you should be able to:

▪ use Euler’s Formula to convert exponential form of complex numbers to polar form;
▪ identify the imaginary roots of an auxiliary equation; and
▪ solve for the solution of general homogeneous linear equations with imaginary roots.

Course Materials
Lecture Videos
Distinct Imaginary Roots of Auxiliary Equations

Click: Solved Problem #1 for Distinct Imaginary Roots of Auxiliary Equations: https://bit.ly/DistinctIm1

Solved Problem #2 for Distinct Real Roots of Auxiliary Equations: https://bit.ly/DistinctIm2

Repeated Imaginary Roots of Auxiliary Equations

Click: Solved Problem #1 for Repeated Imaginary Roots of Auxiliary Equations: https://bit.ly/RepIm1

Solved Problem #2 for Repeated Imaginary Roots of Auxiliary Equations: https://bit.ly/RepIm2

Discussions
Euler’s Formula

Euler’s Formula for complex numbers is defined as:

𝑒 𝑖𝜃 = cos 𝜃 + 𝑖 sin 𝜃

Let 𝑎 + 𝑏𝑖 be some complex number. Then,

𝑒 (𝑎+𝑏𝑖)𝑥 = 𝑒 𝑎𝑥 𝑒 𝑏𝑖𝑥

Using Euler’s Formula:

𝑒 (𝑎+𝑏𝑖)𝑥 = 𝑒 𝑎𝑥 (cos 𝑏𝑥 + 𝑖 sin 𝑏𝑥)

Next, we will be applying this concept on auxiliary equations with imaginary roots.

Compiled by: Kenneth D. Dugay, LPT | 110


Polytechnic University of the Philippines
Santa Rosa Campus

Distinct Imaginary Roots of Auxiliary Equation

Let 𝑎 + 𝑏𝑖 and 𝑎 − 𝑏𝑖 be the complex roots of an auxiliary equation. Then the solutions are (using 𝑛 as the
arbitrary constant first):

𝑦1 = 𝑛1 𝑒 (𝑎+𝑏𝑖)𝑥 ; 𝑦2 = 𝑛2 𝑒 (𝑎−𝑏𝑖)𝑥

Then the general solution is:

𝑦𝑐 = 𝑛1 𝑒 (𝑎+𝑏𝑖)𝑥 + 𝑛2 𝑒 (𝑎−𝑏𝑖)𝑥

Applying Euler’s formula:

𝑦𝑐 = 𝑛1 [𝑒 𝑎𝑥 (cos 𝑏𝑥 + 𝑖 sin 𝑏𝑥)] + 𝑛2 [𝑒 𝑎𝑥 (cos −𝑏𝑥 + 𝑖 sin −𝑏𝑥)]

But:

𝑒 −𝑖𝑎 = cos −𝑎 + 𝑖 sin −𝑎 = cos 𝑎 − sin 𝑎

Applying this in our equation:

𝑦𝑐 = 𝑛1 [𝑒 𝑎𝑥 (cos 𝑏𝑥 + 𝑖 sin 𝑏𝑥)] + 𝑛2 [𝑒 𝑎𝑥 (cos 𝑏𝑥 − 𝑖 sin 𝑏𝑥)]

𝑦𝑐 = 𝑒 𝑎𝑥 [𝑛1 (cos 𝑏𝑥 + 𝑖 sin 𝑏𝑥) + 𝑛2 (cos 𝑏𝑥 − 𝑖 sin 𝑏𝑥)]

𝑦𝑐 = 𝑒 𝑎𝑥 [𝑛1 cos 𝑏𝑥 + 𝑛1 𝑖 sin 𝑏𝑥 + 𝑛2 cos 𝑏𝑥 − 𝑛2 𝑖 sin 𝑏𝑥]

𝑦𝑐 = 𝑒 𝑎𝑥 [(𝑛1 + 𝑛2 )(cos 𝑏𝑥) + (𝑛1 − 𝑛2 )(𝑖 sin 𝑏𝑥)]

Let: 𝑐1 = 𝑛1 + 𝑛2 and 𝑐2 = (𝑛1 − 𝑛2 )𝑖

Then the general solution for distinct imaginary roots of the auxiliary equation of homogeneous linear
equation is:

𝑦𝑐 = 𝑒 𝑎𝑥 (𝑐1 cos 𝑏𝑥 + 𝑐2 sin 𝑏𝑥)

Example 1. Solve for the general solution of the differential equation:

(𝐷2 − 𝐷 + 2)𝑦 = 0

Taking the roots of the auxiliary equation:

𝑓(𝑚) = 𝑚2 − 𝑚 + 2 = 0

Using quadratic equation:

−(−1) ± √(−1)2 − 4(1)(2)


𝑚=
2(1)

Compiled by: Kenneth D. Dugay, LPT | 111


Polytechnic University of the Philippines
Santa Rosa Campus

1 ± √−7
𝑚=
2

1 ± √7(−1)
𝑚=
2

The rectangular form 𝑎 + 𝑏𝑖 of the complex root of the auxiliary equation is:

1 𝑖√7
𝑚= ±
2 2

where:

1 √7
𝑎= and 𝑏 =
2 2

Then the general solution of the auxiliary equation is:

𝑦𝑐 = 𝑒 𝑎𝑥 (𝑐1 cos 𝑏𝑥 + 𝑐2 sin 𝑏𝑥)

1 √7 √7
𝑦𝑐 = 𝑒 2𝑥 (𝑐1 cos 𝑥 + 𝑐2 sin 𝑥)
2 2

Example 2. Solve for the general solution of the differential equation:

(𝐷4 − 1)𝑦 = 0

Obtaining the roots of the auxiliar equation:

𝑓(𝑚) = 𝑚4 − 1 = 0

(𝑚2 − 1)(𝑚2 + 1) = 0

𝑚2 − 1 = 0 𝑚2 + 1 = 0

𝑚2 = 1 𝑚2 = −1

𝑚 = √1 𝑚 = √−1

𝑚 = ±1 𝑚 = ±𝑖

The roots are 𝑚1 = −1, 𝑚2 = 1, 𝑚3,4 = ±𝑖

We have distinct real roots in 𝑚1 and 𝑚2 so we will be applying the concepts in the previous lesson to get
its solution. For 𝑚3 and 𝑚4 , these are distinct imaginary roots in the form 𝑎 + 𝑏𝑖 but 𝑎 = 0. So the general
solution of the auxiliary equation is:

𝑦𝑐 = 𝑐1 𝑒 −𝑥 + 𝑐2 𝑒 𝑥 + 𝑒 (0)𝑥 (𝑐3 cos 𝑥 + 𝑐4 sin 𝑥)

𝑦𝑐 = 𝑐1 𝑒 −𝑥 + 𝑐2 𝑒 𝑥 + 𝑐3 cos 𝑥 + 𝑐4 sin 𝑥

Compiled by: Kenneth D. Dugay, LPT | 112


Polytechnic University of the Philippines
Santa Rosa Campus

Repeated Imaginary Roots of Auxiliary Equation

The last case for homogeneous linear differential equations is whose auxiliary equations have
repeated imaginary roots. Here, we will still be applying Euler’s formula as per the previous case. But we
will be multiplying 𝑥 𝑘−1 for every 𝑘 repetition of roots. This is done using the concept of exponential shift.
You may also refer to the discussion for repeated real roots of auxiliary equations in the previous lesson.

For example, we have the complex root 𝑚 = 1 ± 2𝑖 repeated twice. Then its solution is:

𝑦𝑐 = 𝑒 𝑥 (𝑐1 cos 2𝑥 + 𝑐2 sin 2𝑥) + 𝑒 𝑥 (𝑐3 𝑥 cos 𝑥 + 𝑐4 𝑥 sin 𝑥)

𝑦𝑐 = 𝑒 𝑥 (𝑐1 + 𝑐2 𝑥) cos 𝑥 + 𝑒 𝑥 (𝑐2 + 𝑐4 𝑥) sin 𝑥

Example 1. Solve for the general solution of the differential equation:

(𝐷4 + 6𝐷 2 + 9)𝑦 = 0

Getting the roots of the auxiliary equation:

𝑓(𝑚) = 𝑚4 + 6𝑚2 + 9 = 0

(𝑚2 + 3)(𝑚2 + 3) = 0

(𝑚2 + 3)2 = 0

The roots are:

𝑚1 = ±√3𝑖, 𝑚2 = ±√3𝑖

Since we have a repeated imaginary root with 𝑎 = 0 and 𝑏 = √3, the solution is:

𝑦𝑐 = 𝑒 (0)𝑥 (𝑐1 cos √3𝑥 + 𝑐2 sin √3𝑥) + 𝑒 (0)𝑥 (𝑐3 𝑥 cos √3𝑥 + 𝑐4 𝑥 sin √3𝑥)

𝑦𝑐 = 𝑐1 cos √3𝑥 + 𝑐2 sin √3𝑥 + 𝑐3 𝑥 cos √3𝑥 + 𝑐4 𝑥 sin √3𝑥

𝑦𝑐 = (𝑐1 + 𝑐3 𝑥) cos √3𝑥 + (𝑐2 + 𝑐4 𝑥) sin √3𝑥

Example 2. Solve for the general solution of the differential equation:

(𝐷2 + 16)3 𝑦 = 0

For the roots of the auxiliary equation:

𝑓(𝑚) = (𝑚2 + 16)3 = 0

𝑚2 + 16 = 0

𝑚1,2,3 = √−16 = ±4𝑖

Compiled by: Kenneth D. Dugay, LPT | 113


Polytechnic University of the Philippines
Santa Rosa Campus

With the roots as:

𝑚1 = ±4𝑖 , 𝑚2 = ±4𝑖 , and 𝑚3 = ±4𝑖

With 𝑎 = 0 and 𝑏 = 4, the general solution for the homogeneous linear equation with imaginary roots
repeated thrice is:

𝑦𝑐 = 𝑒 (0)𝑥 (𝑐1 cos 4𝑥 + 𝑐2 sin 4𝑥) + 𝑒 (0)𝑥 (𝑐3 𝑥 cos 4𝑥 + 𝑐4 𝑥 sin 4𝑥) + 𝑒 (0)𝑥 (𝑐5 𝑥 2 cos 4𝑥 + 𝑐6 𝑥 2 sin 4𝑥)

𝑦𝑐 = 𝑐1 cos 4𝑥 + 𝑐2 sin 4𝑥 + 𝑐3 𝑥 cos 4𝑥 + 𝑐4 𝑥 sin 4𝑥 + 𝑐5 𝑥 2 cos 4𝑥 + 𝑐6 𝑥 2 sin 4𝑥

𝑦𝑐 = (𝑐1 + 𝑐3 𝑥 + 𝑐5 𝑥 2 ) cos 4𝑥 + (𝑐2 + 𝑐4 𝑥 + 𝑐6 𝑥 2 ) sin 4𝑥

Activities/Assessment
Solve for the general solution of the differential equations:

(1) (𝐷2 − 2𝐷 + 2)𝑦 = 0

(2) (𝐷3 +2𝐷2 + 𝐷 + 2)𝑦 = 0

(3) (𝐷3 +7𝐷2 + 19𝐷 + 13)𝑦 = 0

(4) (𝐷3 + 100)2 𝑦 = 0

(5) (𝐷6 + 18𝐷4 + 81𝐷2 )𝑦 = 0

Compiled by: Kenneth D. Dugay, LPT | 114


Polytechnic University of the Philippines
Santa Rosa Campus

References

• Elementary Differential Equations (Free Ed.), William Trench, Trinity University, 2013
• Elementary Differential Equations, Roberto Asin, Jose Sy, Merriam & Webster Bookstore
Inc, 1984
• Differential Equations (2008 Ed.), Sergio Ymas Jr., Ymas Publishing House, 2007.

Compiled by: Kenneth D. Dugay, LPT | 115

You might also like